PHARMACOLOGY

¡Supera tus tareas y exámenes ahora con Quizwiz!

Mefloquine (Lariam) belongs to a class of drugs which is useful in treating which of the following condition Malaria AIDS Hepatitis Cancer Chlamydia

'Malaria falaria is a devastating parasitic disease transmitted through the bite ofinfected Anophe- les mosquitoes. Endemic to tropical and subtropical areas ofAsia, North and South Amer- ica. the Middle East, North Africa, and the South Pacific. Plasmodium vivax is the most common of four human malaria species (P falciparum, malariae, ovale, and vivax). P. r ir ax causes up to 65% of malaria in India and is becoming increasingly resistant to malaria drugs. By contrast, P falciparum is the most deadly species and the subject of most malaria-related research and literature. \fefloquine (Lariam) four malaria species. has been shown to be effective in treating malaria caused by all Other antimalarial drugs are: . . . . . Chloroquin (Aralen) Combination ofatovaquone and proguanil (Malarone) Halofantrine Quinine Conrbination of sulfadoxine and pyrirnethamine (Fansidar)

What are some of the adverse side effects of local anesthetics

-{dlerse effects caused by local anesthetics are categorized into two major grottps: L Efltcts resulting from toxicity L E11'ects resulting from allergy Toricities: . Too much anesthetic in the bloodstream can cause toxicities to the central nervous system /Cr\t and cardiovascular system . . Opioids can increase the systemic toxicity oflocal anesthetics Esters will show greater toxicity in patients with a hereditary deficiency of plasna es- terases. Remember: Esters are metabolized by the plasma enzyme plasma pseudo- cholinesterase, which is a plasma esterase. . The CNS effects include: restlessness, stimulation. tremors. convulsive seizures followed by CNS depression, slowed respiration and even coma . The cardiovascular effects include: bradycardia and reduction ofcardiac outpr.rt Allergy-': . Hypersensitivities or allergic reactions to local anesthetics, particularly from the amides, are rare. These reactions are manifested as dermatological reactions and edema at the in- jection . . site. Asthmatic wheezing syndromes have occurred in response to local anesthetic injections Allergic reactions are more prevalent with the ester-type rather than the amide{ype anesthetics. They also display cross-allergenicity. Ester local anesthetic allergic manifestations include: nasolabial swelling, itching, and oral mucosal swelling Ilethylparaben, which was used as a perservative, can a)so cause allergies

Name the 8 classes of drugs used in cancer chemotherapy.

1. Alkylating agents - these agents alkylate DNA such that it cannot replicate. Popular agents include Cisplatin and Cyclophosphamide, 2. Anthracyclines these agents destroy DNA such that the cell cannot replicate. Popular agents include Daunorubicin and Doxorubicin. 3. .{ntibiotics - these antibiotics are not used for antibacterial therapy but were specifically designed for cancer chemotherapy. The agent in this class is Dactinomycin. -1. -{ntimetabolites - these agents interfere which selected biochemical reactions necessary tbr cell growth. Popular agents are s-Fluorouracil (5-FU), 6-Mercaptopurine and \Iethotrexate, 5. -\ntimicrotubular - affects the microtubular assembly with cells to inhibit cell mitosis. The popular agent is Paelitaxel (Taxol). 6. -{ntiestrogen these agents block the tumors on which estrogen has a stimulatory effect. The popular agent is Tamoxifen (Nolvadex). ? \'inca Alkaloids \-incristine. - these are mitotic spindle poisons. Examples include Vinblastin€ and 8. Gonadotropin hormone-releasing antigen - these inhibit gonadotropin secretion, an action \\,hich is effective in reducing certain carcinomas. The popular agent is Leuprolide. \ote: The anticancer drugs Asparaginase and Interferons do not fall within any category but are used in the treatment ofcertain cancers. Asparaginase deprives tumor cells ofcertain amino acids such that protein production is blocked. Interferons boost the immune system. Agents used in treating breast canc€r: . . Antiestrogens: tamoxtfen Aromatase inhibitors: letrozole Q''lolvadex), fulvesttant (Fasloder), andtorcmifene /Feuara) and exernestar\e (Aromasi ) (Fqreston).

The maximum dose of Articaine (septocaine) thats recommended in one appointment is expressed as mg/kg body weight as opposed to total milligrams. Whats the maximum recommended dose for children and adults for articaine 7 mg/kg 100 mg/kg I mg/kg 300 mg/kg

7 mg/kg In a tlpical 70 kg adult male, the dose of 7 mghg would equate to a total of490 mgs. Thus. the maximum recommended amount of articaine that could be given to a 70 kg adult in one appointment is 490 mgs. The table belo*' shows the following number of dental cartridges containing 1.7 ml vof ume of solution to provide the indicated amounts of articaine hydrochioride 4% and epinenhrine l:100.000.

The average time for ths onset of drug effect after subcutaneous administration is: Immediate 5 minutes 15 minutes 30 minutes

15 minutes Routes of Drug Administration: . Oral route: It takes approximately 30 minutes for onset of drug effect after drug is swallowed. Oral route allows for use ofmany different dosage forms including tablets, capsules and liquids. . Intramuscular injection: Onset of action of drugs injected into muscle occurs rap- idly (approximately 5 minutes) because ofhigh blood flow lhrough the muscles. . Subcutaneous injection: Onset ofaction ofdrugs injected under the skin takes about l5 rninutes. . Inhalation: Gases such as nitrous oxide are absorbed rapidly though the lungs and gain access to the general circulation within 5 minutes. . Topical: This route includes ointments and creams applied to the skin and mucous membranes. It is not intended for syslemic drug administration. . Patch delivery: Onset ofaction ofdrugs in skin patches is about 15 minutes and sim- ilar to subcutaneous injection. Skin patches release drug into the blood stream over a l2 to 24 hour Deriod.

The maximal recomended adult dose of lidocaine is 300mg. How many mL of 2% lidocaine need to be given to reach this level

15mL lmportant information to solve qucstions . . ofthis type: There arc 20 mg ofLidocaine in every milliliter of2%o Lidocaine 3n0 mg (maximal recommended dose) - 20 ng (in every mL) = 15 mLs . . 20 mg x 1.8 mL 300 mg - ffu every carpule) = 36 mg / I carpule 36 69 = 3.3 carpttles (8 cdrpules of 224 Lldocaine can be used) Remember; Lidocaine has serious drug interactions with beta-blockers and cim€tidine that de- crease lidocaine clearance 3070 or more. Propranolol, metoprolol, and nadolol are reported to re- ducc lidocaine clearance due to the decreasg in cardiac output caused by thc beta-blockers. Decreased cardiac output rcsults in reduced liver blood flow which explains the decline in Iido- carne clearance caused by these drugs. Cimetidine also decreases lidocaine clearance, but thc mcchanism ofthe interaction is different. Because cimetidine does not change liver blood llow. it is believed that cimetidine decreases lidocaine clearance by inhibiting hepatic microsomal en- z1mes. Lidocaine clearancc may be accelerated by concomitant use ofphenobarbital or phenytoin. Bolh ofthese agents are known to be hepatic drug metabolizing drug inducers, and this is lhe Drobable mcchanism oftheir druc interaction with lidocaine.

A dental anesthetic carpule contains 1.8 ml of a 2% solution of lidocaine with 1:100,000 epinephrine. How much lidocaine and epinephrine does the carpule contain? 3.6 mg lidocaine and 0.18 mg epinephrine 3.6 mg lidocaine and 0.018 mg epinephrine 36 mg lidocaine and 0.18 mg epinephrine 36 mg lidocaine and 0.018 mg epinephrine

36 mg lidocaine and 0.018 mg epinephrine KeJ- to question: I ml ofa 2olo solution oflidocaine with l:100,000 epinephrine contains 20 mg of lidocaine and 0.010 mg of epinephrine. A dental carpule contains 1.8 ml solution. Therefore, 1.8 ml of2% solution oflidocaine with I : 100,000 epinephrine contains 36 mg of lidocaine and 0.018 mg epinephrine. Epinephrine (vaso constrictor) is included in local anesthetics for the following reasons: . . . . It prolongs the duration ofaction It reduces toxicity It reduces the rate ofvascular absorption lt provides a hemostatic effect to reduce bleeding at injection site

A teaspoon can hold what volume of liquid 5 milliliters l0 milliliters l5 milliliters 20 milliliters

5 milliliters

When phentolamine mesylate (oraverse) is given after a dental procedure to reverse the effects of local anesthetic about how fast is the return to normal sensation compared to a normal time for the anesthetic to wear off 10% decrease in the time for normal sensation to return 25% decrease in the time for normal sensation to return 50% decrease in the time for normal sensation to return 80% decrease in the time for normal sensation to return

50% decrease in the time for normal sensation to return Thus if a local anesthetic such as lidocaine with epinephrine normally takes about 180 minutes to wear off, after phentolamine injection, the anesthetic will wear offin approximately 90 minutes. This represents a 50o% decrease in time for normal sensation to be re- tored. Note: OraVerse dose is based upon the number of cartridges of local anesthetic with vasoconstrictor administered. . . . l/2 a caftridge (0.2mg) of OraYerse if ll2 cmidge of anaesthetic was administered I cartridge (0.1mg) of OraYerse if I carhidge of anesthetic was administered 2 cartridges (0.8mg) of OraVerse if2 cartridges ofanesthetic were administered Note: OraVerse (so lutionJbr injection/dental cartridge) is administered as a submucosal iniection,

The following drugs are used to treat which viral disease Didanosine Zidowdine Ritonavir Indinavir Delavirdine

AIDS AIDS has been recognized since l98l as a unique clinical syndrome brought on by in- fection with the human immunodeficiency virus | (HIr/-l) or virus 2 (HIV-2).The major cellular defect caused by infection with HIV is a depletion ofT cells, primarily the sub- qpe T-helper cells kaown as CD4 cells. This results in a compromised immune system, * hich becomes susceptible to opportunistic infections. HIV is a type ofretrovirus that is responsible for the fatal illness flom AIDS. A retrovirus has R\A as its nucleic acid and uses the enzyme reverse transcriptase to copy lts genome into the DNA olthe host's cells chtomosomes. This DNA segment is then permanently incorporated into the host cell's DNA within the nucleus. The integrated DNA segment can produce nerv RNA in the cytoplasm of the host cell. The new RNA in tum synthesizes viral proteins, which are eventually passed on to other host cells such as the immune system macrophages. Ultimately enough ofthe human immune cells are compromised such that immune function is lost. The HIV drugs such as didanosine @ rand name Wdex), zidovtldine (brand name Retro- vil), ritonrvir (brand name Norvry', and indinavir @rand name Crixivan) work by in- hibiting certain steps in the HIV inlection process within the target cells to halt the destruction of the immune system by the HIV retrovirus.

Alkylating agents are most effective in treating all of the following EXCEPT Chronic leukemias Lymphomas AIDS Myelomas Carcinomas of the breast and ovary

AIDS The alkJ-'lating agents contain a diverse group ofcompounds which all form alkyl bonds to nucleic acids. All ofthese agents share a similar mechanism of action and mechanism of resistance. The alkylating agents lorm covalent bonds with nucleic acids, and pro- reins. The N-7 position ofguanine is a common binding site.

Wwhich has little value in treating acute inflammation Ibuprofen Acetaminophen Aspirin Naproxen Nabumetone

Acetaminophen Acetaminophen inhibits central prostaglandin synthesis -it (asplrlr/ in blocking pe- ripheral prostaglandin synthesis, it has no anti-inflammatory activity and does not affect platelet function and therefore will not affect clotting time. Note: In large doses 8 hours), acelaminophen can cause hepatic necrosis. Concurr€nt use with the following drugs may increase the risk of hepatotoxicity: barbitu- mtes, hydantoins, carbamazepine, rifampin, sulfi npyrazone, and ethanol. Acute overdosage ofacetaminophen can result in hepatotoxicity and is life threatening. Acetominophen is metabolized to a highly toxic int€rmediate product, which normally is detoxi- hed by glutathione. When glutathione is depleted, the toxic intermediate attacks other cells, causing necrosis. Syrnptoms that appear in the ltrst 24 hours are nausea, vomiting, drowsiness, lethargy, malaise, and confusion. Note: N-acetylcysteine (NAC) is the specific antidote for acetaminophen poisoning. Acetaminophen is preferred over aspirin when an analgesic or antipyretic drug is indicated and also the patient: . . . . Is asthmatic Is at added risk for an ulcer ls experiencing bleeding Is taking anticoagulants . . Is allergic to aspirin p Z 5 g within Is taking drugs such as probenecid or methotrexate Remember: Aspirin and Nonsteroidal Anti-inflammatory Agens (Ibuprofen, Naproxen, \qbumetone, and others) inactiYate the enzyme known as cyclooxyg€nase. Since cyclooxy- genase s).nthesizes the prostaglandins, the inhibition ofthis enzyme results in the inhibition of prostaglandin synthesis. Cyclooxygenase has an acronym ofCOX' Thus the Nonsteroidal Anti-inflammatory Agerrts (NSAlDs) are also known as COX inhibitors. As a result, they ha\e analg€sic, antipyretic, and anti-inflammatory actions. NSAIDS must be used cau- riously in patients with peptic ulcer disease.

Which non-narcotic analgesic would you use in a patient anticoagulation medicine Ibuprofen (Motrin, Advil) Acetaminophen (Tylenol) Aspirin Naproxen sodium (Aleve)

Acetaminophen (Tylenol) Acetaminophen has no effect on platelets nor the coagulation pathways and does not affect bleeding times or prothrombin times even with high doses. Acetaminophen has two major pharmacological actions: an analgesic effect and an an- tipyretic (ever redticey' effect. Acetaminophen is not effective enough to reduce severe pain, but it is effective in reducing mild to moderate pain. Acetaminophen is a weak in- hibitor of prostaglandin format ion. Large doses ofacetarrinophen can cause liver toxicity. Alcohol can seriously increase the hepatotoxic potential of acetaminophen. There are approximately 100 deaths annually due to liver toxicity produced by ingesting large continuous doses of acetaminophen. Acetaninophen very rarely causes drug sensitivities and can be given to patients having an allersv to asnirin.

The autonomic nervous system has cholinergic nuerons that secrete Epinephrine Norepinephrine Acetylcholine Dopamine

Acetylcholine Chofinergic is a term for a nerve ending that releases acetylcholine Acetylcholine receptors are called cholinergic recepton. They are subdivided as follows: . . *** Muscarinic receptors fsites,): - At neuroeffector sites for all postganglionic cholinergic neurons (this is chqracteris- tic of a ll parasympathetic postgqnglionic nerves) - At neuroellector sites ofpostganglionic sympathetic n€rYes to the sweat glands and a few blood vessels (tl ese Nicotinic receptors (silesr: postganglionic nerves are also cholinergic) - At the skeletal neuomuscular - At ganglionic sites /*** junction (involving sonatic nerves) The same type of nicotinic receptor is present in sympathetic ganglia, parasympathetic gangliq, and the adrenal medulla) Drugs that resemble ACh in chemical structure and bind to these receptors imitate the ef- fects of parasympathetic postganglionic activity. Important: L Preganglionic neurons ofboth divisior's (parasympathetic a ergic. as are postganglionic neurons ofthe parasympathetic division. 2. Postganglionic neurons ofthe sympathetic division are usually adrenergic. \ote3 .. &tt:' (l sympathetic) are cholin- 1. The action of acetylcholine at postganglionic parasympathetic sites is often referred to as a muscarinic response. 2. The term nicotinic response is used to describe the stimulating action ofACh on the ganglia, as well as its actiol at the neuromuscular muscle. junction of skeletal 3. Muscarinic receptors are usually linked to Go, phospholipase C (PLC), andCa".

Which is a compulsive uncontrollable dependence on a substance habit or practice to such a degree that cessation causes severe emotional mental or physiological reactions Habituation Tolerance Addiction None of the above

Addiction Habituation is an acquired tolerance from repeated exposure to a particular stimulus. Psychological and emotional dependence on a drug, tobacco or alcohol result from the repeated use of the substance but without the addictive, physiological need to increase dosage. Tolerance is the phenomenon ofdecreased responsiveness to a drug following chronic administration. Note: Physiological dependence is common to all forms ofdrug dependence and abuse. These drugs ofabuse all have the ability lo change one's mood and sensory perception.

Whats the phrmacological catergory of the following drugs Epinephrine (Adrenalin) Phenylephrine (Neo-synephrine) Albuterol (Proventil;Ventolin) Isoproterenol

Adrenergic agonists Note: The adrenergic receptors are divided into the subtypes of: . . alphal (o.1) and alpha2 (a2) betal (B l) and beta2 (82). Some common adrenergic agonists, their main us€s and rec€ptor preference: . Epinephrine: reverse anaphylaxis, vasoconstriction, bronchodilation - alphal, alphal, betal, and beta2 . Nor€pinephrine: vasoconstriction . . . -alpha1, alpha2. and betal Isoprot€renol: bronchodilation - beta1, and beta2 Phenylephrine: nasal vasoconstriction - alphal. and alpha.r Albuterol: bronchodilation - beta.) . Terbutaline: bronchodilation . . . - betal Clonidine: antihlpertensive - alpha2 \Iethvldopa: antihypertensive - alpha.' Dobutamine: cardiac stimulation - alphat, and betal

Cephalosporins have which of the following modes of action Affect bacterial cell wall Affect bacterial DNA Affect bacterial protein synthesis Interfere with bacterial metabolic pathways

Affect bacterial cell wall Basic mechanisms of actions ofantibiotics: . Affect bacterial cell wall: - Penicillin family - Cephalosporin family . . . Agents affecting bacterial DNA: - Fluoroquinolone family (i. e., ciproJloxacin) - Meronidazole (F lagt l) Agents affecting bacterial protein synthesis: - Tetracycline family - Erythromycin family - Clindamycin - Chloramphenicol Agents interfering with bacterial metabolic pathways: - Sulfonamides (sulfa drugs)

Which is the most likely compliction seen with chemotherapy treatment Renal failure Alopecia Peripheral neuropathy Glaucoma

Alopecia Alopecia (hair Ioss) occurs with administration ofmost clremotherapeutic agents one to tlr o weeks after treatment. Other common side effects include GI upset, increased inci- dence ofinfection (especially Candidias,9, and degeneration of lymphatic tissue. Remember: Most chemotherapy drugs have been shown to be teratogenic in humans and should be avoided by pregnant women. \ot€: \Iethotrexate mav cause ulceration of the oral tissues.

The major natural mineralcorticoid in humans is Dexamethasone Aldosterone Cortisol Prednisone Triamcinolone

Aldosterone Cortisol, prednisone, dexamethasone, and triamcinolone are all glucocorticoids. Aldosterone is secreted by cells located in the zona glomerulosa ofthe adrenal cortex. The secretion ofaldosterone is regulated by ACTH and by the renin-angiotensin system and is very important in the regulation of blood volume and pressure. Aldosterone pro- motes rerbsorption ofsodium into the blood from the glomerular filtrate. Potassium is lost in the urine because of the electronegativity that is created by the reabsorption of sodium in the kidney tubules. Note: Increased blood aldosterone levels will result in high sodium and low potassium levels in the plasma. Remember: Decreased sodium concentration causes the juxtaglomerular kidneys to secrete renin, which converts angiotensinogen to angiotensin I. Angiotensin I is converted to angiotensin II, which, in tum, stimulates the adrenal cortex to release al- dosterone. , -,- 1. Addison's disease is caused by the hyposecretion of aldosterone and corti-rNote{,] sol. *if' ,. 2. ADH (Vasopressin) decreases the production of urine by increasing the re- absorption of water by the renal tubules (it increases the permeobiliry of the collecting ducts and distal tubules). Without ADH, there would be extreme Ioss ofwater into the urine. 3. At high concentrations, ADH causes arterioles to constrict (increases blood Dressure).

Factors influencing hepatic drug metabolism include which of the following? Microsomal enzyme inhibition Microsomal enzyme induction Plasma protein binding Genetic factors Liver disease All of the above

All of the above Factors Influencing Hepatic Drug Metabolism: . Microsomal enyzne inhibition: many drugs and environmental agents can inhibit many of the CYP isoforms of the P-450 microsomal drug metabolizing system. Thus many drugs which ordinarily are metabolized by the particular CYP inhibited will not be effectively metabolized and will achieve higher than expected blood levels. . Microsomal enzyme induction: agents which induce higher levels ofthe microsomal drug metabolizing enzymes may cause a more rapid metabolism ofother drugs thus resulting in lower than expected blood levels ofa drug. . Plasma protein binding: drugs highly bound to plasma proteins will not enter the liver to be metabolized, thus resulting in a longer plasma halfJife ofthe drug. . Genetic factors: there is individual variance through genetic factors which contribute to differing rates of drug metabolism in the hepatic microsomal enzyme system. . Liver disease: hepatic impairment and liver disease most often will result in impair- ment ofthe microsomal drug metabolizing system. This most often results in elevated levels of unmetabolized drug. Note: Mos! drugs are given at lower doses in hepati cally impaired individuals.

Alpha adrenergic receptors are located on Vascular smooth muscle Presynaptic nerve terminals Blood platelets Fat cells Neurons in the CNS All of the above

All of the above There are two main types ofadrenergic receptors: I . Alpha (o) receptors responsible for most of the excitatory effects such as vaso- constriction and contraction ofthe uterus and spleen. l. Beta (R) receptors responsible for most ofthe inhibitory effects such as vasodilation and relaxation ofrespiratory smooth muscle. - \ote: Two fnportant EXCEPIIONS to the above: some alpha receptors mediate relax- atio of gastrointestinal smooth muscle, and some beta receptors mediate increases in the torce and rate of contractions ofthe heart. -\lpha receptors fall into two groups: 1. Postjunctional alpha, adrenergic receptors are found in radial smooth muscle of the iris. arteries, arterioles, and veins; in the GI tract. 2. Prejunctional alpha, adrenergic receptors mediate the inhibition of the release of norepinephrine. Bela receptors fall inlo t\\o groups: 1. Postjunctional betar adrenergic receptors are found in the myocardium (heart), the intestinal tract smooth muscle, and adipose tissue. 2. Postjunctional beta, adrenergic receptors are found in bronchiolar and vascular smooth muscle.

The Controlled Substsnce Act of 1970 uses which criteria for inclusion of a drug into one of the five schedules? Degree to which it produces physiological dependence Degree to which it produces physical dependence Potential for abuse Medical usefulness All of the above

All of the above Tle Controlled Substancc Acl of 1970 uses the following critcria fbr inclusion of a drug into onc ofthc fivc schcd- 1 . PorcntiaL for abuse r'', oi inportan, :. \tcdicaluscfulness L Degfcc to which it .1. produces physiological dcpcndcncc Dclrcc to *hich it produccs physical dcpcndcncc Remember: Schedule I d gs (LSD, hercin, etc.)@nnot be prescribed and a.c made availablc only for specific ap- prored research pro.jccts. Schedule II drxgs fdmphetanines, noryhirc, cotleine, oxycodone elc.)$n be prescribed but can not be refilled. A ne\\ prcscription mustbcwritten forrefills. Prescriptions lor Schedule II drugs can not bc called into the pharmacy ovcr thc telephone. Schedule III drugs (,,,)d/ocodotie *'ith acetaninophe , cotleine \\'ith arctaninophen, etc.) may be called in to thc out need ofa new writlcn prcscriplion. *** pharmacy over the telephonc. Thc prcscribcrcan authorizc rcfills with- Thc prcscribcr must hsve a Drug Enforcement Agency ruthorization nnmber (DEA #) in ordcr lo prcscribc schcdulcd drugs. Clinical testing ofdrugsr Bcfore a drug can be approvcd for salc lo thc public there is a set ofclinical tests that must be pcrformcd. Thcrc is thc Pre-Clinical Research Stage. Here the drug is s),nthcsizcd and Dcrformed. and institutional rcvicw boards asscss thc studies and make recommcndations on how to procccd. rccommcndations are positivc, thcn an application to the FDA occurs and clinical tesls bcgtn. 'Phase purified. Animal tcsts arc t: clinical studics in this phase represent thc first time that an investigational ncw drug is tested on hu- ofthcsc studics is study in a clin;cal sctting thc metabolism, structurc-reactivity rclationships. mcchanism ofaction, and side ellccts ofthe drug in humans. lfpos- mans cithcrhealthy voluntccrs or somctimcs patierts. The siblc, purposc phasc 1 studies arc uscd to determinc how ellcctivc thc drug is. Phasc I studics are usually conducted on 20 to E0 subiccts. . The purpose of phrse 2 clinical trials is to dctcrminc thc cfficacy ofa drug to trcat patients with a spccific dis- casc or condition. as well as lcam about common short{crm sidc cffccts or risks. These studics are conducted on a la.gcr scale than phasc I sodics and tlpically involvc scvcml hundrcd . Phase 3 cLinical trials patients. provide morc information about the effects a d safcty ofthe drug and they allow scicn- gcrcrally involvc scvcral hundrcd to scvcral thousand tists to cxtrapolate thc rcsults ofclinical studies to thc gcncral population. Phasc 3 studics *** After a succcssful pcoplc. phasc III, thc drug company submiis to the Food and Drug Administration 1FD,r/ a Ne\r' Drug Application fND,4/ to markct thc dru8

Verapamil is useful for the treatment of Angina Hypertension Supraventricular tachyarrhythmias All of the above

All of the above \-erapamif is the prototypical Class lY (celciu channel blockers) antiarrhythmic agent. It hibits the intraccllular entry ofcalciurn through the slow channels ofthe calcium dependent tissues of the myocardium, rvhich are concentrated in the SA and AV nodcs. It is the drug ofchoice for the suppression ofparoxysmal supraventricular tachycardias stemming from the AV node (wlrlclr is characteri:ed by a rcpid cer.liac rate, usually 160-190 per minule). Of the calcium channel antagonists available, only vcrapamil and diltiazem possess significant antianhythmic activity. Note: Calcium channel blockers (e.g., verapanril, dillazem and nifedipile) are useful as antianginal agents and antihypertensive agents as rvcll. They block calcium cntry through the mem- branous calcium ion channels ofcardiac and vascular smooth musclc. This has thrcc cffccts: (vasodilato,", and total periphcral rcsistance deoreascs, reducing after-load and reducing myocardial oxygen requirements. 1. Peripheral arteriolcs dilate 2. lncreases oxygen delivery to the myocardium. 3. Reduces blood pressurc ifhypertension is present. lmportant; Nilrites (amyl nilrite) and nitratcs (nittoglyceri\) are fast-acting antianginal agents that are uscd to relieve acute anginal attacks. \ote: For angina, nitroglycerin and nifedipinc arc usually used before verapamil.

"Epinephrine reversal' is a predictable result of the use of epinephrine in a patient who has received a/an: Beta-blocker Alpha-blocker Adrenergic agonist All ofthe above

Alpha-blocker Explanation of answer: One of the best known effects of the alpha-receptor trlocking ^gents (antiadrenergiy' is their ability to reverse the pressor action of adrenaline fepl- nephrine).In the absence ofblocking agents, epinephrine and norepinephrine both cause the blood pressure to rise. After the alpha-receptors have been blocked by an alphablocker, the pressor effect of norepinephrine is reduced or abolished while epinephrine brings about a fall in blood pressure. This is because epinephrine stimulates both alpha and beta-receptors in the cardiovas- cular system but norepinephrine only stimulates alpha-receptors (norepinephrine lacks beta2effbcts). After blockage ofthe alpha-receptors, only the beta-receptors can be stim- ulated. Note: A pressor response (produces an increase in blood pressure) is mediated by alphareceptors and a depressor response (produces a decrease in blood pressure) is mediated bv beta. receDtors.

The most widely used tricyclic antidepressant drug (TCA) is Imipramine (Tofranil) Amitriptyline (EIavin) Desipramine (Norpramin) Nortriptyline (PameIor) Doxepin (Sinequan)

Amitriptyline (EIavin) The tricyclic antidepressants are generally considered to be the drugs of first choice for the treatment ofunipolar disorder (depresslor). These drugs inhitlit the neuronal re-up- take of norepinephrine and s€rotonin (5-HT) in the brain. This results in the potentiation oftheir neurotransmitter actions at postsynaptic receptors. Important: Amitriptyline displays the greatest anticholinergics effects (especially xe- rostomia), desipramine the least. Drowsiness is the most frequent CNS adverse reaction. Anticholinergic adverse side ef- fects include: dry mouth, constipation, blurred vision and tachycardia. The selective serotonin reuptake inhibitors (S,SR1s) have revolutionized the treatment ofdepression. The most important clinical distinction ofthe SSRIs from all other antide- pressants is their very high specificity for blocking the reuptake of serotonin. Fluoxetine (Prozac:)is (Zololi) the prototlpe and has the longest half-life. Paroxetine (Paxil), Sertraline and Fluvoxamine (Luvox) have shorter half-lives and can be given once daily. Citalopram (Celexa) and escilalopram (Lexapro) are used for generalized anxiety disor- ders. These drugs are also effective for treating panic attacks. Side effects include nau- sea. headaches, anxiety, agitation, insomnia, and sexual dysfunction.

The antibiotic of choice for standard prophylactic regimen of antibiotic coverage for the prevention of bacterial endocarditis is Penicillin VK Tetracycline Erythromycin Amoxicillin

Amoxicillin Premedication requirements for patients with valvular heart disease or congenital cardiac defects. If in doubt, have patient consult their physician as to need. CurrentAmerican Heart Association Guidelines: Published June 2007, JADA Volume 138, page 142. Standard Regimen . Amoxicillin - Adults: 2.0 g orally 30-60 minutes prior to appointment - Children: 50 mglkg Allergy to Amoxicillin: (not to exceed adult dose) orally 30-60 minutes prior to appointment Note: Only ifallergy is not ofanaphylactic type. Any one ofthe following can be used. . Clindamycin -Adults: 600 mg orally 30-60 minutes prior to appointment -Children: 20 mg,&g orally 30-60 minutes prior to appointment 'Cephalexin -.\dult.: 2.0 g orally 30-b0 minutes prior to appointment -Children: 50 mg/kg orally 30-60 minutes pdor to appointment ..{zithromycin . -Adults: 500 mg orally 30-60 minutes prior to appointment -Children: l5 mg/kg orally 30-60 minutes pdor to appointment Clarithromycin -Adults: 500 mg orally 30-60 minutes prior to apporntment -Children: l5 mg/kg 30-60 minutes pdor to appointment

Amphetamines are sympathomimetic amines that cause rapid release of: Dopamine in the brain Serotonin in the brain Norepinephrine in the brain Acetylcholine in the brain

Amphetamines pass readily into the CNS and cause a rapid release of norepinephrine in the brain. Amphetamines increase systolic and diastolic blood pressures and act as weak bronchodilators and respiratory stimulants. They have a high potential for abuse, result- ing in tolerance, psychological dependence, and severe social disability. Abuse causes ex- treme violence and paranoid psychotic behavior Note: Amphetamines and ephedrine are indirectly acting sympathomimetic (sympathetic-type) drugs. These drugs demonstrate tolerance and are orally active, unlike epinephrine and norepinephrine. Therapeutic uses of amphetamines: . Attention Deficit Hl?eractivity Disorder (ADHD; hyperkinesis) dextroamphetamine (Dexedrine) and a mix of dextroamphetamine with amphetamine (Adderall); (Adderall XR) sometimes used instead ofmethylphenidate (Ritalin). . Narcolepsy in these patients. . Weight loss - - dextroamphetamine (Dexedrine) is used to prevent daltime sleepiness phentermine (Ionamin) - structurally similar to dextroamphetamine. 1. Tyrosine hydroxylase catalyzes the rate limiting step in the synthesis of norepinephrine (tr'E) and epinephrine. The enzyme is inhibited by metyro- sine. 2. Terrnination oftransrnission by NE takes place primarily by the reuptake ofNE into prejunctional nerves and secondarily into other cells. Monoamine oxidase (MAO) and catechol-O-methyl transferase (COMT) then play a role in metabolizing the NE.

Which of the following is classified as an antifungal agent Bacitracin Amphotericin-B Polymyxin-B Neomycin

Amphotericin-B This is an antifungal agent given intravenously or orally for the treatment ofsevere sys- temic fungal infections caused by fungi such as Candida species. Bacitracin, polymyxin- B and neomycin are not antifungal agents. These are antibiotics effective against sus- ceptible bacteria. Antibiotics in general do not have antifungal properties.

Which two penicillins are referred to s aminopenicillins Penicillin VK Ampicillin (Polycillin) Cloxacillin (Cloxapen) Amoxicillin (Amoxil)

Ampicillin (Polycillin) Amoxicillin (Amoxil) These trro antibiotics are characterized by the amino substitution of penicillin G. They \\ ork against many gram-positive organisms and some gram-negative bacteria such as Haemophilus influenzae, some Escherichia coli, and Proteus mirabilis. They are not penicillinase resistant. They are used for upper respiratory infections. The major difference in the drugs is the higher oral absorption, higher serum levels, and longer half-life for amoxicillin compared with ampicillin. Amoxicillin is given orally; ampicillin can be given orally and IV Remember: Oral amoxicillin is recommended as the drug ofchoice for standard general prophylaxis for bacterial endocarditis in patients undergoing invasive dental procedures. Note: Parenteral ampicillin is recommended as the drug of choice in patients unable to take oral medications and who are not allergic to penicillin for prophylaxis for bacter- ial

When two drugs have similar effects are administered in combination and the resulting response is the sum of the individual actions of each drug when given alone then this result is reffered to as An additive effect A synergistic response An antagonistic effect Cumulative action

An additive effect An additive effect occurs when additive drugs are administered. The response is no greater than the sum ofthe individual actions ofeach drug when given alone. There is no enhancement ofpotential ofthe individual drugs as a result ofbeing used in combination. A synergistic response occurs when the combined action of two drugs having similar pharmacological effects is greater than th€ sum of the individual actions. Alcohol is s).nergistic with the Valium family ofdrugs fi.e., Valium, Xanax, Halcion, etc), narcotics and barbiturates. Alcohol should be avoided when takins these medications.

The following effects result from what type of pharmacological agent -Low BP - Vasodilation -Orthostatic hypotension An alpha-adrenergic receptor blocker (alpha-blocker) A beta-adrenergic receptor blocker

An alpha-adrenergic receptor blocker (alpha-blocker) Examples ofselective alphal-blockers used in medicine: . Doxazosin (Cardura) used to fieat hypertension . Prazosin (Minipress) used to treat hypertension . Terazosin (Hytrin) manage mild/moderate hypertension; treatment ofbenign pro- static hyperylasia . *** :iolcra -Ji... frPd) Tamsulosin (Flomax) used to treat benign prostatic hyperplasia (BPH) These drugs selectively block tr1-type receptors but not o2-type Important: Alphay-blockerc cause orthostatic hypotension, also known as postural hy- potension. This is a fainting spell which occurs because ofa rapid fall in blood pressure when moving from the supine to the upright position, as in getting rapidly out ofthe dental chair. The symptoms are similar to simple fainting, however the condition is related to positioning. Note: Other adverse effects include tachycardia, nasal congestion and dry mouth. *** Orthostatic hypotention can also result from centrally acting drugs (i.e., clonidine and methyldopa) and direct vasodilators (i.e., hydralazine and diazoxide) 1. Alpha-blockers inhibit the vasoconstrictor response to epinephrine and levonordefrin. 2. Beta-blockers increase the vasoconstrictor response to epinephrine, but re- duce the tachycardia resulting from epinephrine. 3. MAO inhibitors (i.e., phenelzine and tranylcypromtue) should not be used with indirectly acting sympathetic drugs (i.e., Tyramine and anphetamines) and with several other drugs such as opioids, especially meperidine. 4. Epinephrine and levonordefrin have exaggerated effects when given with neuron depleting agents like reserpine and quanethidine.

All are classified as glycoprotein 2b/3a inhibitor type of antiplatelet agents EXCEPT Abciximab (Reopro) Anagrelide (Agrylin) Epttfibatide (Integrilin) Tirofiban (Aggrastat)

Anagrelide (Agrylin) Glrcoprotein IIb,4Ila inhibitors are reversible antiplatelet agents uscd !o prcvcnt acut€ cardiac ischemiccomplicfiions and used in paticnts with acute coronarv svndrome. Thev are adntinistered imtravcnously. Tleie a{cnts block the plafelet glycoprotein IIb/IIIa receptor, rhe binding site for fibrinogen, von Willebrand r'3.tor. and othcr ligands. Inhibition of binding at this final common receptor reversibly blocks phtel€t ag- gregation and prevenls thrombosis- Platclet aggregation inhibition is revcrsible following ccssation ofthc lV 3cmr:inration ollhc drug minutes after an intruvenous infusion. Note: The glycoprotein IIb/lIIa inhibitors h3! i a raPid onset ofaction, Their maximal antiplatelet effect occur within minutes after an intravenous in- 1-li[]r Important: Thc most serious adverse effects of GPIIb/llla anragonists include major bleeding, in- '- .. . ::h:.rl hcn un hrce u.lJ rhromboc\ ron(nir

A patient was given penicillin 15 minutes ago and develops the following signs and symptoms. What reaction was the patient having Laryngeal edema Urticaria (welts that itch) Severe hypotension GI disturbances Bronchoconstrictron (airway constriction) Shock

Anaphylactic reaction aka anaphylactic shock This reaction most conmonly occurs with parenteral administration ofpenicillin. This reaction can be fatal if countermeasures such as the injection ofepinephrine are not taken promptly. Epinephrine prevents the release of substances from mast cells and an- lagonizes the actions of histamine and leukotrienes of smooth muscle. The most common adverse effect ofpenicillin therapy is an allergic reaction. These reactions occur in up to 109/o of patients receiving penicillin. The most common manifes- ration is a mild rash. Allergic reactions to penicillin are classified into three groups: l. fmmediate onset reactions (araphylaxis) ocatr within 30 minutes; Ig-E mediated. 2. Accelerated allergic reactions arise 30 minutes to 48 hours after administration. Urticaria, pruritis, wheezing, and local inflammatory reactions. In general, not life- threatening. Delyed- allergic reactions take longer than two days to develop. 80-90% ofpenicillin reactions are ofthis type. Basic skin rashes, which are generally mild.

Opioid drugs (narcotics) are used as all the following EXCEPT Analgesics Anti-inflammatories Antitussives Antidiarrheals Preanesthetic medications

Anti-inflammatories Opiates are very effective analgesics, They suppress the cotgh reflex (tntitussive). They cause constipation and thus are effective antidiarrheal agents. When used as preanes- thetic medications, opiates permit a reduction in the amount of general anesthetic re- quired for surgical anesthesia. Some common side effects ofopiates (narcotics) include: sedation/drowsiness, dizziness and nausea. Other less common adverse effects are vomiting, h)?otension, irregular/la- bored breathing /dyspnea), , : lightheadedness, nightmares and insomnia. - - ... l. Respiratory d€pression is dose related and is the cause of death in nar- i\oq: '.td;a cotic drug overdose. It can happen with any of the narcotics. 2. Abuse can and does occur with all the narcotics. They are all controlled drugs falling under DEA Schedule II or DEA Schedule III ofthe schedules of controlled substances. 3. A DEA number is required in order to prescribe narcotics.

Ethyl alchohol (ethanol) causes a marked diuresis by inhibiting the production of which hormone Growth Hormone Insulin Antidiuretic Hormone (ADH) Epinephrine

Antidiuretic Hormone (ADH) (aka vasopressin) ADH is a hormone that decreases the production of urine by increasing the reabsorp- tion of water by the renal tubules. Without ADH, there would be extreme loss of water into the urine. Ethanol (ethyl alcoftoi) inhibits the production ofADH. Remember: Ethyl alcohol dilates blood vessels of the skin, depresses the CNS, and in blood levels in excess of400 mg o/o usually results in coma and death. Alcohol acts pri- marily on the nerve cells within the brain. Alcohol interferes with communication be- tween nerve cells and all other cells, suppressing the activities of excitatory nerve pathways and increasing the activities of inhibitory nerve pathways. Note: Recent evidence has shown that ftequent ingestion of moderate amounts ofalco- hol in any form (beer, wine, distilled spiits) will reduce the risk of heart disease, par- ticularly in men. The order in which alcohol affects the various brain centers is as follows: . Cerebral cortex . Limbic system . . . Cerebellum Hypothalamus and pituitary gland Medulla lbrain stem)

Of the amide-type local anesthetics, which is the only one that is metabolized in the bloodstream rather than the liver? . Articaine (Septocaine, Zorcaine) Mepivacaine (Carbocaine) Lidocaine (Xylocaine) Prilocaine (Citanest) Bupivacaine (Marcaine)

Articaine (Septocaine, Zorcaine) Articaine (Septocaine, Zorcaine) isan amide-type local anesthetic. However, it is chem- icalll unique in that it has an €ster group attached to its molecule which can be acted upon by plasma cholinesterase to render it ineffective. Therefore, it is the only amide q hich is metabolized in the bloodstream and not the liver. Rapid metabolism ofthis ester bond gives it a short halflife. .{nicaine is supplied as articaine HCL 4% solution with epinephrine l:100,000 ard as articaine HCL 4olo solution with epinephrine l:200,000. It is indicated for local, infiltrative, or conductive anesthesia in both simple and complex dental and periodontal procedures. The onset ofanesthesia following administration ofarticaine has been shown to be I to 6 minutes after injection. Complete anesthesia lasts approximately I hour. Articaine is contraindicated in patients with hypersensitivity to local anesthetics ofthe amide type or to sodium bisulfite. (Carbocaine

Whats the class of the following drugs Etanercept (Enbrel) Infliximab (Remicade) Adalimumab (Humira)

Antirheumatic agents There are currently three tumor necrosis factor alpha (TNF) inhibitors FDA approved for the treatment ofRA (listed in order oftheir approval k4): . Jbr Etanercept (Enbrel) is used to reduce the signs and symptoms ofactive rheumatoid arthritis in patients who have had inadequate response to one or rnore disease-modifr- ing anti-rheumatic drtgs (DMARD).It is a recombinant DNA-derived protein which binds to tumor necrosis factor alpha (l'ly'F). . Infliximab (Remicade) is used to treat Crohn's disease and rheumatoid arthritis. Infliximab is a chimeric monoclonal antibody that binds to tumor necrosis factor alpha (TNF) thereby reducing the inflammatory actions ofthis endogenous compound. . Adalimumab (Humira) is a fully human anti-TNF monoclonal antibody with high specificity for TNF. Adalimumab binds specifically to TNF and blocks its interaction with the p55 and p75 cell surface TNF receptors, thereby interfering with endogenous TNF activity. Adalimumab binds to both soluble as well as cell bor.urd TNF. It is ad- ministered by subcutaneous injection every two weeks but can be increased to weekly, ifneeded. Adalimumab is effective in RA, Psoriatic arthritis, and ankylosing spondyli- tis, and Crohn's disease. \ote: Tumor necrosis iaclor alpha (TNF) is a pro-inflammatory cytokine produced by macrophages and lymphocfes. It is lound in large quantities in the rheumatoid is produced locally in the joint joint by synovial macrophages and lymphocyes infiltrating the synovium. TNF is one ofthe critical cytokines that mediate struction due to its activities on many cells in the joint joint as well as effects on other organs and bodv svstems.

Exemestane (aromasin) and letrozole (femara) are anticancer drugs classified under which category Antibiotics Aromatase inhibitors Antimetabolites Alkylating agents Antimicrotubular

Aromatase inhibitors Exemestane is an irreversible, steroidal aromatase inactivator. It prevents conversion of androgens to estrogens by tying up the enzyme aromatase. In breast cancers where growth is estrogen dependent, this drug will lower circulating estrogens. Exemestane is used in the treatment ol advanced cancer in post menopausal women whose disease has progressed following tamoxifen therapy. Letrozole (Femara) worksby a similar mechanism as above. It is used as first line treatment ofhormone receptor positive or metastatic breast cancer in postmenopausal women. It is also indicated as an extended adjuvant treatment of early breast cancer in post- menopausal women who have received 5 years ofadjuvant tamoxifen therapy.

Which two may interact with warfarin to cause increased bleeding Hydrocodone with acetaminophen Aspirin Acetaminophen Ibuprofen (Motrin; Advil)

Aspirin Ibuprofen (Motrin; Advil) Ibuprofen and other non-selective NSAIDs (inhibitors of both cyclo-oxygenase I and q clo-orygenase 2 enzymes) snch as naproxen (Anaprox) and flurbiprofen (Ansaid) in- hibit platelet aggregation. This action would enhance the anti-coagulant effect of war- fain (Coumodin) to increase the risk ofbleeding. Aspirin inhibits platelet aggregation to potentiate the anticoagulant effects of warfarin (Coumadin) and increase the risk ofbleed- lng. Acetaminophen is a non-narcotic analgesic that do€s not affect platelet aggregation nor does it affect the coagulation pathway. Thus, it will not affect the anticoagulant nature of warfartn (Coumadin). Acetaminophen can be given safely to patients taking warfarin lCoumadi . Hyrocodone with acetaminophen is a combination ofa narcotic analgesic with aceta- minophen. Common brand names for this combination are Vicodin, Lorcet and Lortab. Hydrocodone and the entire drug class ofnarcotic analgesics do not affect blood clotting and will not enlance the anticoagulant effects of warfarin (Coumadin). Narcotics with acetaminophen can be given safely to patients taking warfarin (Coumadin).

All of the following are used to treat psychosis EXCEPT Chlorpromazine Thioridazine (MeIIariI) Risperidone (Risperdol) Azelastine (Astelin) Quetiapin) (Seraquel) Clozapine (Clozaril) Olanzapine (Zyprexa) Haloperidol (Haldol)

Azelastine (Astelin) Azelastine (Astelin) is an antihistamine nasal spray. Antipsychotics are primarily indicated for treatment ofpsychosis associated with schiz- ophrenia, paranoia, and manic symptoms of manic-depressive illness Typical antipsychotic drugs (lirst generation) are more potent antagonists of D2 dopamine receptors than ofDl receptors. First generation drugs include the phenoth- ia,zines (chlorpromazine, fluphenazine, mesoridazine, and thioridazine) , perphenazine, prochlorperazine, trifluoperazine, the thioxanthines (/riothixene) and the butyrophenones (haloperidol). Note: Haloperidol is a potent dopamine antagonist. Newer (secontl generation) or atypic l antipsychotic agents affect different receptor siles compared with first generation antipsychotics. They bind dopamine, including D1, D2, Da, and D5 receptors, with selectively for limbic dopamine receptors. They have increased at'linity for serotonin (5-HT ) receptors compared with D2 receptors. They exhibit reduced ability or an inability to induce EPS. Second generation agents include clozapine, risperi- done. olanzapine, quetiapine, ziprasidone, and aripiprazole.

All of the following drugs on the right are used to treat what condition? Dexmethylphenidxte (Focalin) Extended-release methylphenid|te (Concefta) Mixed amphetamine salts (Adderal) Atemoxetine (Strattera) Controlled-delivery methylphenidate (Metadate CR) Lisdexamfetamine (Vyanse) Insomnia in adults Tourette's sy,ndrome in children Attention deficit hyperactivity disorder (ADHD) lnsomnia in children

Attention deficit hyperactivity disorder (ADHD) Drugs used to treat ADHD: . Methylphenidate (Ritalin)z arnrld central nervous system stimulant. In children with ADHD, this drug results in an increase in attention span, reduction in hyperativity, and an improvement in behavior. . Dexmethylphenidate (Foc in): a form of methylphenidate called dexmethylphen- idate. . Adderal (dextroamphetamine): the brand name for mixed amphetamine salts which act the same as methylphenidate in treating ADHD. . Atemoxetine (Struttera)t the brand name for atemoxetine, the first non-stimulant approved for treating ADHD. It is approved for use in children and adults. . Metadate CR: the brand name for a controlled-delivery methylphenidate, another long acting form ofthe drug. . Lisdexamfetamine (Vyvanse): Lisdexamfetamine is a prodrug; after adminishation it is converted to dextroamphetamine (Adderall) in the rntestines and/or liver. The dextroamphetamine, an amphetamine and stimulant for the brain, is responsible for the ef- fect of lisdexarnfetamine in ADHD. . Extended-release methylphenidate (Concerlq): a long-acting form of methylphen- idate.

Erlthromycin is a ? antibiotic which binds to the ? ribosomal subunit of susceptible bacteria, The result is the inhibition of protein synthesis. Bactericidal; 30s Bactericidal; 50s Bacteriostatic; 30s Bacteriostatic; 50s

Bacteriostatic; 50s Erythromycin-type antibiotics are members of the Macrolide family of antibiotics. Members of the Macrofide family . Azithromycin (Zithromax; Z-Pak) . Cl ar ithr omy cin ( B i ax in ) . Erythomycin base (E-mycin; Eryc) . Erythomycin ethylsuccinate . Erythromycin stearate /generic lE (Etythrocin) E S./ and brand names) are. All of the €rythromycins are very effective against gram-positive bacteria but not so effeclive against gram-negative bacteria. AII ofthe erythromycins act through the same mechanism to bind to the 50s ribosomal subunit ofsusceptible bacteria. This causes the RNA to dissociate from the ribosome and prevents protein synthesis. \ote: GI Tract upset is the most common side effect of the erythromycins food.1.

All of tle following statements concerning barbiturates are true EXCEPT Barbiturates may increase the half-lives of drugs metabolized by the liver Barbiturates depress neuronal activily by increasing membrane ion conductance (primarily chloride), reducing glutamate-induced depolarizations and potentiating the inhibitory effects of GABA Compared with benzodiazepines, the barbiturates exhibit a steeper dose-response relationship Barbiturates may precipitate acute porphyria in susceptible patients

Barbiturates may increase the half-lives of drugs metabolized by the liver This is falsel barbiturates may decrease the halflives of drugs metabolized by the liver. Barbiturates induce the formation ofthe liver microsomal enz),nes that metabolize drugs. This leads to an increased clearance ofthe affected drugs and possibly leads to a decrease in the drugs effectiveness. The uses ofthe barbiturates are determined by their duration ofaction: 1. The ultrashort-acting agents are used intravenously for the induction of general anesthesia. For extensive procedures, they are used to induce stage III surgical anes- thesia. For very brief procedures, they may be used alone. 2. The short-acting agents can be used orally for their hypnotic, calming effect. These agents can be given preoperatively, before a dental appointment, to allay anxiety. 3. The intermediate-acting agents can also be prescribed to relieve anxiety before a dental appointment, although their effects will last longer than those of the shoft-acting agents. These agents are used for daytime sedation and for the treatment of insom- nia (they ,1. suppress REM sleep). The long-acting barbiturates are used primarily for daytime sedation and the treatment ofepilepsy. Generally the primary pharmacological effects ofthe barbiturates involve the brain and spinal cord (CNS - depression ofl. At sedative doses barbiturates do not effect or have linle effect on the cardiovascular and respiratory systems. Barbiturates are metabolized in the liver. The chronic use ofthe barbiturates causes an increase in liver microsomal en- z)me activity that appears to be the result ofincreased synthesis ofenzyme. These drugs possess serious drug dependence potential. They do not possess significant analgesic DroDerties.

The heart is generally considered to have predominantly which type of adrenergic receptors? a. Alpha 1 receptors b. Beta 1 receptors c. Alpha 2 receptors d. Beta 2 receptors

Beta 1 receptors

Which component of lidocaine local anesthetic Water Bisulfites Lidocaine Epinephrine (vasoconstrictor)

Bisulfites Patients may exhibit hypersensitivity to sulfites contained in some anesthetic solutions. Sodium metabisullite prevents the oxidation (deterioration) ofthe epinephrine vaso- constrictor in those commercial preparations containing epinephrine. Most ofthe patients reacting to bisulfites have a history of asthma and the airway is hyperactive to the sulfites. Allergic reaction usually results in an asthmatic slardrome ofwheezing and bronchial con- striction. Bisulfites are present in only those commercial preparations containing vasoconstrictor. Preparations without vasoconstrictor such as mepivacaine 3olo 37o) do not contain bisulfites. Important: All local anesthetics except cocaine are vasodilators, however, mepivacaine has Iess of a vasodilator effect compared to the others and, therefore, is the drug chosen \\'hen a vasoconstrictor is not used with the local anesthetic. \ote: Hypersensitivity or allergic reactions to local anesthetics, particularly the amides, are much more rare than allergic reactions to the bisulfites. .

All of the following are common adverse effects caused by the use of tetracyclines EXCEPT Photosensitivity Nausea Bone marrow disturbances Diarrhea Discoloration of teeth and enamel hypoplasia in young children

Bone marrow disturbances Another common adverse effect is the occurrence ofopportunistic (superintbction) in- fections caused by Candida albicans, This is due to the wide spectrum of antibacterial action which alters normal flora. Examples include both vaginal and oral candidiasis. These conditions are also known as vaginal and oral yeast infections. Members of the tetracycline family ofantibiotics include: Short acting: . Intermediateacting: Long acting: Tetracycline (Generic) .Demeclocycline (Declomycin) . Minocycline (Minocin) . Doxycycline (4bramycin) Remember: The usefulness of the tetracyclines in the treatment of odontogenic infections is limited since they can cause "yeast'' infections very easily. They have been used as alternatives to penicillin in patients with ANUG (acute necrotizing ulcerative gingil lll.r/ u'ho require antibiotics. \ote: In adults, Fanconi syndrome can be caused by various things that damage the kid- ne1s. including certain medications (azathioprine, cidolbvir gentamicin, and tetrucy- c/ire). Fanconi syndrome is a disorder ofthe kidney tubules in which certain substances normally absorbed into the bloodstream by the kidnevs are released into the urine instead.

Adrenline (epinephrine) stimulates Alpha1 -receptors only Beta1-receptors only Both alpha 1,2 and beta 1,2-receptors Both alpha1 and beta1-receptors only

Both alpha 1,2 and beta 1,2-receptors Remember: Alpha reccptor responses are predominantly ercitatory in nature, while beta receptor ie- iponses are €\citatory in nature in the heart and inhibitory elsewhere.

Antihypertensive agents lower blood pressure by reducing total peripheral resistance and by reducing cardiac output through a variety of mechanisms ACE inhibitors inhibit the conversion of inactive angiotensin 1 to angiotensin 2, a vasoconstrictor The first statement is true; the second statement is false The first statement is false; the second statement is true Both statements are true Both statements are false

Both statements are true Antihypertensiv€ agents arc classified by mechrnisms otaction: . Diuretics: irclude thiazide and thiazideJike fNd CIJ inhibitors.) such as hydrochlorothiazide foop diuretics such as furosemide /Zasi.r/ and brmet aide (Bumer). These drugs inhibit sodium reabsorption in renal tubular cells within the kidney !o cause excess sodium and u nary excretion resulting in reduced blood volume. . Beta-adren€rgic receptor blockers (6era-bloct?/sl: rcduce the volume ofcardiac output irlto the circu- lation resulting in reduce peripheral pressure. . l. Cardiosefective beta-blockefs (bela (Tenormin). metopfolol (Lopressor, Toprol XL), 2. Non-cardioselective (beta I Two types: rreceptor (HCTZ) and block in heart muscle/: examples includ€ at€nolol ^ceb\tolol (Secral), esmolol (Brevibloc) and beta, beta-blockers: examples include nadolol propr^nolol lndera0, tirnolol (Betinol), pellbntolol (Levatol), and sot^lol *** Note: labetalol alphar receptors. (BetapaL") Ard ndate) and c^rledilol (Col€g) are non-selective beta-blockers that also block Alpha-adr€nergic receptor blockers: cause dilation ofarterioles and veins and reduce peripheral vacular resistance.Two tvpes: . L Cardioselective alpha-blockers (alpha sreceptor (Corya blockert: examples includ€ doxazosin fcdr"dut tl). pf^zosin (Mi iprcss)^11d,ter,'zosin l. \on-cardioseleclive (alphal ir., and (Hvtrin). and alpha) alpha-blockcrs: cxamples include phentolamine phenoxybenzamine (Dibe zNline.). Angiotensin-converting enzyme inhibitors (ACE Zet ti l)- r^fiipril (Altace) Inhibitors), examples includc lisinoprll (P and etral^pril (Vdsolec/i inhibit thc convcrsion of inactive angiotensin I to the angiotensin II, a vasoconstrictor. This results in peripheral vasodilation and secondarily, an in- crease in urinary volumc cxcrction. Both actions result in reduced blood pressure. . Angiotensin II receptor blockers l anl and candesaftalr f/nBt: examples include losartan (Cozaar), \alsartan (Dio- (Atacand). ARBS block the effects of angiotcnsin II by blocking the binding ofangiotensin Il to its receptors. They do not effect bradykinin. . Calctum channel blockers examples include Verapamil (Calail), Azem /Cardizen), and nifedipine (Procardia): ^mlodipine lNorvasc), inhibit calcium entry into vascular smooth muscle causing vasodilation ofcoronary and peripheral blood vessels thus iowering blood pressure.

Antacids do not prevent the overproduction of acid Antacids do neutralize the acid once it is in the stomach The first statement is true; the second statement is false The first statement is false; the second statement is true Both statements are true Both statements are false

Both statements are true Gastric antacids are drugs that directly neutralize the gastric acrd (HCL) secreted in the stomach. Antacid therapy is directed at decreasing the concentration and total load ofgastric acid. Some common over-the-counter antacid products: . . . . . . *** Sodium bicarbonate products. - Alka-Seltzer Calcium carbonate products: - Amitone - Tums Aluminum hydroxide products: - Altema GEL - Amphojel Magnesium hydroxide products: - Milk of magnesia Bismuth salt products: - Pepto-Bismol \Iagnesium and aluminum products: - Maalox - \4ylanta Aluminum hydroxide is the most potent of these but has less neutralizing capacity than calcium carbonate or sodium bicarbonate. Note: Dyspepsia means an impairment ofthe power or function ofdigestion.

Epinephrine is the agent of choice for treating an anaphylactic reaction because of its stimulatory effects on both alpha and beta adrenergic receptors. All ofthe following are desirable effects of epinephrine that make it the agent of choice for treating an anaphylactic reaction -It has vasopressor activity -It has bronchodilator properties -It causes increased cardiac output -It has a rapid onset of action The first statement is true; the second statement is false The first statement is false; the second statement is true Both statements are true Both statements are false

Both statements are true Epinephrine is the agent of choice for treating an anaphylactic reaction because of its stimulatory effects on both alpha and beta adrenergic receptors. Alpha receptor stimulation results in a vasopressor response (elevation ofblood pressLtre). Beta receptor stimu- lation results in air way dilation and increased cardiac output. In this way, epinephrine counteracts the vascular effects of histamine-related anaphylaxis. Note: Epinephrine is administered either intravenously, sublingually, subcutaneously or intramuscularly. There is a very rapid onset of action when given by these routes. Other therapeutic indications for the use ofepinephrine: . To control superficial hemorrhage . As a component of local anesthetic solutions to prolong duration ofanesthesia .Asthma bronchodilatorproperties

Celecoxib (Celebrex) is in which category of drugs Salicyclates Opiates COX-2 selective inhibitors Non-selective COX inhibitors Steroidal anti-infl ammatories

COX-2 selective inhibitors Cyclooxygenase, or COX, is the enzyme which produces prostaglandins. Two forms of COX exist: COX-I, and COX-2. The COX-I enzyme produces prostaglandins in the GI Tract. The prostaglandins formed act as a protective substance against the formation of gastrointestinal ulcers. The traditional NSAIDs such as ibuprofen, naproxen, aspirin. and others inhibit the COX-I enzyme thus diminishing the fomation of the protective prostaglandins. Gastrointestinal ulcers are therefore a potential adverse effect with these drugs. The COX-2 enzyme produces prostaglandins at the sites of surgery infection and inflammation. When this enzyme is inhibited, less prostaglandins are produced and there is less pain and inflammation. The traditional NSAIDs such as ibuprofen, naproxen and aspirin inhibit COX-2 along with COX-I. Thus they are effective in reducing pain and inflamrnation, but are capable of inducing gashointestinal ulcers. The COX-2 selective inhibitors will reduce pain and inflammation without any significant risk ofcausing gas- trointestinal ulcers. For the traditional NSAIDs such as ibuprofen, naproxenJ and aspirin, because they in- hibit both COX-l and COX-2 enzymes, they belong to the category of non-selective COX inhibitors. For celecoxib (Celebrex), because it inhibits COX-z enzyme only' it belongs to the category of COX-2 selective inhibitors. The COX-2 selective inhibitors: - - Are not salicylates because they are not aspirin drugs Are not opiates because they do not work like morphine Are not steroidal antiinflammatories because they are not corticosteroids such as hy- droc ort isone

Nlcotine is rapidly absorbed across the pulmonary caplllary membrane and is delivered to the brain in high concentration within seconds of inhalation. The nasal spray has the fastest delivery of the NRTs and more closely resembles the onset of the nicotine effects of smoking. The first statement is true; the second statement is false The first statement is false; the second statement is true Both statements are true Both statements are false

Both statements are true \fan) phannacologic approachcs havc bccn uscd to hclp pcoplc placcnlcnt thcrapy l\'Rl stop smoking. h current usc arc thc nicotinc rc- products, bupropion r'4'han ll/elbutriD zt\d \arcnicline (Cha tn). \icotine rcplaccmcnt producls are meant to help palients stop smoking. They arc not mcant to belp gct a paticnt rhrough a long flight or an all-day mccting in a nonsmoking building. .'lhcp^tch (trunsdamdl) is thc most commonly used tvpe ofNRT' Patchcs arc availablc in l5 and 2l mg strcngths for trcatmcnt inilialion. 2. Thc gum dclivcrs nicotinc faster than thc patch and can be uscd for incidcncc ofcraving. Thc gum is availablc in two strcngths: 2 mg and 4 mg. 3. Thc lozenge also comcs in 2 mg and 4 mg doscs and dclivcrs about 25% morc nicotinc than gum. Thc nasal spray has thc fastest delivery ofnicotinc ofthc NRTs. 5. Inhaled nicotine (xslng .1. the nirctine ifihaler) is absorbcd in lhc mouth not in the lungs. Each cartridgc contains l0 mg nicotinc and dclivers 4 mg. It also contains I ng mcnthol. 6. Bupropion as a systcmic medication sccms to rcducc thc craving forcigarcttes orlhc urgc !o smokc. Its mcchanism ofaction is unclcar 7. !arenicline is thc ncwcst dmg in thc smoking ccssation arscnai. Note: Thc FDA has issucd an alcrt to providcrs to monitor patients taking this medication for dcpressior, scvcrc mood swings, abnormal drcam statcs. and thoughls ofsuicidc.

All of the following drugs are direct vasodilators EXCEPT Hydralazine (ApresoIine) Diazoxide (Proglycem) Captopril (Capoten) Sodium nitroprusside (Nipride) Nitroglycerin (Nitrostat)

Captopril (Capoten) Captopril is an angiotensin-converting enzyme inhibitor. Direct vasodilators exert their antih)?ertensive effect by a direct vasodilator action on the smooth muscle of arterioles, resulting in a decrease in peripheral resistance and blood pressure. Compensatory responses may be marked and include salt retention and tachycardia. Adverse side effects include GI upset, headache, dizziness and tachycardia. Not€: nitroprusside, diazoxide, hydralazine and nitroglycerin are parent€ral vasodilators which are used in hypertensive emergencies. ?J]'dhavebeen applied to the management ofhypertension. Vertpamil (Calan SR), Nifedipine (Procardia), and Calcium channel blockers are also effective vasodilators firclirect) Diltiazem (Cardizen) have been given orally for the treatment of mild to moderate hy- pertenslon. Remember: Hydralazine and minoxidil are peripheral vasodilators.

All these drugs maay be useful in controlling salivary secretions to help in obtaining a dry field EXCEPT Atropine sulfate Carbachol Glycopyrrolate (Robinul) Belladonna derivatives Propantheline bromide (Pro-banthine)

Carbachol Carbachol is a direct-acting cholinergic drug. The other drugs are all classified as anticholinergics. They block postganglionic cholin- ergic fibers. Contraindications to their use include: . . . . Glaucoma Cardiovascular problems Obstruction of the GI or GU tract Asthma These drugs also reduce spasms of smooth muscle in the bladder, bronchi, and intestine; relax the iris sphincter; d€cr€ase gastric, bronchial, and salivary secretions; decrease per- spiration; and accelerate impulse conduction through the rryocardium by blocking vagal imoulses.

Triazolam (Halcion), a preoperative sedative in dentistry, is metabolized in the liver by the P-450 isoform CYP 3A4 enzyme. Drugs which lnhibit the actions of CYP 3A4 would affect triazolam in which way? Cause an increase in serum levels of triazolam Cause a decrease in serum levels of triazolam Cause no change in serum levels of triazolam

Cause an increase in serum levels of triazolam Triazolam is known to interact with drugs that inhibit its metabolism via the CYP 3A4 enzyme. Drugs that inlibit the metabolic pathway may have a profound effect on the clearance oftriazolam. The resultant effects would be an increase in serum concentrations \\ith an associated unexpected increase in the actions of triazolam. Consequently, triazolam should be avoided in patients receiving very potent inhibitors ofCYP 3,A.4. -\ntifungaf agents (itroconazole, ketoconazole, Jluconazole, joint and damage and de- miconazole, voriconazole) can significantly elevate the serum levels of triazolam resulting in toxicity with thera- peutic doses. These antifungal agents inhibit the CYP 3A4 isoform responsible for heparic metabolism oftriazolam. Thus the normal metabolism oftriazolam is inhibited. Do not administer triazolam to patients taking any ofthese antifungal agents.

Phentolamine mesylate (OraVerse) is a drug used to reverse soft tissue anesthesia and the associated functional deficits resulting from a local dental anesthetic containing a vasoconstrictor. Which statement best describes its mechansim of action? Causes vasoconstriction and decreased blood flow in infection area Prevents the influx of sodium into the neuron by blocking neuronal channels Causes vasodilation and increased blood flow in infection area Prevents the efflux of sodium out of the neuron

Causes vasodilation and increased blood flow in infection area Phentolamine rnesylate (OraVerse) rs an alphat-adrenergic receptor t locker. Phento- lamine mesylate competitively blocks alpha-adrenergic receptors to produce brief an- tagonism of circulating epinephrine and norepinephrine and antagonism of the r asoconstrictor in the anesthetic preparation. This results in vasodilation of the blood vessels. The local anesthetic is thus carried away at a more rapid rate from the injection site resulting in a more rapid retum to normal nerve sensation.

Which of the following antibiotics is considered a broad spectrum antiobiotic Penicillin VK Cefaclor(Ceclor) Penicillin G Erythromycin

Cefaclor(Ceclor) Cefaclor is a member ofthe cephalosporin family ofantibiotics. The cephalosporins are peni- cillin-like in action against bacteria. They are bactericidal antibiotics and act like the peni- cillins and interfere with cell wall synihesis through inhibition of the synthesis of the peptidoglycan in the cell wall. The antibiotic binds to the enzymes e., transpeptidase) that build/maintain the cell wall. This makes the cell wall osmotically unstable. Bacteda eventu- fi. ally lyse, resulting in death ofthe cell. Cephalosporins act against a wide range ofgram-positive and gram-negative bacteda. Cunently there are four generations of cephalosporins. Each newer generation has signifi- cantly greater gram-negative antimicrobial properties than the preceding one, and a de- creased maximum acti\ ity against gram-positive organisms. Important c€phalosporins within each generation: . First: cephalexin (Keflex), cephradine (Velosqf, cefadroxrl (Duncefl, cefazoin (AnceJ). Most active against S. aureus, Group A Beta-hemolytic streptococci, and Pneumococcus. . Second: cefaclor (Ceclor), cefuroxime (Ceftin), cefoxrtin (Mefoxn), cefprozrl (Cefzil), cefpodoxime /Zartlr). Still have eJficacy against gnm-postivie organisms but also possess good activity against E. coli and H. influenzae. . Third: cefixime (Suprax), cefoperazoqe (Cefobid), ceftriaxone (Rocephir. Have the broadest spectrum ofactivity ofall cephalosporins and are extremely effective against gram- negati\e organisms. They are more active against E. coli, Klebsiella pneumoniae, Enteroh:rcrer. Salmonella. and Shigella. . Fourth: cefeprme (Maxipime). Effective against Pseudomas aeruginosa. \ote: Approximately 107o of individuals expressing allergy to the penicillin family of antibiotics will have cross allergenicity to the cephalosporins. Remember: Antibiotics containing this betalactam ring are referred to collectively as beta- lactam antibiotics and include the penicillins, cephalospodns, and the two newer groups of aeents. the carbaDenems and the monobactams.

Which of the followlng bacterial enzymes belong to the fanily of beta-lactamases? Cephalosporinase Penicillinase ATPase Protein kinase

Cephalosporinase Penicillinase Beta-lactamases are €nzymes produced and secreted by a wide range of gram-positive and gram-negative bacteria as a defense weapon against cephalosporin and penicillin antibiotics. These enzymes destroy the beta-lactam nucleus within these antibiotics by splitting open the betalactam ring structure. This action renders the antibiotic ineffectir e. Those beta-lactamases that work against cephalosporins are called cephalospori- nases, and those that work against penicillins are called penicillinases. By combining clavulanic acid with a penicillin, the betalactamase enzyme is penna- nently inhibited by the acid, and the antibacterial activity ofthe penicillin is n.raintained. One popular commercial preparation is Augmentin, which contains arnoxicillin and clavulanate potassium. Augmentin is used orally as pill or liquid form. Sulbactam is an- other beta-lactamase inhibitor lt is available for intravenous and intratnusculat use com- bined with ampicillin under the brand name Unasyn.

All are catecholamines EXCEPT Epinephrine Norepinephrine Chlolpromazine Isoproterenol Dopamine Dobutamine

Chlolpromazine Chlorpromazine is a dopamine antagonist and antipsychotic agent. Catecholamin€s are any one ofa group of sympathomimetic compounds composed of a catechol molecule and the aliphatic portion ofan amine. Some catecholamines are pro- duced naturally by thebody (called endogenous) andftnction as key neurological chem- icals (i.e., epinephrine, norepinephrine and dopamine). Note: Epinephrine, \orepinephrine and Isoproterenol are considered to be direct-acting catecholamines.

A sedative often used in the management of anxious pediatric dental patients Pentobarbital Secobarbital Chloral hydrate Meperidine

Chloral hydrate Traditionally chloral hydrate has been used orally in the preoperative managem€nt of the anxious pediatric dental patient. It has an unpleasant odor and a bitter, caustic taste, which can be partially masked in a flavored syrup. Chloral hydrate is rapidly absorbed after oral administration with an onset of 15 to 30 minutes. Its duration ofaction is about 4 hours. For use in children, chloral hydrate is available as a 500 mg/5 ml solution. The usual dose for a child is 50 mg/kg up to a maximum of lg. Important: Children will often enter a period ofexcitement and irritability before becoming sedated. It affects brain centers that control wakefulness and alertness. Chloral hydrate does not relieve pain. ;'Nnt"q" "'"ee;;l l. Chloral hydrate is a prodrug and is metabolized to the active metabolite, trichloroethanol. 2. Chloral hydrate's metabolite (trichloroethanol) may displace warfarin from its protein binding sites resulting in an increase in the hypoprothrombinemic re- sponse to warfarin. 3. Sedative effects and/or respiratory depression with chloral hydrate may be additive with other CNS depressants - monitor for increased effect. These other CNS depressants include ethanol, antidepressants, narcotic analgesics, and benzodiazepines. ,1. Be careful when using Chloral hydrate, serious toxicity can result if the dose is not controlled.

Which antibiotic may cause bone marrow disturbances (aplastic anemia) and has limited use due to its side effects Penicillin Tetracycline Chloramphenicol Doxycycline

Chloramphenicol Chlorarnphenicol is a broad-spectrum antibiotic effective against gram-positive and eram-neqative bacteria and against anaerobes. It is used as a second or third line drug in medicine to treat serious infections due to organisms resistant to other less toxic antibi- otics. For example it can be used to treat the following: Typhoid Fever, Bacterial Menin- gitis. Anaerobic Infections, Rickettsial Diseases, and Brucellosis. Chlorarnphenicol inhibits protein synthesis in bacteria and, to a lesser extent, in eukary- oric cells. The drug readily penetrates bacterial cells, probably by facilitated diffusion. Chlorar.nphenicol acts primarily by binding reversibly to the 50s ribosomal subunit. The most important adverse effect of chloramphenicol is on the bone marow Chloramphenicol affects the hematopoietic system in two ways: . by an non-dose-related idiosyncratic response manifested by aplastic anemia, lead- ing in many cases to fatal pancytopenia . by a dose-related toxic effect that presents as anemia, leukopenia, or thrombocytope- nla Important: The risk of aplastic anemia does not contraindicate the use of chloramphenicol in situations in which it is necessary; however, it emphasizes that the drug should never be employed in undefined situations or in diseases readily, safely, and effectively treatable with other antimicrobial agents. Note: Fatal chloramphenicol toxicity may develop in neonates, especially premature ba- bies" when they are exposed to excessive doses of the drug. The illness, the gray baby syndrome, usually begins 2 to 9 days after treatment is started.

All are H1 receptor blockers EXCEPT Diphenhydramine HCL (Benadryl) Chlorpheniramine maleate (Chlor-Trimeton) Cimetidine (Tagamet) Loratadine (Claritin) Desloratadine (Clarinex)

Cimetidine (Tagamet) Cimetidine is an antihistamine H2-receptor blocker. Antihistamines are antagonizing agents that compete for receptor sites with natuml histamine, which is stored preformed in cyoplasmic granules of tissue mast cells and blood ba- sophils. It is released in response to lgB-mediated (immediate) allergic reactions and plays an important role in hay fever, urticaria and angioneurotic edema. Note: Histamine also plays an important role in the control ofacid secxetior] (HCL) in the stomach. There are two t)?es of histamine receptors, H1-receptors, which play an extremely important role in allergic reactions and H2-receptors, which are important in gastric acid secretion. The antihistamines are divided into H,and H2-receptor blockers depending on the histamine receptor they compete for. H1 receptor blockers include: 1. First-generation (classical) agetts: . Diphenhydramine HCL (Benadryl) and chlorpheniramine maleate *** (Chlor-Trimetron) These agents have a broad spectrurn of action which includes antihistaminic, an- ticholinergic, antiserotonergic, antibradykinin and s€dative propefiies. l. Second-generation agents: . CetiiztneHCL ratadne (Clarinex) *"* Qfrlec), fexofenadine HCL (Allegra),lorctadine (Claritin) and deslo- Most ofthese agents, because oftheir poor CNS penetratjon (do not ctoss the bloodhrain barrier). cause less sedation and drowsiness than the first-generation agents. lmportant points: All ofthe H,-receptor blocke$ do not preYent the release of histamine but rather compete with free histamine for binding at Hl-receptor sites. ln general, the binding is competitive. however, some second-generation agents bind non-competitively at higher doses. Common side effects include drowsiness, dizziness, anticholinergic effects and thrcat), and nausea. They can both stimulate and depress the CNS.

Tetracyclines are the drugs of first choice in the treatment of all of the following EXCEPT one. Mycoplasma pneumonia Chlamydia infections Rickettsial infections Staphylococcal infections

Staphylococcal infections Popular Tetracyclines include: . Tetracycline: used to treat acne, gonorrhea and syphilis in patients allergic to peni- cillin. exacerbations of chronic bronchitis, Mycoplasma infections and Chlamydia infections, and Rickensia infections. . . \Iinocycline Dox) clclin€ (Minocin): used to treat acne, anthrax and meningococcal prophylaxis. (Vibramycin)| used to treat infections caused by Rickettsia, Chlamydia and \{ycoplasma; altemative to mefloquine for malaria prophylaxis and treatment of slphilis.

Whlch antlbiotic is used cautiously du€ to its side effects (pseudomembranous colitis, severe GI upset)? Azithromycin (Zithromax: Z-Pak) Clindamycin Penicillin VK Cephalexin (Keflex)

Clindamycin

Which antibiotic is not only effective against most staphylococci, aerobic and anaerobic streptococci but is most effective in treating infections due to bacteroides species Penicillin VK Erythromycin Cephalexin (Keflex) Clindamycin (CIeocin) Vancomycin Tetracycline

Clindamycin (CIeocin) Clindamycin binds to the 50s ribosomal subunit, blocking bacterial protein synthesis. Its use is restricted by its side effects such as severe diarrhea and pseudomembranous colitis. These side effects are caused bv the oversrowth of the bacterium known as Clostridium difficile. Clindamycin is bacteriostatic and is active against most gram-positive (i.e., Streplococ- cus pneumonioe, viridans, antl pyogenes as v'ell qs Staph),lococcus aureus) and many anaerobic organisms, including the anaerobic gram-negative bacteria Bacteroides frag- ilis. In dentistry, clindamycin is an alternate antibiotic in the following situations: . When an.roxicillin cannot be used for the standard regimen for prevention of bacter- ial endocarditis in patients undergoing dental procedures . For treatment ofcommon oral-facial infections caused by aerobic gram-positive cocci and susceptible anaerobes . For prophylaxis for dental patients with totaljoint replacement Important: Clindamycin can be given to patients allergic to penicillins since there is no cross all€rgenicity between penicillins and clindamycin. \ote: \bncomycin is given IV and is used most often in serious or life-threatening staph)lococcal or streptoccocal infections. It remains the drug ofchoice for severe cases ofClostridium difficile. Adverse effects include ototoxicity which may be pemanent and the "red man" syndrome which is characterized by a sudden and profound fall in blood pressure with or without a maculopapular rash over the face, neck, upper chest, and exEemlnes.

All are classified as low moleculr weight heparin type anticoagulants EXCEPT Enoxapadn (Lovenox) Dalteparin (Fragmin) Tinzaparin (Innohep) Clopidogel (PIavix)

Clopidogel (PIavix) Clopidogrel . - - _ :tyotedr -.... fPlarrt inhibils blood clolting by inhibiting platelet aggregttion in an irreversible manne. L Thcsc typc ofanticoagulants agcnts f/olr mols.ular \|eight hepdin, arc uscd to treat acute symp_ tomadc dcep vcin lhrombosis. Tlrcy arc uscdlo preventdccpvcin thronlbosis following knce orhip su.- :,: scry' l;i;*:.- 2. Thcy arc administcrcd subcutancously sincc they arc unablc lo bc absorbcd frorn the Gl tract. 3. Standard heparin consists ofcomponcnts wilh molccolar wcights ranging from 4.000 to 30.000 dal- tons with a mean of 16,000 daltons. Low molecular-lieight heparins rangc in molccular wcights from 2,000 to 8,000 daltons. 4- Heparin acts at multiplc sites in ihc coagulatior systcm and binds with antithrombin III at two spc- citic sites, rcsulting in irs anticoagulanl cffcct- At thc first sitc factor Xa is tlcutlalizcd and at ihc scc- ord silc factor lIa A,rr?rrbr, is ncutralizcd. 5. Low molecular tleight heparins havc a small eflect on hihit factor Xa hol lla). 6. Thc antidote fbr heparin is pa(ial thromboplastin timc but strongly in- protamine; thc antidotc lbr warfarin is vitamin K.

Which trical anesthetic has no place in the routine practice of dentistry? Articaine Cocaine Lidocaine Bupivacaine Prilocaine

Cocaine Cocaine is a naturally occurring ester ofbenzoic acid and was the first local anesthetic used in dentistry and medicine. It is potent and extremely toxic, and it is the only local anesthetic that causes definite vasoconstriction (all other locql anesthetics sodildtors). Cocaine is commercially available in a variety of forms and is applied to mu- cous membranes of the oral, laryngeal, and nasal cavities for use as a topical anesthetic. Cocaine causes significant euphoria (most likely due to its blockqde of reuptake of dopantine in the bralzl, and abuse can lead to a physical dependence. Despite being an excellent local anesthetic, the risk of abuse and the intense local vasoconstriction it pro- duces prevent cocaine from being more widely used clinically. Important: Cocaine has no place in the routine practice of dentistry. The pharmacology ofcocaine is unique among local anesthetics in that the drug inhibits the uptake ofcatecholamines by adrenergic newe terminals. It therefore potentiates the ac- tion ol endogenously released and exogenously administered sympathomimetic amines such as dobutamine, dopamine, or epinephrine. It increases the risk of developing car- diac arrhythmias and hypertension (i.e., $)mP at homim e t ic am i n e s ). cocaine incresses the pressor activity oJ these

The following drugs belong to what pharmacuetical class of agents -Darbapoieten alpha -Pegfilgrastim -Sargramostin Immune modulators Monoclonal antibodies Colony stimulating factors Interferons Immunosuppressants

Colony stimulating factors These drugs stimulate the production ofneutrophils and erythroid progenitor cells in the hematopoietic process. . Darbepoetin alpha: induces erythropoiesis by stimulating the division and differ- entiation of erythroid progenitor cells. It is used to treat anemia associated with chronic renal failure. . Pegfilgrastim Qtleulasta): stimulates the production, maturation and activation of neutrophils. It is used to decrease the incidence of infection by stimulation of granu- locyte production in patients with nonmyeloid malignancies. . Sargramostin (Leukine)z is used for myeloid reconstitution after autologous bone marrow transDlantation.

All of the following are clasasified as corticosteroids EXCEPT Hydrocortisone Methylprednisolone Prednisone Compazine Triamcinolone Dexamethasone

Compazine Prochlorperazin e (Compazine) is an anti-psychotic medication in a group ofdrugs called phenothiazines. T\\'o r-ypes of corticost€roids: L Glucocorticoids affect carbohydrate, lipid, and protein metabolism. They are used as to treat numerous disorders, primarily through their anti-inflammatory and immunosuppresslve acnons. 2. Mineralocorticoids regulate sodium and potassium metabolism. l. These dmgs are used to treai a variety ofconditions which include asthma, arthritis, allergies, aphthous stomatitis, lupus erythematosus, and TMJ pain. 2. Contraindications to their use include latent infections (fingal, viral, or bacte' ndr, AIDS, herpes infections, gastric ulcers, and congestive heart failure. 3. Adverse reactions include Cushing's syndrome (obesit)' qnd weakening oJ uus- cles), hyperglycemia, osteoporosis, peptic ulcers, and an increased risk of infection. 4. Corticosteroids do not cure any disease. They represent replacement only in Ad- dison's disease. Inhaled corticost€roids used for asthma do not achieve significant blood levels to cause the advene effects listed above for systemic agents. Populal aerosol corticosteroids are triamci- nolone (Azmacort), beclomethason€ freconase),fluticasone (Flovmt) andbudesonide nicott). Localized infections with Candida albicans occur frequently in the mouth and pharyrx with repetitive use of inhalant corticosteroids. Nasal spray corticosteroids used for seasonal allergies also do not achieve significant blood levels, and are use for their localized effects. Popular nasal spray corticosteroid triamcinolone fNasocorf , fl uticasone (Flonase) and budesonide (Rhinocort). (Pal- products are

What are four criteria to consider when selecting an analgesic agent for a patient? Concurrent medication Pregnancy Type of pain Location of pain Age of patient Sex of patient Patient's weight

Concurrent medication Type of pain Age of patient Pregnancy Criteria considered when selecting an analgesic agent: l. Type ofpain: . . Severe Mild . Moderate 2. Ag€ ofthe patient: . . . . Infant ) When calculating dosage, the height, weight, body surface area and renal hepatic Child .f function must be taken into account Adult Elderly: drug response is affected by age-related changes in physiology and pharmokinetics 3. Corcurrent medications: Consider unwanted interactions, especially with the elderly. 4. Pregnancy: Because virtually any drug a pregnant womar takes can cross th€ placenta and enter the fetal circulation, drug use in pregnant patients is a source of special concem (clrcck with patient s OB/GYN). FDA Rating System for the T€ratog€nic Eff€cts of Drugs: The FDA, the govemm€nt agency that ovemees the safety ofdrugs, provides the most widely used system to grade the teratogenic effects ofmedications. The FDA assigns a safety category for medications by using a slefter system: A, B, C, D, and X, with A being considered the most safe. X category medications: Studies in animals or humans have demonstrated fetal abnormalities or there is positive evi- dence of fetal risk based on adveme teaction reports from investigational or marketing experience, or both, and the risk ofthe use ofthe drug in a pregnant woman clearly outweighs any possible benefit (/or example, safer tlrugs or otherfotms of therapy are available).

The major natural glucocorticoid is Triamcinolone Cortisol Dexamethasone Prednisone Prednisolone

Cortisol Tte corticosteroids are steroid hormones produced by the rdrenal cortex, They consist oftwo major groups: l. Glucocorticoids: have irnportant effects on metabolism, catabolism, immunc responses and inflammation. Tle majo ty ofthe anti-inflammatory and immunosuppressive actions ofthe glucocor- ticoids are probably the result of their action on arachidodic acid metabolism. They induce the synthesis of a protein that inhibits pbospholipase A2, thus decreasing the production of both prostrglandins and leukotrienes. The major natural glucocorticoid is cortisol. The synthetic glucocorticoids include hydrocortisone (Cortef), .ortisone, prednisone (Delatasone), predrisolone (Delta-Cortef), dexamethasone (Decadtuh), triamcinolone (Aristocort), firethylyprednisolone fMed,"d/), and betamethasone s/or?e/. Glucocorticoids are most often used as anti-jnflammatory and immunosupprcssive agmts. Adverse effects ofthe short-term administration ofsystemic glucocorticoids include secondary infections, hlperglycemia, and a range ofmood and behavioral changes. Loog-term therapy may cause osteoporosis, cataracts, hypertension, myopathy, and adrenal ins!fiiciency. 2. Mineralocorticoids: regulate sodium and potassium reabsorption in the collecting hrbules ofthe kidney. The major natural mineralocorticoid in humans is aldosterone. Other mineralocorticoids in- clude deoxycorticosterone and fludrocortisone. Mineralcorticoids are used in replacemcnt thcrapy in hypoadrenocorticism or Addison's disease. \ote: Corticosteroids do not cure any disease. They represent replacement only in Addison's disease. Contraindicrtions to conicosteroid use include: latent TB or fungal infection, AIDS, herpes infections and patients with peptic ulcer disease pephc ulcers. (specrfcally, gastric ulcers) -these drugs themselvcs may cause Toric effects ofthe corticostercids include growth inhibition, hlperglycemia, osteoporosis, psychosis and salt retention

The symptoms of chronic caffeine consumption can include all EXCEPT Feelings of anxiety and nervousness Sleep disruption Coughing Irritability Diuresis Stomach complaints Palpitations and arrhlthmias

Coughing Caffeinism is a term used for people who are dependent upon catTeine (i.e., sulfer side efects li'ctn hating too much calfeine, tdke l.tryer"amounts and neetl to keep drinking calfeine to function properll).lt is thought to occur ifyou have an intake ofabove 600 to 750 mg ofcaf- terne per day /rrore than 10 cups of coffeel. Drinking more than 1000 mg per day is well into :he to\rc range. Important: Caft'eine stimulates the CNS unequally, with the cortex being the most and the :oinal cord beine the least excited.

When a drug is administered repeatedly, a higher coneentntion of the drug than is desired may be achieved. The effect of this excessive accumulation is knorvn as: Additive effect Synergistic response Cumulative action Idiosyncrasy

Cumulative action An additive effect occurs when additive drugs are administered. The response is no greater than that which would be expected had the drugs been given one at a time. There is no enhancement ofpotential ofthe individual drugs as a result ofbeing used in com- bination. A synergistic r€sponse occurs when the combined action of two drugs is greater than the sum oftheir individual actions. Examples of synergism: Alcohol is synergistic with diazepam (Valium), narcotics, barbiturates and phenothiazines. lt should be avoided if taking these drugs. Idiosyncrasy is a response to a drug that is unusual or abnormal or one that grossly deviates from the routine reaction. Tle Controlled Substancc Acl of 1970 uses the following critcria fbr inclusion of a drug into onc ofthc fivc schcd-

St johns wort is a herbal supplement which can interact with some conventional medications to Increase the effectiveness of medications Decrease the effectiveness of medications Increase the toxicity of medications

Decrease the effectiveness of medications St Johns Wort may induce the cytochrome P450 enzyme system to result in a more rapid metabolism ofmany drugs in rhis pathway. More rapid metabolism of a drug equates to less active drug available and less c/edre4 effectiveness. Specifically, data indicates probable intcractions betwe€n St Johns Wort and iis effect in decreasing the actions ofihe following drugs: . . The HIV-l protease inhibitor indinavir The antirejection medication cyclosponn . The sedative midazolam . The antihypertensive nifedipine Drug-drug interactions can involve prescription or nonprcscription fo),er-the-counter) drrJgs. Dnrgs interact by acting at lhe same receptor or signal transduction paihway, or more commonly, a drug may af]'ect the phar- macokinetics ofanother dnrg. The most common form ofdrug-drug inleraction is one drug affecting the metabolism ofanother drug, involving eifier induction or inhibition ofmetabolizing enzymes. . I nd uction of metabolism : is a reaction to certain drugs in *,hich the number of liver enzymes increases, resulting in a reduction in the €flic! ofthe other drug. . Inhibition ol metabolism: is a process by which one drug either competes for metabolism ofanolher or directly inhibits drug-metabolizing enzymes. l. Herbal medicines such as g.rlic, ginseng, and ginkgo /gir,(go 6i1o6., are thought to interact -\otee: *ith anticoagulant or antiplatelet th€rapy. Remember: Chamomile contains coumarin and '' should not be used in patients taking warfarin for fear ofan additivc affect. 2. St. John's wort is the herbal product most often reported !o be involved in drug-herb interac- l. Echinac€a has potential interactions with immunosuppressants. 4. Many herbal weight loss and "herbal speed" products rely on the pharmacodynamic interaction between ephedra and caffeine. Two primary alkaloids containcd in ephedra doepbedrine -have -ephedrine rdditive cardiovascular effects when taken with caf]eine. At higher doses, the ephedra-caffcine interaction has been cited as a cause ofdcath. Important:The rate ofdrug metabolism can vary greatly. depending on the cytochrome p450 isozyme profile ola patient. Over the next few years, genetic screening ofthc c)'tochrome p450 system will be able to iden- riry $hich patients are likely to develop toxicity or drug-drug interactions, and with which drugs. Ultimately, metabolic screening based on studies using highly-specific probcs prescribed. *** fde- and pseu- could bclp decide whether or not a drug is

All of the following are pharmacologic effects of glucocorticoids EXCEPT Stimulate protein breakdown, which results in increased plasma amino acid levels Stimulate gluconeogenesis in the liver and inhibit peripheral glucose use Impaired wound healing Reduce the immune response Decreased lipolysis Inhibit local edema, capillary dilation, migration and activation of white blood cells, and phagocytosis by macrophages Increase hemoglobin concentration and increase the numbers ofcirculating red blood cells and platelets

Decreased lipolysis This is false; glucocorticoids affect the mobilization of fats from areas of deposition. Increased Iipolyisis occurs in areas ofadipose accumulation, and serum fatty acid concentration increases. Other effects of glucocorticoids include an anti-inflammatory action, immunosuppression, and an anti-allergenic action. Note: These effects occur in target cells following the interaction ofthe steroid with a specific glucocorticoid receptor \ote: Beclomethasore, budesonide and flunisolide are special glucocorticoids /used as inhalers) that hale been developed for use in chronic rsthmr and bronchial disease. These agents readily penetrate the airFay mucosa but have very short half-lives aft€r they enter the bloo4 so that systemic effects and toricity are greatly reduced. Remember: The pharmacologic effccts of mineralocorticoids include an increase in sodium rctention and an increase in potassium depletion which can lead to edema and hypertension if excessive and may lead to dehydration and hypotension ifinsulficielt.

AII of the following are effects of insulin EXCEPT one. Decreased gluconeogenesis Increased triglyceride storage Decreased protein synthesis Increased glycogen synthesis

Decreased protein synthesis This is false; insulin increases protein slrnthesis. Insulin is a pancreatic hormone secreted by the pancreatic beta-cells of the islets ol Langerhans and is essential for the metabolism of glucose and for the homeostasis of blood glucose -/at. (it reduces blood glucose by increasing the {onversion to glycogen and Insulin injection is by subcutaneous administration. Remember: Insulin is required in treating type I diabetes mellitus because the beta cells of the pancreas are devoid of insulin. In treating type 2 diabetes mellitus, oral hypoglycemics can often be used because the beta cells are able to secrete insulin, although in a more sluggish manner

Which best describes the mechanism of action of local anesthetics on the nerve axon Decreases sodium uptake through sodium channels of the axon Increases potassium outflow from inside to outside the nerve Increases the membrane's permeability to sodium Increases the excitability of the nerve axon

Decreases sodium uptake through sodium channels of the axon Whcn a local ancsthctic solution is injcctcd ncar thc nervc, thc solutron intcrfcrcs with the uptakc ofsodium from out- sidc to insidc thc ncrvc. Thc localanesthctic blocks thc spccific sodium channels thcrcby blocking thc sodium uplakc. This dccrcascs thc ncrvc cxcitability below a critical lcvcl and nervc impulscs fail to propagalc along lhc axon. Sincc axons carry pain scnsations, thcsc scnsalions lvill nol bc carricd and a blockagc ofpain rcsults. L Local ancsthctics reversibly block ncrvc impulsc conduction and : :. lion al thcir administcred site. :Notc*; 2. 1-n.u1 onesthctics havc no effects on potassium at thc ncn,c axon. l. Small. nonmyelinated ncrvc fibcrs, which conducl pain producc rcvcrsiblc loss ofscnsa- and tcmperaturc scnsations, arc affcclcd firsr, follo$cd by touch, proprioccption fpler.fl/re),and skclctal musclc tonc /rroto,r.

Nitazoxanide (Alinia) is an oral antiprotozol agent used to treat which of the following conditions? Leprosy Malaria AIDS Diarhea caused by Clostridium diflicile Diarhea caused by Giardia lamblia

Diarhea caused by Giardia lamblia *** This type of diarrhea is an intestinal infection also known as Giardiasis, and is the most common protozoan infection in the United States. Antiprotozoal Agents: . Nitazoxanide (Alinia): is an antiprotozoal agent which is used in the treatment of Giardia, and those protozoal infections caused by Cryptosporidium parvr,rm. Its mechanism is to interfere with the electron transfer reaction wilhin the protozoa essential to anaerobic metabolism. , Atol.a'quone (Mepror): is an antiprotozoal agent used to treat Pneumocystitis carinii pnetmonia (PCP) in patients who are intolerant to co-trimoxazole. . Eflornithine (Vaniqa): has orphan drug status for the treatment of meningoencephalitic stage ofTr)?anosoma brucei gambiense tnfection (sleeping srcinesy'. This drug is also indicated to be used as a cream to reduce unwanted hair from face and ad- jacent . areas under the chin. Furazolidone (Furoxone)'. is a antiprotozoal agent used to treat dianhea caused by susceptible Giardias lamblia and Vibrio cholerae. . Jletronidazole (Flab-l): is a synthetic antibacterial and antiprotozoal agent that is eflective against Trichomonas vaginalis, which causes trichomoniasis. In addition, it is one ofthe most effective drugs available against anaerobic bacterial infections. These include infections caused by Helicobacter pylori, Clostridium difficile, Bacteroides species and Fusobacterium species. Note: Alcohol should be avoided because metron- idazole and alcohol together can cause severe nausea, vomiting, cramps, flushing, and headache. It should is not used in pregnancy because of potential adverse effects on the fetus.

All of the following are cholinergic actions except one Slowing of the heart Dilation of the pupils The stimulation of the smooth muscles of the bronchi, GI tract, gallbladder, bile duct, bladder and ureters The stimulation ofsweat, salivary tear and bronchial glands

Dilation of the pupils This false; cholinergic action constricts the pupils. Cholinergic actions: . The stimulation ofsweat, salivary, tear and bronchial glands . The stimulation ofthe smooth muscles ofthe bronchi, GI tract, gallbladdel bile duct, bladder and ureters (i.e., urinotion) . Slowing ofthe heart (bradycanlia) . Constriction ofthe pupils 0n rosrt Important: Acetylcholine is the chemical mediator at all autonomic ganglia and parasympathetic postganglionic synapses. It is also the transmitter substance of the neuromuscular junction in skeletal muscle (local anesthetics prcvent or rcduce the lib- eration of ACh at the NMJ) and sweat glands. Acetylcholine causes an alteration in cell membrane permeability to produce the above actions. .{nticholinergic actions: .The secretions of all glands in the nose, mouth, pharynx and respiratory tract is in- hibited (unpIeasant "dry mouth") . An inhibitory effect on motility throughout the GI tract uritla4 retention) . (tachycardia) . . The heart rate increas€s -\ rise in body temperature Dilation of the puprls (mydriasis) fmal cause constipatio and \ot€: Termination of transmission by ACh takes place primarily by metabolism by acetylcholinesterase located on postsynaptic or postjunctional membranes.

The positive inotropic effect of digoxin (Lanoxin) Is dependent upon a normal cardiac rhythm Directly increases the force of myocardial contractions ls antagonized by beta-blockers All of the above

Directly increases the force of myocardial contractions This positive inotropic effect is independent ofa normal sinus rhythm and adrcnergic stimulation. Thc cardiac gf]cosides are often called because several come from the digitalis (lbxglove) plant. Digoxin /larroxir) is the most versatile and widely used. "digitalis" \otei The) are used to trcat most supraventricular arrhythmias, cardiogenic shock and chronic heart lailure. These drugs help the heart beat more strongly (posilire inotopic effect), more slo\\ly (bradycardia) ^nd more elficientl]. Cardiac gl]cosides inhibit the Na*-K*-AlPase membrane pump by inhibiting the adenosine triphos- phate enz)-mes (transport ATPase or Na-K-ATPase). Na*-K*-ATPasc splits adenosinc triphosphatc in the nel\e and muscle cell and thus provides the energy necessary for transporting sodium across the cell membranc. Key point: This inhibition of thc NalK*-AIPase enzyme leads to an increased calcium ion influx u'hich augm€nts the positive inotropic effect ofcardiac glycosides. Adverse side effects in- clude nausea and vomiting, appetite loss, dianhea, ventricular anhyhmias, heart block, and visual and mental disturbances. Contraindications to thci.use include ventricular fibrillation andventricular tachy- cardia. Drug interactions: Many drugs aff'ect digoxin levels. However, digoxin docs not affect the levels ofother drugs. ln addition, when beta-blockcn are added to digoxin in patients with AV conduction abnormalities, complete heart block can result. Erythromycin. clarithromycin and tetracycline may increase digi- talis absorption and toxicity. Thyroid replacement therapy incrcases dose requirements ofdigoxin. D gs lhiazide and loop diurelics/ increasc digitalis toxicity Drug treatment of mild to moderate heart failure proceeds in the following order:(l) Diuretics in pa- that lower plasma potassium levels ae.&, tients \1ith fluid retention (2) ACEIS orARBS in all paticnts unless contraindicated (3) Beta-blockers in relention) p^tients unless contraindicatcd and (4) Digoxin. Note: Ifthese medications are not suflicient to control heart failure, the following additional drugs may be given: (1) all stable inininalJluitl Spironolactone (a aldosterone antctgorlist) (2) Nitrates and hydralazine and (3) Calcium channel block- ers (i.e., amlodipine and felotlipine only) Remember: Most drugs useful in treating cardiac arrhythmias act primarily by increasing the re- fractory period ofcardiac muscle.

All are cholinergic drugs EXCEPT Acetylcholine Methacholine Bethanechol (Urecholine) Dobutamine Carbachol (Isopto-Carbachol) Pilocarpine hydrochloride

Dobutamine Dobutamine is an adrenergic agonist. Cholinergic drugs are compounds that mimic the actions of the endogenous neurotransmitter acctyl_ chofine (muscorimic or nicotinic or both), to cause a response in an effector. These drugs include ACh, various choline-ester /,4 Clrl. Direct-acting, or cholinomimetic, agents combine with cholinergic receptors congeners ofACh, and some alkaloids. With few exceptions fi.e., nicotine and olher ganglionic stintu- 1drlt, these agents exe.t prominent muscarinic or parasympathomimetic effects. These drugs are longerlasting than ACh because they are not subject to rapid metabolism like ACh. ACh is metabolized by acetylcholinesterase, located near receptors forACh. In the plasma and other sites, ACh (and many olher esters) are rnetabolized by pseudocholinesterase. The other cholinergic agonists used as drugs are metabolized slowly or not at all by these enzymes. Direct-acting agents: . Choline esters: The most noticeable effects ofthese drugs are a fall in blood pressure attributable to generalized vasodilation, flushing ofthe skin, a slowing ofthe heart rate, and an infieased tone and actilit-v of both the CI and urinary tracts. Topical application ofthese drugs to the eye causes miosis and a decrease in innaocular pressure. These dmgs include: . . . . Acet:-lcholine chloride: used in ophthalmology to produce miosis Bethanechol: used for postoperative abdominal distcnsion and urinary rctention Carbachol: used in ophthalmology to produce miosis Cholin€rgic alkaloidsi These drugs include muscarine, pilocarpine, nicotine and lobeline. Pilocarpine is the most useful alkaloid being employed as a miotic and to treat open angle glaucoma Pi- locarpine is also used to stimulate salivary flow in patients suffedng ftom xerostomia due to radiation therapy in the treatment ofhead and neck cancer. Indirect-acting agents: . Cholinesterase inhibitors: These drugs include physostigmine, edrophonium, neostignine, malathion and sarin. They inhibit acetylcholinesterase at both mucarinic and nicotinic sites. They are indirect agonists at both muscarinic and nicotinic sites. Note: Ifany ofthe cholinergic agents are administered before acetylcholine, the action ofacetycholine will be enhanced and prolonged.

Implicit in the interaction of a drug with its physiological receptor is that the drug ? creates effects but rather modulates ? physiological cell functions Does; extrinsic Does not; extrinsic Does; intrinsic Does not; intrinsic

Does not; intrinsic There are four major families ofphysiological receptors that drugs can bind to produce effects: . Receptors as enzymes 6.e., cell surface protein kinases): These kinases exert their regulatory effects by phosphorylating proteins within the cell which alters the cellular biochemical activities. By binding to these kinases, drugs can also cause the alteration in biochemical activities resulting in a drug effect. . Ion channels: Drugs can bind to ion channels in cell membranes to cause opening or closing. This alters the cell's membrane potential to result in a drug effect. . G protein-coupled receptors: When drug binds to these receptors, second messengers are produced such as cyclic AMP to produce an effect within the cell. This results in a drug effect. . Receptors in c€ll nucleus: Receptors for steroid hormones are soluble DNA-tran- scription factors within the nucleus that regulate the transcription of specific genes. Modification ofthe transcriptions ofthese genes results in a drug effect.

AII of the following drugs are serotonin 5-HT1D receptor agonists used to treat migraines EXCEPT Sumatriptan (Imitrex) Frovatriptan (Froya) Naratriptan (Amerge) Donepezil (Aricept) Rizatriptan (Maxalt) Almotriptan (Axert) Zolmitriptan (Zomig)

Donepezil (Aricept) Donepeztl (Aricep, is a cholinesterase inhibitor used in the treatment ofAlzheimer's disease. Migraine medications: . The most commonly used migraine medications are the serotonint agonists, commonly known as the "triptans." f5-11fl receptor All 5-HTl receptor agonrsts (triptans) have a similar chemical structure and a comparable mechanism of action. 5-HTl 5-HTtD receptors located on the extracerebral, intracranial blood vessels that become di)ated during a migraine attack and on nerve terminals in the trigeminal system. Ther- apeutic activity is caused by activation ofthese receptors, which results in cranial ves- sel constriction, inhibition of neuropeptide release, and reduced transmission in trigeminal pain pathways. . Ergotamine derivatives demonstrate partial agonist and/or antagonist activity against dopamine. tryptaminergic, and alpha-adrenergic receptors, depending on their site. Er- gotamine derivatives have three primary actions. They (1) depress central vasomotor centel (2) constrict peripheral and cranial blood vessels, and s and (3) reduce extracranial blood florv and decrease hyperfusion ofthe basilar artery area. Examples include: erotamine tartrate (C aferyo t) and dihydroergotamrne . (D H E - 4 5 ) Combination drugs: Midrin is a combination capsule that contains 65 mg of isometheptene mucale (an ergotamine derivative), 325 mg of acetaminophen, which exerts an analgesic effect, and 100 mg ofdichloralphenazone, a mild sedative that acts centrally to allay an"riety. Note: Caffeine, which is added to many migraine combina- tions. helps to promote constrictive properties, and enhances absorption. Note: Methylsergide (Sansert) for prophylaxis against migraines. blocks 5-HT2 receptors fs erotonin antagotrist) and is used

Of the following which is a clinically significant adverse reaction due to metoprolol Drowsiness Hallucinations Arthralgia Dry mouth

Drowsiness Metoprolol (Lopressor Toprol-XL) is a competitive, betar-selective adrenergic receptor b\ocker (carlioselectlver, which is most similar to Atenolol. It is used in the treatment ofhy- pertension, acute angina pectoris and may be helpful after a heart attack. Atenolol (Tenormin) is a competitive, betal-selective adrenergic receptor blocker (cardiose- lective), wtth alo]ig plasma halflife (1or1g du'ation ofdction.). It is used in the treatment ofhy- pertension and chronic angina pectoris. Due to its low lipid solubility, Atenolol is renally eliminated, minimally metabolized, and has a low potential for causing CNS side effects com- pared to lipid soluble beta-blockers (e.g., Propranololl. *** Metoprolol and Atenolol are both longer-acting and more predictable than Propranolol in producing therapeutic plasma levels. Because they are betar selective adrenergic receptor- blockers, they are also safer to use in patients with a history of asthma or bronchitis. lt is used to treat hypertension and to control ventricular arrhythmias. It has a low lipid solubil- -{cebutolol (Sectral) is a betar-selective adrenergic receptor blocker (cqrdio.selective). in. $hich reduces its likelihood of producing adverse Crr-S effects. lt also has mild intrinsic s) mpathomimetic activ tly (partial agottist actiritl at beta: rc.eptors) similar to Pindolol. Important points: L \\'ith all "selective" beta-blockers, selectivity for the betal receptor is lost at high doses. -\s the dose is increased they also block the beta2 receptors, thereby having eff'ects on bronchial smooth rnuscle. l. The most common side effects ofbeta-blockers are weakness and drowsiness.

Which of the following side effects are most common with the use of diazepam (valium) Rash, itch Mouth, throat ulcers Drowsiness, fatigue Diffiiculty with urination

Drowsiness, fatigue Diazepam is prescribed in the treaftnent ofanxiety, nervous tension, muscle spasm, and as an anticon!'ulsant. Contraindication to use: Acute narrow angle glaucoma. It is used intravenously as the agent ofchoice to reverse status epilepticus induced by a local anesthetic overdose. Diazepam affects the limbic system olthe brain /iori. It has a high therapeutic index, produces some degtee ofannesia, and can be locally irritating to tissue and may produce local thrombophlebitis when given intravenously. \ote: Propylene glycol, which is in the ry mixture, is the main cause ofthrombophlebitis.

Granisetron (Kytril) and ondansetron (Zofran) are selective 5-HT3 receptor antagonists used to treat what condition? Breast cancer Osteoporosis Emesis caused by cancer chemotherapy Mucositis caused by radiation therapy

Emesis caused by cancer chemotherapy The 5-HT3 receptor is a serotonin receptor which when activated during chemotherapy for cancer, causes emesis (nausea and vomiting). Both granisetron and ondansetron are indicated for prophylaxis of chemotherapy-related emesis, prophylaxis of nausea and vomiting associated with radiation therapy, and prophylaxis and treatment ofpostoperative nausea and vomiting (PONV). Other s€rotonin 5-HT3 receptor antagonists include: dolasetron palonosetron @/oxr) \ote: 5-HT3 stands for 5-hydroxytryptamine type 3 receptor. (Anzemet) and

What substance produces all of the following physiological actions Constricts arteriolar blood vessels (vasoconstriction) Relaxes bronchial smooth muscle (bronchodilation) Decrease blood volume in nasal tissues Causes a hypertensive response Produces physiological actions opposite to that of histamine

Epinephrine Therapeutic indications for epinephrine: . . . . To alleviate symptoms ofan acute asthma attack To treat bronchospasm associated with hypotension, as in anaphylaxis To treat hypersensitivity reactions Agent ofchoice for anaphylactic reactions (given sublingually or subcutaneoush)) . It is added to local anesthetics as a vasoconstrictor to prolong the activity ofthe local anesthetic solutions, by decreasing the rate of diffusion and absorption from the injectlon site . . To restore cardiac activity in cardiac arrest To relieve congestion ofthe nose, sinuses, and throat Common side effects: headaches, agitation (anxiei)), and tachycardia. Important: Epinephrine should be used with caution in patients with high blood pressure and hyperthyroidism. These patients may have an increased sensitivity to epinephrine.

All of the following drugs are indirect acting adrenergic agonists EXCEPT Tyramine Amphetamine Epinephrine Methamphetamine Hydroxyamphetamine

Epinephrine Two types ofadrenergic agonists: 1. Indirect-acting: are those that cause the rel€as€ of stored norepinephrine at the postganglionic nerve endings to produce their effects. Tyramine and amphetamines cause release of norepinephrine. 2. Direct-acting: are those that interact directly with the alpha or beta receptors. Direct-acting adrenergic agonists may be receptor selective or receptor non-selective: . Phenylephine . Clonidine : : alphal selective agonist alpha2 selective agonist . Dobutamine = alphat and betal selective agonist Terbutaline - beta2 selective agonist . . . Albuterol : beta2 selective agonist Epinephrine: alphar,2 and betar,2 agonist . Norepinephrine: alphal.2 and betal agonist . Isoproterenol = betal.2 agonist Remember: Some literature refers to direct and indirect-acting adrenergic agonists as direct and indirect-acting sympathomimetic ag€nts, These terms are almost always sln- onl mous. They are agents that bring about tissue responses resembling those produced by stimulation of the sympathetic nervous system.

List the three main reasons why epinephrine is included in local anesthetics

Epinephrine, a vasoconstrictor' is included in dental local anesthetic preparations for the following three reasons: L It prolongs the duration oflocal anesthesia. 2. To provide a hernostasis such that local bleeding is conlrolled or reduced. 3. To delay the absorption ofanesthetic into the systemic circulation thus reducing the chance of systemic toxicities.

Whlch of the drugs below can prolong the QT interval of cardiac electrical conduction and thus can lncrease the risk of cardiac arrhythmias? Erythromycin lbuprofen Hydrocodone Penicillin VK Azrtbromyctn (Z-Pak)

Erythromycin Erlthromycin is one ofthe drugs confirmed to prolong the as ha\ ing a risk ofcausing anhythmias. The rance benveen the Q lng. -{ long in\'olving QT QT point ofthe QRS ), QT interval and is accepted QT interval is measured as the time and dis- complex and the end ofthe T wave in the ECG trac- syndrorr.re was first described in the 1950's as a congenital syndrome interval prolongation and syncope and sudden death. These congenital long QT svndromes were characterized by a peculiar electrocardiographic appearance of the QRS complex involving a premature atria beat followed by a pause, then a subsequent sinus beat showing marked QT prolongation and deformity. This type ofcardiac arrhyth- mia was originally termed "torsade de pointes" (from the french " /R?grt- t**isting of the points "). Erythromycin, an antibiotic used to tr€at bacterial infections, is considered as having a risk of causing torsade de pointes, an unusual adverse reaction for an antibiotic. Clarithromycin, an antibiotic within the erlthromycin family also causes prolongation ofthe QT interval. Azithromycin, another member of the erythromycin family of antibiotics is not associ- ated with prolonging the QT intewal.

Disulfiram (antabuse) is used in the management of Nicotine abuse Ethanol abuse Opioid abuse NSAID abuse

Ethanol abuse Disulfiram is not a cure for alcoholism but is a deterrent to ethanol consumption. Disul- firam is an antioxidant that interferes with the hepatic oxidation ofthe acetaldehyde me- tabolized from alcohol. Specifically it inhibits aldehyde dehydrogenase, a mitochondrial enz]'me found in the liver Even the ingestion of small amounts ofethanol results in high concentrations ofacetaldehyde in the body. The unpleasant reaction that occurs (called the Disulliranr-Ethanol Reaction or DER) consists ofa throbbing headache, dyspnea, throbbing in the neck, nausea, copious vomiting, thirst, tachycardia and hypotension. \ote: lletronidazole, also inhibits aldehyde dehydrogenase. Ethanol is a sedative-hypnotic drug and is the most important alcohol ofpharmacologic interest. Its abuse is responsible for many socioeconomic problems. Drugs that are syn- ergistic with ethanol include diazepan, meperidine, pentobarbital and chlorpromazine. When combined with alcohol these drugs could cause fatal oversedation. Remember: Synergism refers to the combined action oftwo or more drugs that is greater than that achieved with a single drug.

Humulin 70/30 is the brand name for a mixture of insulins containing 70% insulin NPH and 30% regular insulin injection. The advantage of this is what Fast onset, short duration Slow onset, short duration Fast onset, long duration Slow onset, long duration

Fast onset, long duration Humulin 70/30 mixture is a popular form ofinsulin that many diabetic patients take. lts advantage is that after a single injection, the regular insulin component /30o2) fast onset ofblood sugar control beginning one-half hour after injection and the insulin NPH component (702o) starts acting within a couple ofhours to provide a long duration ofblood sugar control. Note: Using either agent alone would not provide both the fast onset of insulin action along with the long duration. Premixed insulins are a combination of specific proportions of intermediate-acting and rapid-acting or short-acting insulin in one bottle or insulin pen (the the brqnd name indicate the percentage of each 4,pe o./ insulin).

Which drug is a synthetic opioid analgesic used as an intravenous sedative and is more potent than morpbine? Meperidine (Demerol) Pentazocine (Talwin) Propoxyphene (Darvon) Fentanyl

Fentanyl Fentan!-l (Sublimaze)is a potent narcotic analgesic used primarily as an intravenous supplement during conscious sedation procedures or proccdures requiring general anesthesia. Fcntanyl is 80- 100 times more potent than morphine. Fentanyl is also avaifable as a lollipop-type lozenge (brand name Actiq) for transmucosal absorp- tion and as a transdcnnal patch (brand name Duragesic) for dclivcry thrcugh a skin. Note: Fcntanyl congeners include Alfentanil, Sufentanil, and Remifentanil. Meperidine patch applied to the (Denerol) is used as an intravenous supplement during conscious scdation proce- dures. However, it is less potent than morphine and much less potent than fentanyl. Meperidine (Demerol) is also wed as an oral medication for pain control aftcr dental surgery. Note: A mctabo- lite, normeperidine, is a CNS stimulant. lmportant; Contfaindicated with MAO inhibilors (e.g., phen;lzine ard tru llcypromine) Pentazocine (Talwin) is chemically related to morphine and has weak analgesic properties. 1t is a mixed agonist-antagonist drug, having agonist activity at some reccptom and antagonist activ- ity at other rcceptors. It is not used intravenously to produce conscious sedation. Pentazocine has abuse liability. Note: Talwin Nx tablets contain naloxone which is added to deter misuse. Propoxyphene fDanon) in the form ofpropoxyphene napsylate with acetaminophen is known as Darvocet-N 100 and is useful tb. pain control after dental surgery. It is taken orally and not used inlravenously. Darvocct-N 100 has low abuse liability. l\-ote: lt is less potent than codeine and a metabolite, norpropoxyphene, is a CNS stimulant. Tramadol (Ultran) is an opioid partial agonist. It is not a scheduled drug. Use great caution in combination with MAOIs (e.g., phenylzine and tranylcypromine).

which corticosteroid is adminsitered by inhalation to treat asthma? Hydrocortisone Prednisone Cortisone Fluticasone (Flonase) Methylprednisolone (Medrol)

Fluticasone (Flonase) Corticosteroids in the inhaled form, decrease the inflammation in the airway in asthma. Reduction of inflammation enhances the bronchodilating effects ofthe beta2-adrenergic agonists. The following are some ofthe other popular inhaled corticosteroids used in the treatment of asthma: . . . . Triamcinolone (Azmacort) Beclomethasone (B Budesonide (Pulmicort) Flunisolide (AeroBid) eco n.tse ) Note: Inhaled steroids very often cause a fungal infection ofthe mouth and throat. Combination products used in the treatment ofCOPD and asthma include: . . . albuterol/ipratropir'l'm (Combivent) fluticasone/salmeterol (Advair Diskus) budesonide/formoterol (Symbicort) Important: Leukotriene modifiers act on inflammatory mediators ofasthma, the LTs lalso htoutr as slo$.reqcting substance of anaphylnris [SRS-A]), which contributes to ainr ay obstruction. There are two subclasses: (l) Leukotriene receptor antagonists which include montelukast (Singulair) and zafirlukast (Accolate) (2) 5Jipoxygenase inhibitor tleukotriene s!nthesis inhibitor) which includes zileuton (Z,tflo).

Oral contraceptives block ovulation by inhibiting which two anterior pituitary hormones below? Follicle stimulating hormone (FSH) Growth hormone (GH) Thyroid stimulating hormone (TSH) Luteinizing hormone (LH) Adrenocorticotrophic horrnone (ACTH)

Follicle stimulating hormone (FSH) Luteinizing hormone (LH) In addition to the above effects, oml contraceptives produce altemtions in the genital trac! including changes in cervical mucus, rcndering it unfavomble for sperm penetration even if owlation occurs. Changes in the en- dometrium may also occur .endering it unfavorable for nidation (lnrp gens and progesterone-like compounds (p/ogertrrs) . lantotion of the are used for oml contraception. -fertili.ed Estrogens: are a group of chemically similar steroid hornones. In humans, estrogens are made primarily in the female ovaries and in small amounts in the male testes and the adrenal glands, brain, and fat ofboth sexes. Estradiol is the most abundant and potent natural estrogen in humans. Other inlcude ethinyl estradiol and mestranol. . Progestins: ar€ a gloup of chemically similar steroid homones as well. In humans, progestins are made primarily in the female ovaries and male testes. Progesterone is the most abundant and potent progestin in humans. Other progestins include levonorgestrel, rorethindrone, medroxyprogesterone, norgestimate and norgestrel. Types of oral contrac€ptives: . Combination: oml contraceptive agents usually contain both an estrogen agent and a Combination drugs include: . . . ethinyf estradiol and norethindrone fovcor ethinyl estradiol and fevonorgestrel fPolria 50, Brevicon 21, and Modicon 28) 0.15/30, Alesse 28 and Aviane 21) thinyf estradiol and norgestrel (Cryselle, Ovral, and Oryestrel) . Progestin-onfy: nor€ fiindrone (Micronor) . Emergency contrrception: levonorgestrol (P/dn B) \\'ernings/Prccautions with OIal Contraceptives: . Tle risk of cardiovascular side effects increases inwomenwho smoke cigarettes, especially those who are >.15 .veaN ofage. . May increase the risk ofthromboembolism. Women with h)?ertension shouldbe encouraged to use anon- hormonal folm of contraception. \ote: Antibiotics have the potential to diminish the effectiveness oforal contraceptives. Advise patients to use additional method ofbirth control when taking antibiotics and oral contmceptives concunently.

When writing a prescription q.i.d. is an abbreviation for Twice a day Every 4 hours Three times a day Four times a day

Four times a day Other popular and common abbreviations used when writing prescriptions: . . . . . . b.i.d. t.i.d. : : twice a day three times a day q.l2.h.: every 12 hours q.4.h. = every 4 hours stat. = immediately h.s. . p.rn. . a.c. : : . h. = ' at bedtime : as needed before meals qfrar ma.la Sig. = Label Note: Always document prescriptions that are given to a patient in the patient's chart, along with the date they were written and any specilic instructions for patient use. Remember: . I grain = 65 mg .l ounce = 30 g or 30 mI

The prototype loop diuretic is Hydrochlorothiazide (HCTZ) Furosemide (Lasix) Spironolactone (Aldactone) Triamterene (Dyrenium)

Furosemide (Lasix) Diuretics are used to tr€at congestive hcart failure by relieving edema and symptoms ofdyspnea arising liom pulmonary congestion. They are also used to treat h].pertension, and in the management ofedema associated $irh heparic or renal disease. Cetegories of widely used diuretics: . Thiazides: inhibit sodium reabsorption in th€ distal portion ofthe renal tubule within the kidney causing increased excretion ofsodium andwater Prototpe agent is hydrochlorothi^zide (HCTZ) Otherexamples include chlorothiazide (Diuril), indapamid. (Lozol), and metolazon (Zarcxolyn). Note: Thes€ drugs can cause hypokalemia (ab nornally lot level ofpotarsium in lhe blood) of sodium in the blood)and may increase plasma uric acid. ,hponatremia (abnormally loN' level . Loop diuretics: inhibit reabsorption ofsodium and chloride in the ascendiog Loop ofHcnle thus causing increased secretion ofwater, sodium and chloride. Proto0pe agent is furosemide include bumetanide (B umer), eth^crynic ^cid (rasir. other examples (E.1ecfin), nnd torcefiide (Demadat) . Note| These drugs can cause hr?eruricemia, tinnitus, hearing loss, hlponatremia and excessive fluid loss. . ** Potassium-sprring diuretics: . Sodit|m channel blockers: these agents inhibit sodium reabsorption through sodium channels in renal epithelial cells. This inhibition creates a negative potential in the luminal membranes ofprincipal cells, located in the distal convolutedtubule and collecting duct. Negative potential reduces secretion ofpotas_ sium andhydrogen ions- Conserve potassium while caus ing diuresis. Thus, no potassium is lost fiom the body as is the case with other diuretics such as the lhiazides and loops- Triamlerene amiloride (Midamor) are examples of this drug group. . Aldost€rone antagonists: thes€ drugs competitively inhibit the aldostercne receptor This causes in- crcased amounts of sodium and water to be cxcreted, while potassium is retained Spironolactone dactone) and epleJerone (1aplal are examples. Fixed-dose combination therapy: Dyazide is the brand name for the combination oftriamterene and hldrochlorothiazide filcfz). eflicacy than eith€r on€ individually. Notei Positive itrotropic dlngs Tlis product combines the (D)'refiiutfi) and potassium-sparing diuretic with HCTZ for geater (drugs that make muscle contract more forcefullv) canbe used in the acute trerlment of heart failure, these include dobutamine, dopamine, inamrinone, and milrinone These drugs may be given intravenously to stimulate heart conhactions and help keep blood circulating They are only used temporarily because long-term use shortens life.

For which of the following conditions can Ranitidine be used Prostatitis GERD (heartburn) Toxic-shock slndrome Renal failure

GERD (heartburn) H2-receptor antagonists compete with histamine at the H2-receptor. Histamine produces a s ide variety ofphysiologic actions in many tissues. While H2-receptors are located in the GI tract and in vascular and bronchial smooth muscle, H2-receptor antagonists com- pete * ith histamine only in the GI tract. Inhibition of histamine at the parietal cell in- terteres u ith one of several mediators for signaling the parietal cell to secrete acid. H'-bfockers incfude: cin.retidine (Iaga met), ranrtrdrne ntzatidire (Axicl).These /dry moutlt, ose (Zantac), famotidlne (Pepcitl) and agents are all rev€rsible competitive antagonists ofthe actions of histamine on H,-receptors. These agents are used to treat acid-peptic disease, espe- cialll duodenal ulcers and occasionally gastric peptic ulcers. These drugs are also used to treat Zollinger-Ellison syndrome disease /GERDJ. (a hypersecretory disease) and gastroesophageal reflux Note: Cimetidine has an antiandrogen effect (can lead to impotence, Ioss oJ libido, and gtnomastia).It also inhibits liver metabolism which can lead to an increase in activity ofother drugs such as warfarin and carbamazepine.

Erythromycin is well know to cause adverse CNS effects GI effects Hematologic effects Renal effects

GI effects Adverse Gl effects are reported for approximately 2lo% ofpatients receiving er)thromy- cin. about 107o ofpatients receiving clarithromycin, and less than 504 for azithromycin. In general, oral bioavailability of erythromycin is poor. It is readily inactivated by stom- ach acid, and several salts have been developed to overcome this drawback. . . **" Erylhomycin stearate (E4tthrocin) Erylhromycin ethylsuccinate ft.t S.) Since they are destroyed by stomach acid, erythromycins are usually entedc coated. This is a term designating a special coating applied to tablets or capsules which prevents their release and absorption oftheir contents until they reach the intestines \ot€: Er,,lhromycin is metabolized in the liveq excretion is mainly via the bile

The most commonly used aminoglycoside Neomycin Streptomycin Tobramycin Gentamicin

Gentamicin Aminoglycosides are potent bactericidal antibiotics that act by creating iissures in the outcr membrane of the bacterial cell. They are particularly active against aerobic, gram-negative bacteria and act synergistically against certain gram-positive organisms. Gentamicin is the most commonly used aminoglycoside, but amikacin may be particularly effective against resistant organisms. Aminoglycosides are used in the treatmcnt of severe infections of the abdomen and urinary tract, as well as bacteremia and endocarditis. They are also used for prophylaxis, especially against endocarditis. Resistance is rare but increasing in frequency. Avoiding prolonged use, volume depletion and concomitant administration of other potentially ncphrotoxic agents decrcases the risk of toxicity. Single daily dosing of aminoglycosides is possible because of their rapid concentration-dependent killing and post-antibiotic effect and has the potcntial for decreased toxicity. Aminoglycosides: . Gentamicin (Garamycit), Amikacin (Aniken), and"tobramycin (Nebci These are effective against serious infections caused by aerobic gram-negative bacteria, including E. Coli, En- terobacter, Klebsiella, Proteus, Pseuodomonas aeruginosa, and Serratia. . Streptomycin: The first aminoglycosidc and q,as berculosis. ls seldon used today. . \eom]cin (Myi/iadn, and Kanamycin used only topically or locally marked lendency to cause ototoxicity. ). shown to be effcctive in the treatment oftu- (Ka trer)i Due to its toxic potential ncomycin is /e.g, in the GI tract). Kanamycin is rarely used bccause of its l. Aminoglycosides may cause severe neuromuscular weakness lasling hours to days \ot6 because oftheir potential curarelike effect. Aminoglycosides may aggravate muscle - rvcakness in patients with muscular disorders such as myasthenia gravis, infant botulism, or parkinsonism. 2. Aminoglycosidcs bind incvcrsibly to thc 30s ribosomal subunit of bacteria, subse- quently inhibiting protein synthesis. 3. Two wcll-known adve$e effects are ototoxicity and nephrotoxicity.

All are useful in treating which medical condition Pilocarpine (lsopto-Carpine) Latanoprost (Xalatan) Betaxolol (Betoptic) Bimatoprost (Lumigan)

Glaucoma Glaucorna is characterized by an increase in intraocular pressure. It is caused by poor drainage of the aqueoushumor (luid in the eye) and can cause blindness. . Pilocarpine (1sop to-Carpine)r eye drops in the eyes causes papillary constriction thus allowing for drainage ofthe aqueous humor to reduce pressure . Latanoprost (Xalatan)z a prostaglandin analog; eye drops in the eye reduces in- traocular pressure by increasing the outflow ofthe aqueous humor . Bet^xolol (Betoptic): abeta-blocker; eye drops in the eye reduces intraocular pressure by reducing the production of aqueous humor . Bimatoprost fZ, miganl same action as latanoprosl (Xalatan)

Of the amino acid neurotransmitters listed below which one is an excitatory neurotransmitter? Glycine Glutamate GABA (gamma-aminobutyric acid) None of the above

Glutamate Chemicals that ransmit the signal from one neuron to the next are called neurotransmitlers They are synthesizcd in the ccll body or nen'e terminal of the pr€synaptic neuron. Neuroffansmitters are rcleased from thc s\napse and cross the synaptic cleft. The d€ndrite on the nerve cell body receives the signal. Various receptors on the posts!naptic membrane ofthe dendrite accept only certain neurotransmitte$. h the brain. 30 differcn! neurotrarlsmitters have been classified as amino acids, amines, and neuropeptides. . Amino acid neutofransmitters: - Glutamate, GABA, and glycine. Glutamate is an excit|tory neurctransmitter. GABA and glycine are inhibitort neurotransmitters. GABA is the major inhibitory neurotransmitter within the CNS. well as serotonin, histamine, and acetylcholin€, - Include the catecholamin€s . \europeptides -most -dopamine, norepinephrin€, and epinephrine -as are also hormones; these include vasopressin, oxytocin, insulin, somatostatin, trin. substance P, endoryhin, and enkephalin. Remember: . Acetltcholin€: effects in CNS generated by interaction with a mixture ofnicotinic and muscarinic recep_ lors . Dopamine: a catecholamine which acts thrcugh at least two subt?es D7 (activates aden![ c\'c|ot"r,r, (inhibits adenyl cjclase) . serotonin: is s-hydroxytryptamine which works through at lcast 14 subreceptor "trtPtominergic" 1. Biosynthetic pathway ofACH: Step (l) Choline (taken up into nenevia action ofpermease) St€p (2) Choline acetylcholinesterase catalyzes the synthesis of Ach liom acetyl CoA and choline 2.Biosynthesis of NE and E: Step (1) Tyrosine to DOPA (enzyme is tvrosine hvdrorylase Step (2) DOPAto Dopamine (enzlme is aromatic L-amino acid de- carborylase) Step (3) Dopamine to NE (enz-vme Step (4) fnorlb is dopamine bela h)dro\lase) in the ddrenal nedulla): NE ro E lenzvme is phen ethanolamine N-meth translbrase)

All are useful in the treatment of what medical condition Colchicine Indomethacin Sulfinpyrazone Probenecid Allopurinol

Gout The therapeutic management of gout involves three different aspects ofthe disease: l. Reducing the inflammation during acute attacks: The drug ofchoice is colchicine. Colchicine impairs leukocytic migration to inflamed areas and disrupts urate deposi- tion and the subsequent inflammatory response. It is most effective when initiated 12 to 36 hours after symptoms begin. Notes: l. Colchicine should never be given IM or subcutaneously (it causes tissue irrita- tion). 2. Colchicine can severely damag€ the liver and kidney; long-term therapy may cause bone marrow depression. Remember: NSAIDs are also important for the treatmenl ofacute gouty arthritis. Indomethacin (rzay cause renal damage or bone maruow depressiar) is most com- monly used. 2. Decreasing uric acid production: Allopurinol (Zyloprin) is the drug ofchoice in the management ofchronic gout. It inhibits xanthine oxidase, an enzyme that converts hypoxanthine to xanthine, and xanthine to uric acid. May cause GI disturbances. 3. Enhancing uric acid clearance: Uricosuric agents include probenecid (Benemid) and sulfinpyrazone (Anturane). These agents act primarily in the kidney (.yecifcally, the proxintal convoluted tubules) and inhibit the secretion of other weak acids (i.e.. penicillin), in addition to inhibiting the reabsorption of uric acid. Remember: Normally penicillins and cephalosporins have to be given in high and fre- quent doses due to their high rate of elimination by the kidneys. Their excretion is slowed by giving probenecid.

Which of the following class of drugs lower blood cholesterol levels by inhibiting a key enzyme in the cholesterol synthesis pathway in the liver HPG-CoB Reductase Inhibitors HCG-CoG Reductase Inhibitors HMG-CoA Reductase Inhibitors HCG-CoB Reductase Inhibitors

HMG-CoA Reductase Inhibitors Coronart artery disease fClD/ is a condition ofnarrol'ing ofthe blood vcsscls oflhc hcarl rcstncting oxygcn flow ro heart musclc. lt has been cor.clatcd with the lcvcls ofblood cholcstcrol and triglyccridcs. Ifnot trcatcd, CAD can lcad ro mloca.dial infarction mlrrjrizing the threat ofCAD. (heart otkrck).D gs such asthc statins, \\'hich lowerblood cholcstcrol arc cficctivc in Currcnrl]. six classcs ofantihyperlipidemic drugs arc availablc. and cach class has its own mcchanism tor lot\'cring l::rd 1c\ cls r-ote: Thctwo major lipoprotcins that arc targctcd arcVLDLSand LDLs. Thc primary lipids ofVLDLS ;J. rhc Ingl!ccridcs. \r'hcrcas thc primary lipids ofLDLS arc cholcstcryl cslcrs. Highcr levcls ofLDL incrcasc thc risk rf..rrdio\ascular drscasc, whercas highcr lcvcls ofVLDL incrcasc the risk ofpancrcalitis. ' H\lG-CoA Reductase inhibitors: thc cnzymc 3-hydroxy-3-mcthylglutaryl cocnzymc A rri. is n..ccssary in thc kcy stcp to synthcsizc cholcsterol. When thc lrrol rs nor produccd in thc livcr and blood lcvcls dccrcasc. Thc family of "statin ,l,frforl. simr'rstatin r.statin /ar.,r/o,"/. . "statin" (H]tlc-Co,l) drugs inhibit this cnzymc, cholcs- rcdnc- drugs" includc atorvasaatin (ZoLor). flu\sstatin (a€r.or, lovastatin (Mewcor) pr^v^st^tin (Prarachol) Fibric acid derivatites: thcirprimary lipoprotein effect is to dccrcase trigiyccridc andraisc HDL concentrations. Tlc] do fiis by incrcasing lipoprotein lipasc activity, which rcsults in i crcascd catabolism ofVLDL. Examplcs in c:udc . gem fi brozi I /lop id) and fenofrbr^te (Tricor) . Bile acid sequestrants: thesedrugs bind tobilc acids, bilc acid scqucstrants increa-sc thc divcrsion ofcholcstcrol ro bilc acid slnthcsis, lowcr intraccllular storcs ofcholesterol, and res lt in crcascd caiabolism ofLDLby the lr\.r E\amplcs include cholestymmine . Other agents: nicotinic acid lrtdcin) fQuc"stdn), and colesevel^m (WelchoU. ;s bclicvcd to act on a hormonc-scnsitivc lipasc; this lcads to inhibition of rcleasc offrcc fatty acids from adiposc tissuc /@ob,srt. Thc inhibition oflipolysis lcads to reduccd frcc fatly acid rranspon ro thc liver and thcrcforc dccrcascd syrlhcsis ofVLDL. . Combination products: examplcs includc lovast^tinlni^cin (Adicor) . Selectivc cholestcrol absorption inhibitors: ezetimibe /Zsltdl ^nd and rostt- ezetimibe/simvastatin (l/ttoritt). is thc firsl agcnt in a ncw class ofdrugs that appcar to act on thc brush border of intcstinal cpi(hclial cclls. whcrc it sclcctivcly inhibits $c absorption ofcholcs' terol from dictary and biliary sourccs. Important: The "statin" dnrgs have thc capability to increase the breakdo*'n ofskeletal muscle thereby re- leasing muscle protein. Ifthc protein overloads the kidneys, renal failure could result. The ert'thromycin drugs enhance the capabilities ofthc "statins" to cause this effect.It is advisable for patients medicatedwith a "statin" druts nor to be gi\en eryhromycin nroducts

All are symptoms of acute aspirin toxicity EXCEPT Tinnitus (ringing in the ears) Vertigo (dizziness) Nausea Hallucinations Respiratory alkalosis Vomiting

Hallucinations An overdosage of saficyl^tes (aaie aspirin /o-riciry/ is life threatening and requires intensive supportivc trcatment in a hospital. Initial symptoms include respiratory alkalosis with hypcrpnea and tachypnea,nausea, vomiting, hlpokalemia, tinnitus, headache, dizziness, confusioq dehydration, hyperthermia, hy- pcractivity, and hcmatologic abnormalities, progressing to coma and respiratory collapse. Chronic aspirin toxicity presents with the following signs and symptoms: salicylism, CNS effects, bleeding and GI disnrrbances. Aspirin inactivates the enzyme known as cyclooxygenase. Sincc cyclooxygenaso synthesizes thc prostaglandins, the inhibition ofthis enz''rne results in the inhibition ofprostaglandin synthesis. As a result, aspirin is analgesic, antipyretic (ever reducing), and anti-inflammatory. Aspirin is an irre versible plat€let inhibitor and can reduce blood clotting to prolong bleeding. Note: Aspirin inhibits both COX-l and COX-2. Low doses of aspirin taken regularly can have a cardioprotective effect. These doses reduce throm- boxane production in platelets to result in the inhibition ofplatelet aggregttion. In this way, aspirin has the ability to inhibit the formation of life-threatening thrombi (6/0011clo/s/. The lirer appearc to be the principal site for salicylate metabolism, although other tissues may also be in\ ol\ ed. Ihe three chicf metabolic products ofsalicylic acid are salicyluric acid, the cthcr or phcnolic glucuronide and the ester or acyl glucuronid€. Excretion ofsalicylates occurs principally via the kid- n€), through a combination ofglomerular filtration and tubular cxcrction, in the form offree salicylic acid. salic] luric acid, as well as phenolic and acyl glucuronidcs. Contraindications to the use ofaspirin: . Ble€ding disorders (aspirin will increase bleeding tine) ' i,fluenzu or chiclerpoxl with or without fever due ro a potcniial association with Reye's syndrome Do not use in children with viral infections (i.e., (this s), dlotne is a serious neutulogical defecl) . ' . . Pregnancy P€ptic ulcers (especictlly faipil'i during the thit'd lrinester) nay cduse bleeding ofthe GI tracl) .{sthma; rhinitis; nasal polyps Concomitant use of anticoagulants

All are used in the treatment of parkinsons EXCEPT Levodopa (Dopar) Bromocriptine (Parlodel) Pergolide (Permax) Haloperidol (Haldol) Selegiline (Eldepryl) Amantadine (Synmetrel)

Halopeidol (Haldol) Parkinson's disease is a slowly progressing, dcgenerative disorder of the nervous system. It has ie\craf distinguishing chamcteristics: tremor (shaking) when at rest, sluggish initiation ofmove_ mcnls and musclc rigidity. ln Parkinson,s disease, nerve cclls in the basal ganglia degcneratc! re_ ofdopamine, ll may be tr€ated fbr ot cured)with a wide varietv of J* g. sultine in lo* er production . Le\ odopa /in couhi ation tith carbidopcJ is the precursor of dopanine. lt is the main treat_ ment 1br Parkinson's discase. It is given with carbidopa to incrcase effectrvcness and rcduce ride . ellects. Bromocriptine or pergolide are dopamine agonists which are often given in addition to lev_ odopa earlv in the treatmcnt to enhance levodopa's action, or may be given laterwhen levodopa,s iid. eft'ects become more ofa problcm. ' selegiline is a selectivc inhibitor ofMAo rype B. the enzyme that is rcsponsible for the ox- rdatir c deamination ofdopamine in the brain. It is used as an adjunct to levodopa. '-\mantadine appears to potentiate dopaminergic responscs. Antiparkinsonian actions are unrelated to thc antiviral cffects. lt is used in thc early stages for mild diseasc. . Haloperidol is used to treat psychotic Antichofinergic drtgs (benztropine and ttiheryphenidyl, certaitl .uttidepressa ts an.l anti_ hi<tunrines such as diphenhydranine) may be given without levodopa in thc early stages ofdis_ ease. $ilh lcvodopa in later stages. \otrs 1. Dental implications ofAnti-parkinson's . agents; Anti-Parkinson's agents (mai y levodopa, the direct dopamine dgonists and the COMT inhibitors) have been associated wilh orthostatic hypotensior. . . . Xerostomia lcarsed by a ticholinetgics antl MAO_B inhibitorst. Schedule appointmcnt times based on when the patient is feeling the bcst. Dyskinesia (abnonnul muscle movenents) caused by some ofthe drugs (nainly let,_ odopa, the direct dopamine agonist.s, and the COMT ir*ibito,"t can prescnt a chal_ lenge whcn trying to perform syndromes. dental treatmcnt.

All of the following are phenothiazines EXCEPT Chlorpromazine (Thorazine) Haloperidol (Haldol) Prochlorperazine (Compazine) Thioridazine Perphenazine Trifluoperazine (Stelazine)

Haloperidol (Haldol) {ntipsychotic agents include: . . First generation (lr^pical) agentsl I . Phenothiazines: . Chlorpromazine . Perphenazine . Thioridazine . . Prochlorperazine T rifrtroperczrne L But'"rophenones: . (Thorazine) (S (Compazine) t e I azin e) Haloperidol is a highly effective antipsychotic drug used to treat schizophrenia. ln ad- dition it has been found to be effective in the heatment ofTourette's s).ndrome. i. Thioxanthenes: . Thiothixene is a less potent antipsychotic. It is used for the treatment ofschizophrenia. Second generation (a\)pical) ag€nts: this group includes clozapine, risperidone, olanzap- ine. quetiapine, ziprasidone, and aripiprazole. These drugs are effective in treating schizo- phrenia and exhibit reduced ability or an inability to induce EPS. Adyerse effects of antipsychotic drugs: . . Dystonia and akathisia (an unpleasant sensation of motot' restlessness) syndrome and ultimately increase the risk of fatal arhythmias . . . . . Long QT-interval Neuroleptic malignant syndrome Parkinsonism Antimuscarinic effects Orthostatic hypotension Con\.ulsions . . Mesoridazine Fluphenazine

Which is a high molecular weight heteropolysacharide that inactivates thrombin and other coagulation and thus prevents blood clotting Prothrombin Fibrin Heparin Plasmin

Heparin Heparin is contained within mast cells and basophils. These cells occur tn connective tissue and in extracellular spaces near blood vessels. Remember: Heparin not only neutralizes tissue thromboplastin, but also blocks thromboplastin generation. 1. The administration ofheparin will result in an increase in bleeding time due \ot6 ro a potentiation of antithrombin III thereby inactivating thrombin. This pre- - \'ents the conversion offibrinogen to fibrin. ?. It is used for prophyla-ris and treatment of thromboembolic disorders.

Which 2 drugs below inhibit blood clotting by affecting the coagulation pathway to prevent fibrin formation Heparin Vitamin K Aspirin Cloprdogrel (Plavix) Warfarin (Coumadin)

Heparin Warfarin (Coumadin) Heparin inactivates thombin and prevents the conversion of fibrinogen to f:Jc.rin (blood clot);w^rfarin (Coumadin) rnterferes with the hepatic synthesis of vitamin-K dependent coagulation factors lI, VII, IX and X resulting in the inability ofthe coagulation pathway to form ftbrin (blood clot). Vitamin K is a group of fat soluble vitamins that are essential for the synthesis of coagdation factors II, VII, IX and X, and prothrombin in the liver. Vitamin K will enhance blood clotting rather than inhibit blood clotting. Aspirin inhibits blood clotting by inhibiting platelet aggregation in an irreversible manner. Inhibition ofplatelet aggregation prevents activation ofthe coagulation pathway; thus no fibrin (c/ot) is formed. Aspirin does not affect the coagulation pathway. Discontinuation ofaspirin for 5 to 7 days allows for normal clotting time to reappear due to the svnthesis of new platelets. Clopidogrel /P/avx) inhibits blood clotting by inhibiting platelet aggregation in an ir- reversible manner. Thus the effects on blood clotting are the same as aspirin. Clopidogrel rPlatix) does not cause gastric ulcers like aspirin does and is the antiplatelet agent of choice in patients with history ofulcers. \ote: Cfopidogrel, aspirin, abciximab (ReoPro), and anticogulants (i.e., heparin) are used to lessen the chance of heart attack in people who need percutaneous coronary in- ten ention fPCl), a procedure to open blocked arteries ofthe heart.

Which is a contraindiction or precution for the use of prilocaine Biliary tract disease Type II diabetes Rleumatoid arthritis Hepatic disease

Hepatic disease Remember: All amides are rnetabolized primarily in the liver, and the metabolites are then renally excreted. Priloc^ite (Citanesrl is a local anesthetic ofthe amide class, used for nerve block, epidural and regional anesthesia. It has an intermediate duration ofaction and is longer acting than Lidocaine. Prilocaine produces less vasodilation than do equal amounts ofLidocaine is somev'hat less potent thqn Lidocaire). It is available as a 4olo solution with or without epinephrine, which prolongs the anesihetic effect. Prilocaine is metabolized to orthotoluidine, a product that can produce m€themoglo- binemia, a condition that is characterized by increased levels of methemoglobin in the blood. Note: This methemoglobin is less effective than hemoglobin in catrying oxygen in the blood. Prilocaine is about one-half as toxic as Lidocaine, but since methemoglobinemia is a possible reaction, Prilocaine is not used for patients with hypoxic conditions ofany kind. \ote: EMLA Cream is a eutectic mixture oflidocaine 2.5% and prilocaine 2.5% formu- lated as an oil in water emulsion. In this eutectic mixture, both anesthetics are liquid at room temperature and the penetration and subsequent systemic absorption ofboth prilo- caine and lidocaine are enhanced over that which would be seen if each component in cn stalline form was applied separately as a 2.5olo topical cream.

The antiviral agent Penciclovir (Denavir) is active against which virus Herpes zoster Genital herpes Herpes simplex type 1 (HSV-|) Papilloma virus

Herpes simplex type 1 (HSV-|) Penciclovir (Denavi is a cream formulation indicated for the treatment of recurrent herpes labialis (cold sores) in adults. This condition is caused by the herpes simplex type 1 virus. Penciclovir is not available for systemic dosing. Penciclovir (Denavir) inhibits viral action by selectively inhibiting herpes viral DNA synthesis and therefore resulting in the inhibition ofviral replication, Other agents indicated for use in treating the condition ofherpes labialis are: . . . . . Acyclovir tablets Acyclovir cream Docosanol cream (lbreva) Lysine tablets Valacyclovir caplets (Valtrex)

A drug with a high LD50 and low ED50 has a High therapeutic index and is, therefore, very dangerous High therapeutic index and is, therefore, relatively safe Low therapeutic index and is, therefore, very dangerous Low therapeutic index and is, therefore, relatively safe

High therapeutic index and is, therefore, relatively safe Explanation: The purpose ofan acute toxicity test is to determine the nature and extent ofthe untoward reactions which might iollow the administration ofa single dose (or an overdose) of a dntg. A quantitative aspect ofacute toxicity testing is the determination of the drug's lethal dose using mice. This is usually expressed as the LD59. Standing alone, it conveys less information than does the ratio of the lethal to the effective doses (LD5/ED50), a quantity which is often known as the therapeutic index drug safety).The greater a drug's therapeutic index, the less lik€ly that fatalities will fol- low an accidental overdose. ..' --.,. 1. ED5g is the effective dose at which 50o% ofpeople will respond. .:Noted.' 2. LD56 is the lethal dose at which, in theory 50% ofpeople will die. .;g..4 *** Lethal doses are always determined in mice, not people 3. The concept oftherapeutic window is not the same as therapeutic index. The therapeutic window describes the range between the lowest therapeutic concentration and the beginning of toxicity. Some drugs have a very nanow therapeutic window, making patient monitoring especially important. 4. Toxicity results when the dose ofthe drug is excessive for the particular pa- tient. 5. Side effect: an adverse effect that occurs within the therapeutic dose range ofthe drug. 6. Idiosyncratic reaction: a reaction to a medication that is unusual and un- predictable, specific to a particular person. Unlike allergy. it can occur on first exposure to the medication, unlike a side effect, it affects only very few individuals.

All of the following drugs are bronchodiltors EXCEPT Epinephrine Levalbuterol (Xopenex) Albuterol (ProventiI) Salmeterol (Serevent) Metaproterenol (Alupent) Histamine Aminophylline

Histamine Asthma is a respiratory disorder characterized by recurring episodes of paroxysmal dyspnea, wheezing on expiration, coughing, and viscous mucoid bronchial secrelions. The episodes may be precipitated by inhalation ofallergens or pollutants, infection, vig- orous exercise, or emotional stress. Management and prevention ofan acute asthma attack: . Epinephrine, levalbuterol, albuterol, salmeterol and metaproterenol are beta. adren- ergic receptor agonists. Thus they stimulate the beta recepton in the airway to cause bronchodilation. They are taken via aerosol inhalers and nebulizers. Important: Albuterol is the drug of choice as a "tescue" agent during an acute asth- matic attack. Note: Arninophylline is an example of a theophylline compound. Theophylline com- pounds are administered orally as bronchodilators in reversible airway obstruction due to asthma or COPD (chronic obstructive pulmonary tlisea.re). These drugs relax bronchial smooth muscle to improve airway function.

Diphenhydramine HCL (Benadryl), chlorpheniramine maleate (chlor-trimeton), loratadine (claritin), and desloratadine (clarinex) are antihistamines at which histamine site listed below Hl-receptor site H2-receptor site

Hl-receptor site Antihistamines are antagonizing agents that compete for receptor sites with natural his- tamine. The two types of histamine antagonists are: l. H1- agonists: competitively block H1-receptors blocking the effects of histamine at these receptors. They block the vasodilation, they block the constriction ofthe bronchi and they block capillary permeability which histamine ordinarily causes. The blockade ofthese effects of histamine overcomes the svmDtoms of seasonal aller- gies. . Actions of Hl antihistamines: - Blocks pain and itch, vasodilation and bronchoconstriction due to histamine - Reduces motion sickress: first-generation only anrl c h I orpheniramine maleate) (i.e., Diphenhydramine HCL - Promotes sleep: first-generation only Remember: The second generation Ht- agonists have the following charac- teristics: Longer halfJives than first generation (l2-24 first hours as opposed to 3-6 hours generation), they do not cross the blood-brain barrieq they produce little or no seda- tion. and have a higher risk of cardiac arrhythmias (long QT Cetirizine HCL (Zyrtec), fexofenadine HCL ellbct). Examples include (AIIegra),loratadine (Cla tin) and desloraradine (Clarinex). :. H2-r€ceptor antagonists: competitively block H2-receptors, blocking the effects of histamine at these receptors. Examples ofthese drugs include: cimetidine (Tagamet), n tidine (Zontac), famottdine (Pepckl) and nrzatrdine (Axrd). They block secretion of stomach acid and are used in the treatment ofduodenal ulcers. \ote: Histamine is found in all tissues, particularly in mast cells and blood basophils. It is released in allergic and inflammatory reactions.

All are ACE inhibitors EXCEPT Captopril (Capoten) Hydralazine (Apresoline) Enalapril (Vasotec) Lisinopil (Zestril) Fosinopril (Monopril)

Hydralazine (Apresoline) Hydralazine is a direct peripheral vasodilator. ACE inhibitors interfere with the conv€rsion ofangiotensin I (d veak rasoconstictol) to angiotensin ll (a highly e.f;[ective constrictor). Thcy do this by being inhibitors of angiotensin-convcrting enzyme (,1CEl. These drugs are used to treat hypertension and congestive heart failure. Adverse cffects includc cough, hypotension, neuhopcnia, anorexia alrd pol]uria. Note: They can alter the sense oftaste and in a few rare cases, cause angioneurotic edema. Angiotensin II is a potent vasoconstrictor and is a stimulus for aldosterone release from the adrenal glands. Reduction in aldosterone secretion results in less waterabsorption and sodium/potassium ex- change in the distal renal tubule, causing a slight increase in serum potassium. ACE inhibitors inhibit the breakdown ofbradykinin, a potent and naturally occurring vasodilator, by blocking thc cnzymc kin- inase IL This though, is thought to be the causc ofthe cough commonly experienced by patients who take this class ofdrugs. OtherACE inhibitors . Benaz,eptll f,4Ct1t (Lotensir./ . include: Moexipril fLr?i|ascl 'Quinapril (Accupril) (Capoten) . R^rnipfll (Altdce) . Angiotensin lI reccptor blockerc . . . *** Losarlan (Coaaar) \'alsafian (Diotan) C andesanan (Atacand) Trandolaprll f,4RBs/ . . (Mat include: ik) ' Lisinopril (Zestil) Eprosaftan (Tevete Olmesaftan (Benicar) . Tebllisafian (Mica rd i s) ) 'Captopril ' Elan .lrbesarlan (Avapro) ARBs block the effects ofangiotensin II by blocking the binding ofangiotensin II to its rcccptors. Thc: do not effect bradykinin. Adverse effects jnclude dizziness, diarrhea and myalgia. Thc antih) pcnensive effects ofARBs have been provcn comparable with those ofthe ACEIs ACEIs and {RBs are ofparticular values for the reatment ofhypettensive patients who have concomitant illnesses iuch as diabetcs, rcnal insufficicncy, left ventricular dysfunction, and CHF \ote: ACE inhibitors and Angiotensin II receptor blockers indirectly inhibit fluid volume increases \\hen interfering with angiotcnsin II because angiotcnsin II stimulates thc r€lease ofaldosterone, which promotes sodium and water retention.

A fear reaction activates the sympathetic division of the autonomic nervous system to result in: Increased salivation Miosis Bradycardia Hypertension

Hypertension Activation ofthe sympathetic portion ofthe autonomic nervous system will cause alphar- adrenergic receptor activation to result in arteriolar vasoconstriction with an associated elevation of blood pressure leading to hypertension. . l. Sympathetic activation ofthe eye would result in mydri^sis (dilstion) not \ot!r miosis. -..--, \liosis 2. Sympathetic activation ofthe heart would result in tachycardia, not brady- cardia. 3. Sympathetic activation ofthe salivary glands would result in a thick ropey- type salivary flow, not increased salivation. (pupillaT, constriction), bradycardia, and increased salivation are physiological effects all resulting from activation of the parasympathetic division of the aulonomic nen'ous system.

All of the following conditions are managed by using anticoagutants and anti-platelet agents EXCEPT Coronary Artery Disease (CAD) Angina Pectoris (Unstable Angina) Myocardial Infarction (Heart Attack) Stroke Hypertension

Hypertension Anticoagulants such as warfarin (Coamadin) and anti-platelet agents such as aspirin and clopidogrel (Plavix) are nsed in the conditions listed for the following reasons: . Coronary artery disease (ClD): will help prevent threat ofmyocardial infarction in CAD patients. . Angina pectoris (anstable angina): will help prevent thrombus from forming within the coronary arteries. . Myocardial infarction (MI): drugs that prevent blood clotting have been shown to prevent the threat of future infarcts. . Stroke: will help prevent thrombus from forming thus preventing threat ofa cerebral embolism. Note: Unless there are other accomparying cardiovascular problems such as those listed above. anticoagulant drugs are not necessary in the treatment and management of hy- penension. These drugs do nothing to lower blood pressure.

Which agent has antiinflammatory properties Codeine Acetaminophen Hydrocodone Ibuprofen

Ibuprofen Ibuprofen inhibits the production of prostaglandins in peripheral tissues at sites where pain and inflammation are present. Inhibition of prostaglandin production reduces the inflammatory respons€ at sites of surgery, injury or infection. Reduction of inflammation results in reduction ofperceived pain. Acetaminophen is a weak inhibitor ofprostaglandin production in peripheral tissues. Thus, the inflammatory response is not affected to any great degree. Acetaminophen re- duces pain through mechanisms other than inflammatory reduction. lt is unclear exactly how acetaminophen uorks to reduce pain. Codeine and h1-drocodone are narcotic analgesics that effectively reduce pain but do not reduce inflammation. Narcotics work within the brain to block ascending pain im- pulses traveling from the periphery into the brain.

Your patient has a history of drug abuse. Which agent could be given (if needed in the treatment plan) with no liability to cause an addiction Phenobarbital Ibuprofen Hydrocodone Meperidine Codeine

Ibuprofen Ibuprofen is an NSAID and is classified as a non-narcotic analgesic. Non-narcotic analgesics have no liability for abuse or addiction. They are not a controlled substances. Phenobarbital is a barbiturate used as a sedative and to treat epilepsy. Barbiturates all have the potential to cause abuse and addiction, and are controlled substances requiring a DEA number fiom the prescriber. Hydrocodone (Wcodin), meperidine (Demerol) and codeine are narcotic analgesics with the potential to cause abuse and addiction. They are all controlled substances re- quiring a DEA number from the prescriber Important: NSAIDs can inhibit the antihypertensive effect ofACE inhibitors, beta- blockers. and diuretics

The following drugs belong to what drug class -Pimecrolimus -Sirolimus -Tacrolimus Irnmune modulators Monoclonal antibodies Immunosuppressants Interferons Colony stimulating factors

Immunosuppressants Pimecrolimus (Elidel) is an immunosuppressant agent used for treatment of mild to moderale atopic dermatitis. Sirolimus (Rapamune) is an immunosuppressant agent used for prophylaxis of organ rejection in patients receiving renal transplants. Tacrolimus (Protopic) is an immunosuppressant agent used to treat moderate to severe atopic dermatitis in patients not responsive to conventional therapy.

Your patients medical history includes modafinil (provigil). Your patient is probably using this drug in order to Improve wakefulness during daytime sleepiness Improve salivary flow in dry mouth disorders Manage psychotic disorder Treat mental depression

Improve wakefulness during daytime sleepiness Modafinil (Provigil) represents a class of central nervous system stimulants used to improve wakefuhess in patients with excessive daytime sleepiness associated with nar- colepsy and shift work sleep disorder. It also has an unlabeled use to treat attention defi cit/hyperactivity disofier (ADHD). The exact mechanism of modafinil is unclear; however part of its action may be do to decreased GABA-mediated neurotransmission.

All re known to cause orthostatic hypotension EXCEPT Levodopa (Larodopa, Dopar) Prazosin (Minipress) Indomethacin (Indocin) Morphine

Indomethacin (Indocin) ndomethacin (ndoclry' is an NSAID which may cause GI bleeding, ulcers and pos- sible stomach perforation. Drugs that may produce orthostatic hypotension: . . -{ntih} pertensiyes: for example prazosm (Minipress) Phenothiazines: for exanple chlorpromazine and thiondazine (Mellaril) . Triclclic antidepressants: for example doxepin (Sinequan), amitriptyline (E/avll) and imipramine I Tolianil) . \arcotics: for example meperidine (Demerol) and rnorphine . -Lntiparkinson drugs: for example levodopa (Larodopo. Dopur) and carbidopa ler odopa /Slnenet) Orthostatic h)'potension (also called postural hypotension) is abnormally lorv blood pressure occuning when an individual assumes the standing posture. Important: Following vasovagal syncope, orthostatic hypotension is the most likely cause of transient unconsciousness in the dental office. Many factors have been identi- fied that rnay be responsible for the development of orthostatic hypotension, including several which are important to the practice of dentistry. They include the administration and ingestion ofdrugs, prolonged recumbency and convalescence, an inadequate postural reflex, pregnancy, various defects in the legs, Addison's disease, physical exhaustion and starvation and chronic orthostatic hypotension creases with age.

Which is the most frequently utilized route of administration for sedation in pediatric patients Oral Inhalation IV IM

Inhalation ** The agent used most frequently is nitrous oxide. Nitrous oxide is a slight sweet smelling, colorless, inert gas. It must always be coupled with no less than 207o oxygen, Nitrous oxide is quickly absorbed from the lungs and is physically dissolved in the blood. There is no biotransformation, and the gas is rapidly excreted by the lungs when the concentration gradient is reversed. It is recommended that the patient be maintained on oxygen for 5 to l0 minutes after the sedation period. Dose response for nitrous oxide (always given in mixture with oxygen): . . l0o/o - 20%o: tingling ofhands, feet, body warmth 20o/o - 40oh: mild sleepiness, relaxation, some analgesia, mind dissociation, height- ened auditory perception . *** *** Above 507o: this is too much nitrous Rapid onset: 5 minutes Rapid recovery: 5 minutes -nausea. sweatinq \ote: Nitrous oxide does not provide enough analgesic effect to preclude the use oflocal anesthesia in dental surgery or restorative procedures. Local anesthetics must be used along rvith nitrous oxide. Remember: Nitrous oxide has minimal depressant effects on the cardiovascular system and no skeletal muscle relaxant properties.

All of the following are considered to be what type of anesthetic Halothane Enflurane Isoflumne Sevoflurane Desflurane

Inhalation anesthetics Inhalation ancsthctics arc drugs which are vaporized tiom thc liquid form and inhaled to produce geneml anesthesia. Thcse diveisc dnrgs are relatively simplc lipophilic molccules ranging from cthcrs such as halogenated hydrocarbons such as halothane and halogenatcd ethcrs such as isofluranc, cnflumne, scvoflumne, and desflumnc. Note: Vaporization ofthcsc liquids occus in a vaporizer and dclivery to the patient occurs through an anestbcsia ma- chine via a nasal mask. Essential Components of Anesthesia: . Analgesia - perception ofpain eliminated . . . Htrpnosis - unconsciousness Depression ofspinal motor r€flexes Musclc rclation Hypotheses of General Anesthesia: . Lipid Theory: bascd on the fact that anesthetic action is corrclatcd with the oil/gas coefficicnts. The higher the solubility ofancsthctics is in oil, thc grcater is thc anesthctic potency. . is conelated with thc ability of ancsthctics to inhibit enzynes activity of a Protein (Receptor) Theoryi based on the fact that ancsthctic potcncy protein. Also, attempts to cxplain thc GABAA rcceptor is a potcntial targct of ancsthctics acton. . pure, soluble Binding theory: anesthetics bind to hydrophobic portion Phermacokinetics of lnhsled Anesthetics: . . Amount that rarches the brain - Indicatcd by oil:gas ratio (lipid soluhili\,) Partial Pressure of anesthetics - 5% ancsthctics = 38 mmHg . Sofubility of gas into blood (blood-g.ts solubiliry coelficient) of the ion channel. - Thc lower the blood:gas ratio, the morc anesthctics will arrivc at thc brain . Cardiac Output - Incrcased CO = greater induction time \lAC hean alveolar concentration). . . A measurc ofpotency I MAC is the concentration necessary to prcvent responding in 50o% ofpopulation . Valucs ofMAC are additive: - Avoid cardiovascular depressive concenration ofpotcnt agcnts

Clindmycin has which mode of action on a bacterial cell Affects cell membrane Interferes with protein synthesis Affects cell wall Interferes with normal biosynthetic pathways

Interferes with protein synthesis Mechanisms of action ofantibiotics on the bacteria cell: . . . Agents affecting cell wall: - Penicillins - Vancomycin - Lnipenem - Cephalosporins - Cycloserine - Bacitracin - Aztreonam Agents interfering with protein synthesis: - Tetracycline - Erythromycin - Azithromycin - Aminoglycosides - Lincomycin - Clarithromycin - Ch loramphenicol - Clindamycin Agents interfering with normal biosynthetic pathways: - Sulfonamides - Trimethoprim - Fluoroquinolones (i.e., ciproJloxacin, norJloxacin, Ievo;floxacin, moxiJloxac'in, and gentilloxacin) \ote: The most corruron clinical cause of bacterial resistanc€ is the use of antibiotics * hen they are not indicated.

These drugs act to induce gene transcription, inhibit cellular growth and alter the state of cell differentiation Immune globulins Immunosuppressants Keratinocyte growth factor Interferons

Interferons Interferons are used for a variety ofconditions including: hairy cell leukemia (intederon alpha-2a) chronic hepatitis B (interferon alpha-2b), recurring genital warts (interferon alpha-n3) and treatment of multiple s clerosis (interferon beta- I a) . Immune globulins provide passive immunity by increasing antibody titers. Immunosuppressants are drugs such as cyclosporin which prevent organ transplant re- jection. Growth factors such as keratinocle growth factor promote cell proliferation and angio- genesis

Which will give the most rapid onset of pharmacological effect Oral administration Subcutaneous injection Intravenous injection Intramuscular injection

Intravenous injection trrhen a drug is given intravenously, it is placed directly into the systemic circulation. The drug is delivered rapidly to all tissues, including the drug receptor sites. For all other routes of drug adminrstralion (v)ith the exception of intra-arterial inj ection), the drug must be systemically absorbed prior to distribution to the drug receptor sites, and therefore the onset of pharmacological effects is slower. .A.lso. rvhen a drug is given by IV injection there is a complete (100o/') bio^v^ilabilif. The entire dose is placed into the systemic circulation. With other routes of administration. the drug may be lost prior to reaching the systemic circulation. For example, with first-pass effects, a portion ofar orally administered drug is eliminated, usually through degradation by liver enzymes, before the drug reaches its receptor sites. Remember: The initial distribution ofa drug into the tissues is determined chiefly by the rate of blood flow to the tissue, whereas drug affinity for the tissue will determine s hether the drug will concentrate at that site. l\,lote: Gastric emptying time and degree of plasma protein binding also have an effect on drug distribution but are less important than the rate ofblood flow to the tissues.

AII of the following drugs are used to prevent or to provide relief of angina pectoris EXCEPT Nitroglycerin (Nitrostat) Isoflurophate Nifedipine (Procardia) Diltiazem (Cardizem) Propranolol (Inderal) lsosorbtde (Isordil)

Isoflurophate Isoflurophate is an organophosphate cholinesterase inhibitor used in the treatment ofglaucoma Angina pectoris is the pain in the heart and chest which occurs during the occlusion ofcoronary arteries. Triggers that can cause occlusion are physical exertion, increased blood pressure, and vasoconstriction. Antianginal drugs work by reducing cardiac rate and force, reducing peripheral vascular resistance, or dilating coronary blood vessels. Nitroglycerin is a coronary artery vasodilator. It relaxes blood vessels to provide in- creased blood flow and oxygenation to the heart muscle. It is sublingually effective within 2-4 minutes. The nitroglycerin skin patch releases the drug over a 12 hour period to pro- \ ide sustained blood levels for prevention ofangina. The two most common adverse ef- fects caused by nitroglycerin are orthostatic hypotension and headache. \ifedipine lProca rdia) and diltia,zem (Carulizem) are calcium channel blockers used to prevent angina attacks. These drugs are used to dilate coronary blood vessels for im- prored blood flow to heart muscle. Note: Calcium channel blockers as a class have been associated with causing gingival hyperplasia. Propranolol (lnderal) ts representative ofthe beta-blockers used to prevent angina attacks. Atenolol (Tenormin) is another popular beta-blocker used for this purpose. Betablockers are used to decrease the work load ofthe heart such that less oxygen is required.

All of ihe following statements concerning edrophonium are true EXCEPT one. It is a direct acting cholinergic agonist (cholinomimetic) It is a rapid-acting, short-duration, injectable cholinesterase inhibitor . It is the drug of choice for diagnosing myasthenia gravis because of its rapid onset of action and reversibility It is also useful in differentiating a myasthenic crisis from a cholinergic crisis

It is a direct acting cholinergic agonist (cholinomimetic) This is falsei edrophonium is an indirect-acting cholinergic agonist (cholittomimetic) as are pyridostigmine, physostigmine, and neostigmine (thq, are \ote: These drugs are indirect agonists at both muscarinic and nicotinic sites. qll (Evoxac). cholinesterase inhibitors). Pyridostigmine is prescribed in the treatment ofmyasthenia gravis, however edrophonium is used to diagnose myasthenia gravis and not in the treatment ofit due to its Yery short du- ration of action, Symptoms ofa cholinergic crisis include bradycardia (decreased heart rate),lacrimation, extreme salivation, vasodilation and muscle weakness. Because a cholinergic crisis can result in muscle weakness like that ofa myasthenic crisis, distinguishing the two conditions may be dif- ficult. Administration ofa short-acting cholinomimetic such as edrophonium will improve a ml asrhenic crisis but wors€n a cholinergic crisis. Remember: Typical cholinergic effects caused by stimulation of acetylcholine receptors kholi ergic receptorg include salivation, miosis, excessive sweating, flushing, increased GI motility and bradycardia.

Which two gastrointestinal drugs listed below reduce the formation of stomach acid by inhibiting the proton pump of the stomach parietal cells? Ranitrdine (Zantac) Omeprazole (Prilosec) Cimetidine (Tagamet) Famottdine (Pepcid) Lansoprazole (Prevacid)

Lansoprazole (Prevacid) Omeprazole (Prilosec) Hydrochloric acid (l{Cl) is produced by the parietal cells ofthe stomach through a pump u ithin each cell which pumps protons (H.) into the stomach contents. The pump is called tbe H-/K' ATPase pump. HCI is used for food digestion but an abundancy can cause heart bum and acid indigestion. Omeprazole and lansoprazole inhibit the pump such that no protons are pumped into the stomach contents and thus no HCI is produced. These nvo drugs are classified as proton-pump inhibitors. Stomach acid can also be reduced by inhibiting the effects of histamine in the stomach at the histamine type-2 receptors (H2 receptors). Ordinarily, histamine stimulates the gastric parietal cells to produce hydrochloric acid. Ranitidine, cimetidine and famotidine block the effects of histamine by blocking at the H2-receptors. These tkee drugs are classified as H2-receptor blockers. Both the proton-pump inhibitors and H2-blockers are used to feat heartbum, indigestion, sour stomach, active duodenal ulcer disease and gastroesophageal reflrx disease (GERD). Remember: Antacids neutralize excess stomach acid by a chemical reaction. Antacids include aluminum hydroxide (Amphogel), bismuth subsalicylate (Pepto-Bismol), calcium carbonate (Tums) magnesium hydroxide (Maalox) and sodium bicarbonate.

Which artiarrhlthmic agent is effectlve only on the ventricle and is often administered IV to treat life threatening ventricular arrhythmias? Quinidine (Quinidex) Lidocaine (Xylocaine) Flecainide (Tambocor) Propafenone (RytmoI) None of the above

Lidocaine (Xylocaine) When lidocaine is used IV to treat ventricular arrhythmias, it acts on the fibrillating ven- tricles to decrease the cardiac excitability and spares the atria. It can effectively reverse a life-theatening situation. Quinidine is considered as the prototype antianh)'thmic agent and is used primarily to treat atrial fibrillation. It is not effective in treating life-threatening ventricular fibrilla- hon. Flecainide (Tambocor) is potent antiarrhythmic agent, effective in a wide range ofven- tricular and atrial arrhythmias and tachycardias. Propafenone (Rythnol) is used to treat both ventricular arrhlthmias and supraventricu- lar tachycardias.

Which drug is the current drug of cholce for the treatment of the manic phase of bipolar disorder (or manic depressive syndrome)? Phenobarbital Imipramine Lithium Haloperidol

Lithium Bipolar disorder for manic-clepressive syndrome) is characterized by cyclical changes in af- fectir e state between the manic and depressive phases ofbehavior Bipolar patients cycle ber$ een rhe trvo aflbcted states. Lithium salts are by far the most important drugs for suppressing mania in patients with affective disorders. It is not useful for the acute manic episodes. Lithium can prevent the oc- currence ofboth the depressive as well as the manic episodes in some but not all the patients. Antidepressants are often administered with lithium to manage the depressive phase ofthe ill- ness if lithium alone is not sufficient. Note: Lithium works inside the cell to prevent the formation of inositol triphosphate and diacylglycerol. These substances serve as second messengers within the CNS and may have widespread influence on neuronal function. Approximately 2570 of patients who suffer from mania do not respond to lithium. Carba- mazepine (qtl anticonvulsant) andvalproic teid (also an a ticonwlsant) may be effective in some relractory cases. Note: Carbamazepine blocks sodium channels and valproic acid blocks both sodium and calcium channels. Neurofeptic agents (also referrecl to as anti-psychotic agents or msjor trqnquilizersl are used in the acute manic episodes. Chlorpromazine, which is a phenothiazine, and Haloperidol, *'hich is not a phenothiazine but acts in a similar fashion, are effective in quelling the exheme mania and psychotic behavior . 1. The most common side effects of lithium, including Gl irritation, fine hand i\ote*. ftsm61 muscular weakness, polyuria, thint, sleepiness, and a sluggish feeling, are ;;.-1 often associated with initial therapy and usually fade within I to 2 weeks. 2. Severe intoxication results in vomiting, diarrhea, unconsciousness, and convul- sions. 3. Sodium restriction leads to greater retention oflithium in the kidney. 4. Diuretics and some NSAIDs reduce lithium excretion and mav cause lithium toxicity.

The most important enzyme systems for biotransformtion of drug molecules are found in Lungs Liver Brain Kidneys Gastrointestinal tract

Liver Hepatic metabolism of drugs occurs in Phase I reactions catalyzed by a microsomal mixed-function oxidase system (a/so known es the P-450 system) and in Phase II reac- tions known as conjugation reactions. Phase I reactions: . Occur in the liver microsomal enzyme system (mixed-lilnction oxidase system or P- 450 system).In this system, drug metabolism occurs in three basic pattems. First, the active parent drug can be converted to the inactive metabolite. Second, an active par- ent drug may be converted to a second active compound which is subsequently con- verted to an inactive compound; and thirdly an inactive parent drug may be transfonned to an active compound. . The most common r€action in drug metabolism is an oxidation reaction in which oxygen in the form ofhydroxyl group is attached to the drug molecule. . There are at least eight distinct groups of microsomal drug metabolizing enzymes: These enzymes are identified as a cytochrome (CYP preJix) followed by their numerical digestion "families" (e.g., lA2).Thusrhe enzyme CYP lA2 is a distinct drug me- tabolizing enzyme that converts a variety of drugs to the oxidized product. Phas€ II reactions: . Conjugation reactions involve coupling the drug with an acid present in cells (rrsa- alh glucuronic acid). \,,lhen coupled to glucuronic acid, the process is known as glu- curonide conjugation with the resulting metabolite refened to as the "glucuronide." . . Conjugations occur in the liver, kidney and to a lesser extent in other tissues. Conjugation ofdrugs results in polar, water-soluble compounds that are rapidly ex- creted in urine. Thus, the parent drug is effectively rendered inactive and transport€d out ofthe body by this process.

The amide local anesthetics are metabolized primarily in the Lungs Plasma Liver Kidney

Liver The amide-type local ancsthctics that are used in dentistry and metabolizcd by the liver includc: . Lrd,ocaine (Xylocairte) . Ptilocaine (Citane.tt) . A(icain (Ultrucaine) .Mepivacaine(Carbocoite)'Bup|acarne(Matcaine)'Etidocatnc(Dkranest) Thesc anesthetics are rnetabolized by the hepatic microsomal enzyme system. The products formcd do not have anesthetic actions and are excreted p marily in the urine as metabolites. These agents should be used wjth caution ornot at all in patients with compromiscd livcr function. Notei The most abundant urinary metabolite of lidocaine is 4-hydroxyxvlidine. Amides are metabolized by three types ofreactions: l. Dealkylation ofthe amino terminus 2. Hydrolysis ofthe amidc bond L H.vdroxylation ofthe aromatic ring The amides vary in protein binding. Lidocaine and mepivacaine are bound moderately. Eridocaine and bupivacaine are highly bound. Note: Bupivacaine is more selective for sensory nerves than etidocaine. \ote: Bupivacainc has the longest duration of action ofany dcntal local anesthetic prescntly available.

The barbiturates phenobarbital (luminal), mephobarbital (mebaral), and primidone (mysoline) are classified as which type of barbiturate Ultrashort-acting Short-acting Intermediate-acting Long-acting

Long-acting The length ofaction can be related to the lipid solubility with the ulhashort being the most lipid solu- bJe and the long acting having thc least lipid solubility. Thcy are mctabolized in the liver, These drugs possess serious drug dependence potential. They do not possess significant analgesic propefiies. The length ofh)?notic action ofthe four different classes after a single dose are as follows: . Ultrashort-acting: 5 to 20 minutes . . . Short-acting: I to 3 hours Intermediate-acting: 3 to 6 hours Long-acting: 6 to l0 hours Agents: . Ultrashort-acting: for induction of general anesthesia. Examples include thiopental (Pentothal) and methohexital fBr"evildl,. . Short-actingi for treating insomnia. Examples include secobarbital (Seconal) and pentobarbital (Nenbutal). . Intermediate-acting: for treating insomnia. Examples include amobarbital (An.vtal) and bfiabat bital lButisol). . Long-acting: for treating certain types ofseizures. Examples include phenobarbital (Z phobarbital ( Mebarul ) and pi'n]id.one (Mysoli ne). Remember: Barbiturates do not possess analgcsic properties. (i.e. phenobatbital)t Ba'rbrrurates rvork by inhibiting the depoladzation ofneurons by binding to the GABAreceptors, which en- Important concept for anticonvulsant effect of lo.rg-acting barbittrates hances the iransmission ofchloride ions. They also increase the threshold for electrical stimulation ofthe motor conex. Remember: Benzodiazepines work similarly to barbin-rates, but they also increase the number of chloride channels whilc facilitating the transmission of chloride ions. This suppresses the :pread of seizlrre activity but does not abolish abnormal discharge from a focus. \ote: The cause ofdeath from acute barbiturate poisoning or overdosage is respiratory failure. Other ad\erse reactions include CNS depression, euphoria, and habituation. The most important therapeutic measure to be taken in a case ofbarbifurate poisoning is to assure adequate respiration.

Which of the following drugs is a mernber of the opioid family and reduces GI motility ? Loperamide (Imodium) Lorazepam (Ativan) Diphenoxylate and atropine (Lomotil) Propranolol (Inderal)

Loperamide (Imodium) Antidiarrheals: ' Opiate and opioid derivatives - Loper ^mide I m mod iu m) | ( L Is an anti-diarrheal which acts on intestinal musclcs to inhibit p€ristrlsis. 2. Is a member ofthe opioid family. It does not penetrate the cenffal nervous system like the opioids such as codeiner thus it can b€ sold ov€r the counter. J. IIas no €vidence of drug abus€ or dependence (utllike other opioicls such os codeine, morphine dnri neperidine). - Diphenoxt-late is an anti-dianh€al and inhibits excessive GI tract motiliry and Gl propulsion. Commercial prcparations contain a sub-therapeutic amount ofatropine to discourage abus€. Diphenoxylat€ and. atropine (Lomatil), rcquires a prescription. . . unlike loperamide Antisecretory:bismuth st$sahcylate ( Adsorbents: attapulgile (Kaopectate) flrrodiu,rr, P epto-B is mo I ) R€member: Laxatives act in the revene manner ofthe anti-diarrheals and increase the motility of the CI ..aci. Tle] are used to trcat constipation. Examples include: magnesium hydroxide (Milt ry'Magnesid), c s- ror oil. \letamucil and methylcellulose. Antiemetics:actonthc'vomitingcenter"inthemedulla-Thiscenterhasfourdifferentsourcesofstimuli.Thc .hemoreceplor rrigger zone (CTz) is located outside the blood-brain barrier near the vomiting center in the medulla. It communicares with th€ vomiting center after input is received from drugs and hormones. The classes of antiemetics are: . Antidopaminergic: . . . . Phenothiazines: prochlorpcrazine (Conqazi ne) a'nd prcmelhazioe (Phenerydn) Benzamidesi metoclopramide /Re8/dnl and fiimcthobenzamide HCL Anticholinergics: . Antihistamines: meclizine Scop) (Antivert), dimenhydrln te Serotonin 5-HT3 receptor antagonists: ond HCL fj$,t/i, and palonosetron (,4/orl) ^t\sgtron flgdnl (Dramamine), and scopolamine A/arrderTrt- HCL (Zotan), dolasetron (Anzemer, granisetron

All of the following are advantages of using nitrous oxide analgesia EXCEPT one. Rapid and complete recovery Vitually no adverse effects in absence of hypoxia Therapeutic sedative for many medically compromised patients Suitable for all ages Rapid onset of action Lowers pain threshold Produces euphoria Pleasant induction Titratable

Lowers pain threshold itrous oxide elevat€s pain threshold Important points about nitrous oxide: . The main therapeutic effect of nitrous oxide is relaxation/sedation scious sedqtion). Mild analgesia is a secondary effect. It is not a respirutory depressant. . . The first symptom ofnitrous oxide onset is tingling of the hands. Nitrous oxide has no local anesthetic properties. Therefore, the addition oflocal anesthesia is necessary in procedures in which pain is anticipated. . (it is used in con- Long term exposure to low doses ofnitrous oxide has been shown to increase the inci- dence ofspontaneous abortions. Environmental contamination by nitrous oxide can be kept to a minimum by employing a scayeng€r syst€m. . Nitrous oxid€ is stored under pressure (7 blue. Oxygen is stored in green tanks. . 50 psi) in sleel cylinderc (in q Nitrous oxide delivery machines come pre-equipped with a failsafe m€chanism that will not allow less than 20y" oxygen to be delivered to the patient. . liquid state) pairltcd lt is a nonirritating, colorless gas with a slightly sweet odor and tasteless lt is very stable and inert chemically at room temperatures. . . It has a rapid ons et (blood:gas solubility coelrtcient It is 1.5 times heavier than air. : 0.47) and termrll'attorr. . Nitrous oxide will diffuse into air-containing cavities within the body faster than nitrogen diffirses out. This results in a temporary increase in either the pressure and/or volume of the cavity depending upon the distensibility ofits walls. This is most noticeabl€ in the bowel . Nitrous oxide interacts with vitamin Bt2 synthesis in the human body by interfering with the enzyme methionin€ synthas€, depl€ting the body of vitamin -B12. When nitrous oxide is used heavily and over an extended period of time, vitamin 812 depletion will probably become a major problem, as it can cause brain and nerve damage.

Azithromycin (Zpak, zithromax) is an antibiotic of the Macrolide class of antibiotics Cephalosporin class of antibiotics Quinolone class of antibiotics Glycopeptide class of antibiotics

Macrolide class of antibiotics Azithromycin (Z-Pak, Zithromax) and clarithrornycin (Biaxin) macrolide class ofantibiotics in which erythromycin is the prototype agent, are members of the The bacterial spectrums ofactivity ofLzithromycin and clarithromycin are similar to that of erythromycin but possess greater intrinsic activity against H. influenzae and Heli- cobacter pylori. These two macrolides concentrate within macrophages, making them useful against organisms that are taken up by macrophages such as Mycobacterium avium intracellulare. The significant tissue penetration ofboth agents and the prolonged elimination half-life of azithromycin (l l -14 hours) allows for once-daily dosing for azithromycin and twice- daily dosing for clarithromycin. Note: Macrolide antibiotics are generally used to treat infections caused by streptococcal bacteria and respiratory infections generally. They are also active against syphilis, Lyme disease and leprosy and tuberculosis. The macrolides can have a toxic effect on the liver, causing liver toxicity andjaundice and so should not be given to people who have an un- derlying liver problem such as an infection with hepatitis C. \facrolides are bacteriostatic. The mechanism of action involves inhibition of protein s! nthesis that results from binding specifically to the 50s ribosomal subunit. This causes the Rr\A to dissociate from the ribosome and prevents protein synthesis. Remember: Intrinsic activity is a measure ofthe ability ofa drug once bound to the recepto

The currently available ganglionic blocker for clinical use is Mecamylamine Hexamethonium Tetraethylammonium Trimethaphan

Mecamylamine Hexamethonium, trimethaphan, and tetraethylammonium are no longer available in the U.S. for clinical use. Nicotinic receptor antagonists (nicotitlic blocking agents) are divided into ganglionblocking drugs and neuromuscular blocking drugs. Remember: two major types of nicotinic receptors: 1. Those at the skeletal neuromuscular *** Neuromuscular blockers act here junction ofthe somatic system 2. Those at the autonomic ganglionic sites (both sympathetic atxd pqrasympathetic) *** Ganglionic blockers act here Aldrough they are among the most potent agents available, ganglionic-blocking drugs are seldom used because ofthe annoying and sometimes disabling parasympathetic ttlock- ade. The side effects (caused by parasympathetic blockade) tnc\rde a very pronounced xerostomia, constipation, blurred vision, and postural hypotension. These drugs have only \'ery limited clinical uses. \Iecam.'-lamine (nversile) is used clinically for: . . The treatment of severe or malignant hypertension An emergency hypertensive crisis *** lt cause a rapid and reversible fall in blood pressure that enables it to ilrlnediately reverse an emergency hypenensir e crisis. . A "bloodless field" surserv

All of the following are antimuscarinic agents EXCEPT Atropine Scopolamine Glycopyrrolate (Robinul) Mecamylamine (Inversine) Propantheline (Pro-Banthine)

Mecamylamine (Inversine) Ilecamylamine is a nicotinic ganglion-blocking drug. The tvpical effects ofanticholinergic drugs include r.r.rydriasis, antispasmodic actions and reduction in gastric and salivary secretiorrs (dry moutlr).lmportant: These drugs are con- traindicated in patients with glaucoma. These drugs have no intrinsic activity oftheir own; they simply occupy the receptor site and pre\ ent acetylcholine from occupying the same receptor Accepted therapeutic indi- cations include treating Parkinson's disease, motion sickless, postoperative bladder syn- drome and traveler's diarrhea.

All of the following are opium alkaloids, EXCEPT one. Meperidine Morphine Codeine

Meperidine Chernical classihcation of opioid analgesics: Opium alkaloids: . . Morphine Codeine Sl nthetic deriYatives: . )Iorphine group: . . Hydromorphone . Oxymorphone (Dilaudid) (l'l umorphan) . Nalbuphine /?fuDaif Codeine group: . . Hydrocodone Oxycodone Synthetic narcotics: . Meperidine group: . (in Vicodin) and arlxiety. especially in the elderly. (in Percodan, Percocet and Tylox) Meperidine (Demerol) . Alphaprodine . . . . (Nisentil) Alfentanil (Alfenta) Fentanyl (Sublimaze) Sufentanil (&r&rta) (in Lomotil) . Diphenoxylate Loperamide (in . Methadone group: . . Methadone Propoxyphene Imodium) (Dolophine) (Darvott)

All these drugs are second generation sulfonylures EXCEPT Glyburide (DiaBeta) Metformin (Glucophage) Glipizide (Glucotrol) Glinepiride (Amaryl)

Metformin (Glucophage) x *** Metformin is classified as a biquanide Antidiabetic agents or oral hypoglycemic agents are drugs used as adjuncts to dietto treat non-insulin dependent diabetes mellit\rs (tfpe . 2 diabetes)that cannot be controlled by diet alone. SulfonJ-lureas, the first drug group introduced into the U.S. in 1955, close potassium channels in ccll mcmbranes, stimulate the beta cells to produce more insulin., and incrcase the sensitivity oftar- geI organs to insulin. . . The original "first g€neration ndrel. and chlorprop^mide " sulfonylureas include tolbnt^mide (Orittdse), tol^z nride (Toli- (Diabinese). These drugs work well in lowering the blood sugat but thel,ha\'e a major drawback. Bccausc they bind to proteins in the blood, thcy can be dislodged by other medications that bind to thcse same idlr'and lead to low blood sugars. . proteins. Once dislodged, their activity can increase tap- Second gen€ration sulfonylureas includc glipizide (Glucotol), glimepiride (Amaryl).These Elyburide drugs have an advantage for those who use other medications since rhey do not bind to carrier proteins in the blood. Bccause ofthis, drug interactions that may cause lorr blood sugars are less likely. Biquanides: metlotfiin (Glucophage.) primarily decreases hepatic glucose production. It also has minor effects on insulin sensitivity in both the liver and peripheral tissues. It has no direct effect on lhe . pancreas and therefore does not enhance insulin secretion. and pioglit^zone (Actos) increase senstivify in the musclc and liverby improving control ofglycemic utilization. This in tum reduces circulating insulin Thiazofidinediones: rosiglitazone (Avandia) levels. Notei Functioning beta-cells are required for these medications to work. . Meglitinides: repaglinide (Prundin) and oateglinide /Slarllxl (Micronase) and lower blood sugar by stimulating the rclease ofinsulin fiom the pancreas in short bursts. Note: Functioning beta-cells a.e required for thcsc medications to work. . o-Glucosidase inhibitors: acarbose fPl€coreJ and miglitol /Gf,set act through inhibition ofpan- creatic d-amylase and membrane-bound intestinal cr-glucosidc hydrolase enzymes. This enryme in- hibition delays glucose absorption. These enzymes do not enhance insulin secretion

All of the following drugs used to treat hypothyroidism EXCEPT Levothyroxine sodium (Synthroid) Liothyronine (Cytomel) Liotrix (Thyrolar) Methimazole (Tapazole)

Methimazole (Tapazole) Thyroid medications: . . Thyroid supplements: used to treat hypothyroidism . . Levothyroxine sodi|dm (synthetic T4) Liothyronine (synthetic Tj) ' Liotrix (T4;73= 4 l) Note: These drugs are usually taken orally in a single daily dose. preferably before breakfast- The treatment of choice for hypothyroidism is T4. lt has a relatively slow onset of action, and its effects are cumulative over several weeks. T3 has a more rapid onset ofaction and dissipation ofaction. Adverse side effects include nervousness, nausea, dianhea, tachycardia, tremoq weight loss and heat intoler- ance. Thyroid suppressants: used to treat hyperthl,roidism . Methimazole . Propylthiouracil (Tapazole) (PIUI \ote: These drugs inhibit thyroid peroxidase, which is an enzyme expressed mainly in the thyroid that liberates iodine for addition onto tyrosine residues on thyroglob- ulin for the production of the thyroid hormones nrne (Tj). - thyroxine (I4.; or triiodothyro- Through reduced absorption of iodine, thyroid hormone synthesis is diminished. Adverse effects include rash, nausea, and agranulocytosis. \ote: Thyrogen is a recombinant DNA source ofhuman TSH useful in the management and lreatment of thvroid cancer oatients.

Which of the following has a clinically significant drug interaction with Amoxicillin? Triazolam (Halcion) Methotrexate Calcitrol Candesartan

Methotrexate *** Amoxicillin in large doses inhibits the renal tubular secretion of methotrexate, thereby causing higher, prolonged serum levels ofmethotrexate. Aminopenicillins (enpicillin (streptococci, and amoxicillin) are characterized by the amino substitu- tion ofpenicillin G. They are able to penetrate gram-negative bacteria more readily than are the natural penicillins or the penicillinase-resistant penicillins. However, the aminopenicillins are not stable to beta -llct^ma;ses (penicillinase) or gram-negative bacteria. of either gram-positive Note: Gram-negative bacteria that are susceptible to aminopenicillins include: H.in- fluenzae, some E.Coli and Proteus mirabilis. Ampicilf in (Pol1,cillin, Onipen) and Amoxicillin (Amoxil, Larotid) are ttsedprimarily to treat infections such as otitis media, bronchitis, sinusitis, and acute bacterial cystitis caused by susceptible organisms. Compared with ampicillin, amoxicillin has a higher oral absorption, higher serum levels, a longer halflife, and is less likely to cause adverse GI ef- lects (diarrhea). Amoxicillin is given orally; ampicillin can be given orally or IV. )rlote: Ampicillin and amoxicillin are preferred agents in the treatment ofurinary tract infections caused by susceptible enterococci (which anaerobic, grampositive cocci that grow in short chains under extreme conditions mainllt in the intestine). are facultatively

The urinary elimination of drugs is controlled by all EXCEPT Glomerular filtration Microsomal enzyme induction Renal tubular reabsorption Active transport through renal tubular cells

Microsomal enzyme induction Glomerular liltration: all drugs are filtered through the glomerulus to enter the renal tubules. The amount of drug varies according to the degree of plasma protein binding, and bound drugs are not subjected to filtration. . Tirbular reabsorption: once they enter the renal tubules, drugs may be reabsorbed back into the blood stream through the renal tubular cells. Reabsorption favors the highly lipid soluble agents; the converse is that highly polar compounds are not effectively reabsorbed and are effectively excreted from the renal tubules. . Active transport through renal tubular cells: the rate of renal elimination also depends on whether active transport into or out ofthe tubular fluid occurs. Other excretory pathways for drugs: . The gastrointestinal tract excretes some drugs through the feces. This is not as preva- lent as urinary excretion. . Most drugs can be detected in saliva after administration, but the salivary glands are not considered a route ofdrug excretion since the drug is re-swallowed along with the saliva. . Lungs excrete volatile compounds that were originally inhaled into the system. Ni- trous oxide and the volatile general anesthetics are excreted by this route. . Some drugs are excreted in the sweat' but this route accounts for only a small per- centage of drug excreiion.

The following drugs belong to what pharmaceutical class of agents: -Adrlimumab -Alefacept -Infliximab -Trastuzumab Immune modulators Monoclonal antibodies Immunosuppressants Interferons Colony stimulating factors

Monoclonal antibodies Humira) rs arecombinant monoclonal antibody that binds to human tumor necrosis factor alpha (TNF-alpha) receptor sites. It is used to treat active rheumatoid anhritis. Adalimumab (Amevive) is a monoclonal antibody used to treat moderate to severe plaque psoriasis. Afefacept Infliximab (Remicade) is a monoclonal antibody used to treat ankylosing spondylitis, Crohn's disease, and rheumatoid arthritis. Like adalimumab, infliximab works by bind- ing to TNF-alpha receptor sites. Trastuzumab (Herceptin) is a monoclonal antibody which binds to the extracellular domain of the human epidermal growth factor receptor 2 protein (HER-2) the treatment of patients with metastatic breast cancer whos€ tumors overexpress the HER-2 protein and who have not received chemotherapy for their metastatic disease. ' It is used for

Which drug is the standard to which all opiods are compared Codeine Oxycodone Hydrocodone Meperidine Morphine Fentanyl

Morphine Opioid analgesics are thought to inhibit paintul stimuli in the substantia gelatinosa ofthe spinal cord, bmin- stcm. rcticular activating system, thalamus, and limbic system. Opiate rcccptors in each of thcse areas inter- act with neurotransmittcn ofthe autonomic nervous system, fhc opioid action ofthc drug manil-e\1. as: . . Dro\\'siness Euphoria . \fentalcloudrng . . producing alteralions in reaction to painful stimuli. Decreased peristaltic motility Nausea and vomiting . . Respiratory depression Depression ofthe cough reflcx . \lrosts @upillan, constriction) . Orthostatic hypotension \Iorphine is a narurally occurring opiate tha! is metaboliTed chiefly through glucuronidation by uridine ciphLriphare glucuronosyl transferase meiaboltle ( m o rp h i n e- 6 - E: lucuron id e ). fUG4 €nzymes in the liver These enzymes produce an active analgesic Srgri and s]mptoms ofacute opioid intoxication:!he intoxicaled person is stuporous or asleep and has con- s-icied pupils /pir-poi t p?./plls/ and depressed respimtion- As the severity ofintoxication increases, coma en- sucs. \ote: Death from acute intoxication by an opioid analgesic is the result ofpmfound, direct respiratory depression. Remember: L Codeine is weaker f/e.rr pol€r, than morphine and less addictiv€. 11 is converted to morphine by cy- rochrome p,l50 2D6. Note: It diffcrs from morphine in that a methoxy (-OXIIj) substitution replaces the hi dro\yl (-OH) group on the aromatic ring ofthe molecule. This relatively minor structural change pro\ldes codeine with significant oral effectiveness. :. ox]codone has a similar potency as morphine. ln combination with acetaminophen it is known as P€r- cocet or Tylox. 3. HJ''drocodone has a similar potency as morphine. In combination with acetaminophen it is known as vicodin and Lorcet. 4. Meperidline (Denero, is more potent than codeine but less potent than morphine butjust as addictive. It has a shorter duration ofaction. It is the only narcotic agent tbat does not cause miosis 5. Methadone (Doloprl,e) is an opioid agonist used in maintenance for treating opioid addiction. 6. Buprenorphine fsrrrle, is an opioid partial agonist uscd to treat opioid dependence. lPupillary con-

Which narcotic is not used in dentistry Morphine Hydrocodone (component of Vicodin) Oxycodone (component of Percocet) Meperidine (Demerol) Codeine Fentanyl

Morphine Therapeutic indication for opioids (narcotic analgesics) are the relief of moderate-to- severe pain, as preanesthetic medications, as analgesic adjuncts during anesthesia, as an- titussives, and as antidiarrheals. Note: They are administered with caution to patients with head injury or those with a history ofdrug abuse and dependency. l. Morphine is not used in dentistry because ofits high addictive liability. 2. Hydrocodone in combination with acetaminophen is known as Vicodin, Lorcet, Lortab, Maxidone, and Zydone. 3. Ilydrocodone in combination with ibuprofen is known as Vicoprofen, 4. Oxycodone in combination with acetaminophen is known as Roxicet Per- cocet, and lllox. 5. Oxycodone in combination with ibuprofen is known as Combunox. 6. Oxycodone alone is klown as Oxycontin. It is more potent than codeine. 7. Meperidine (Demerol) in combination with promethazine is known as Mepergan Fortis. 8. Codeine in combination with acetaminophen is known as Tllenol #3. 9. Fentanyl is available as a transmucosal preparation known as Actiq, a trars- dermal patch formulation known as Duragesic and as an intravenous prepa- ration known as Sublimaze, Among the opiates available for use in dentistry, hydrocodone products are commonly the drues ofchoice.

The CNS contains 3 types of endogenous opiods. Wwhich is not one of them Beta-endorphins Morphine Enkephalins Dynorphins

Morphine opiate receptors in the CNS mediate analgesic activity. Opioid agonists occupy the same receptors as en_ dogcnous opioid peptides, and both alterthe central release ofueurotransmitters ilom afferentrerves sensitive to no\ious stimuli. They decrease presyraptic release ofneurotranmitters and incrcasc postslnaptic potential. The opioid reccplors all have a common general strucfur€. They are characleristically G protein-linked receptors enrbedded in thc plasma nrembrane ofneurons. Once the receptors are bound, a portion ofthe G prorein /Gtl is activated, allowing i! !o diffxse within the plasmn menbmne. The G protein moves within the membrane until it rcaches ils larget -cithcr an cnzyme or an ion channel. Opi[)id receplors in lhe centml nervous system are thought to be activated by endogenous chemicals under Fh\ iiologic condirions. fie body coniains three types ofthese chemicals that . . Beta+ndorpbins bind to opioid receptors in the brain and have produce morphine-like effects potent analgesic activity. Enkephalins bind to opioid reccptors in the brain and are more widely distributed in the brain than lhc bera-endorphins. Seem !o . play a role in pain perception, movement. mood, and behavior peripheml nen ous syslems. Some research supports the theory that they regulate pain at the spinal cord level, in- Dlnorphins are the most powerful ofthese cbemicals and are lbund throughout the central and fluence feeding behavior at the hlpothalamic levcl and function with other cndogenous opioids to regulatc the cardiovascular systcm. Opioid receptors: l. mu (!) mediate morphinelik€ supraspinal analgesia, mitosis, respiratory depression, €uphoria, physical dependence, and supression ofopiale withdrawal. Nolei The protot)?ical opioid agonist for this receptor is morphine, and its analgesic activity is considered to depend on its binding to this reccptor (6) mediate antagonist activity. Note: The €nkephalins are considered to be the q?ical agonist tbr this rec€ptot 2. della L kappa (K) mediate spinal analgesia, respiratory depression, and sedation. \ote: The dynorphins are agonist for this receptor Sites of analgesic action of opioids: The opioid drugs produce analgcsia by actions at several levels ofthe thought to be the R?ical ncwous system, in particular, inhibition ofneurotransmitter rcleasc f.om the primrry alferent terminals in the spinal cord and activrtion ofdescending inhibitory controls in the midbrain. *** Opioid analgesics (i.e., motphine. codeine, mepericline. propoxvp,ftcn€, eL..) mimic endogenous opioids at CNS opiate recepiors, raising the pain threshold and increasing pain tolerance.

The main ingredient in advil is also the main ingredient in which of the following presciption products Percodan Vicodin ES Motrin Naprosyn

Motrin \dYil contains 200 mg ofibuprofen and may be sold over-the-counter; whereas, Motrin contains .100 mg of ibuprofen or higher and can only be sold with a prescription. \onsteroidal anti-inflarrunatory drugs (NSAID, have anti-inflammatory effects resulting frorn their ability to inactivate the enzyme prostaglandin endoperoxide synthase cloon'genase).By doing this they inhibit the cyclooxygenase step ofthe arachidonic acid cascade and thus reduce local prostaglandin synthesis. NSAIDs also have analgesic, and antip)'retic actions. Remember: The traditional NSAIDs such as ibuprofen, naproxen and aspirin inhibit COX-2 along with COX-I. Thus they are effective in reducing pain and inflarnrnation, but are capable ofinducing gastrointestinal ulcers. The COX-2 selective inhibitors will reduce pain and inflammation without any significant risk ofcausing gastrointestinal ul- cers. For the traditional NSAIDS such as ibuprofen, naproxen, and aspirin, because they in- hibit both COX-1 and COX-2 enzymes, they belong to the category of non-selective COX inhibitors. For celecoxib (Celebrcx), because it inhibits COX-2 enzyme only' it be- longs to the category oICOX-2 sel€ctive inhibitors. Note: Rofecoxib (Woxx) and Valdecoxib (Bextra) are also COX-2 selective inhibitors that u,'ere removed from the market because they were found to have added cardiovas- cular risks in some Datients.

Which statement describes the extrapyramidal syndrome (EPS) caused by the phenothiazine type antipsychotics? Orthostatic hypotension Sedation Headache Dry mouth Muscle spasms of the oral-facial region

Muscle spasms of the oral-facial region Ertrapyramidal syndrome lEP.l) refers to a variety of signs and symptoms that are a result ofthe blockade ofdopamine receptors in specific brain regions. These symptoms include: Parkinsonlike movements (shufiIecl gait, pill-rolling eJlbct oJ rigidiry. spasms ofneck and facial muscles, tremors, and loss of muscle movement. Tardive Dvskinesia (TD) is a serious, irreversible neurological disorder that can appear at any age. Tardive Dyskinesia is a side effect of taking antipsychotic/neuroleptics drugs. Symptoms involve rurcontrollable movement ofvarious body parts, including the body trunk. legs, arms, fingers, mouth, lips, or tongue. About 20 percent ofpeople taking antipsvchotic/neuroleptic drugs for more then one year will be affected. Remember: Newer fngers), (second generation) or atypical antipsychotic agents affect differ- ent receptor sites compared with first generation antipsychotics. They bind dopamine, including Dl, D2, Da, and D5 receptors, with selectively for limbic dopamine receptors. They have increased alfinity for serotonin (5-HT) receptors compared with D2 recep- tors. They exhibit reduced ability or an inability to induce EPS. Second generation agents include clozapine, risperidone, olanzapine, quetiapine, ziprasidone, and aripipra- zole. Important: Clozapine is specific for limbic receptors and not for stdated (muscle) ceptors, which explains the low incidence of EPS and TD. muscle re-

If norepinephrine or eplnephrine were to stimulate or combine with the alpha receptors in the eye, which response would you expect? Miosis (contraction of the pupil) Mydriasis (dilation of the pupil) Neither of the above; norepinephrine and epinephrine do not stimulate or combine with alpha receptors in the eye

Mydriasis (dilation of the pupil)

Which drug is the prototype opioid antagonsit Morphine Naloxone Propranolol Ibuprofen

Naloxone Naloxone (Narcaz) is a narcotic antagonist and is used in medical emergencies to teverse narcotic overdose. Overdose of narcotics results in respiratory depression and death due to respiratory shut down. Naloxone will reverse the respiratory depressant effects ofthe narcotics thus counteracting the lethal effects of these agents. Naloxone is given intravenously, intramuscularly or subcutaneously. Note: Naloxone does not have agonist ac- tivity at any opioid receptors. Nalmefene (Revex) and naltrexone (ReWa) are the other two narcotic reversal agents used to reverse the respiratory depressive effects olthe narcotic analgesics. Typical narcotic analgesic drugs which are reversed by naloxone (Narcan), nalmefene (Revex), . . . ' . . and naltrexone Codeine Morphine Hydrocodone Oxycodone \{eperidine (Demextl) Fentanyl (Rena) Lnclude: \ote: Naltrexone (Rel/ia) is also used to treat alcohol dependence.

Your patient has a history of neuropthic pain. They are currently taking oxycodone (oxycontin). Which is an opiate analgesic. Which two re dverse effects of oxycodone Nausea Peptic ulcers Insomnia Constipation

Nausea Constipation The most common side effect of the narcotic (opiqte) analgesics is nausea. The most serious side effect of the narcotic analgesics is respiratory depression. The cause of death from overdose ofnarcotics is respiratory depression and shut down ofthe respira- rory sysrem. Narcotic analgesics do not cause peptic ulcers. Nor do they cause insomnia, but rather would cause drowsiness and sedation since narcotics depress the conscious centers of the brain.

Which form of local anesthetic can readily penetrate tissue membranes Ionized form Nonionized free-base form Both ionized and nonionized free-base forms

Nonionized free-base form Local anesthetic free bases are fat-s oluble (lipophilic) drugs. They are converted to their irater-soluble (hydrophilic.l hydrochloride salts to allow preparation ofan injectable so- lution. ln solution an equilibrium is established between the ionized and the nonionized tbm.rs ofthe local anesthetic. The proportion ofthe drug in the ionized form depends on rhe pKa of the drug and the pH ofthe solution. At the usual solution pH of 6.0 or lower, most local anesthetics are almost completely in the ionized form. This is important because only the nonionized free-base form ofthe drug can readily penerrate tissue membranes. Local anesthesia can only be obtained if sufficient free-base is ar ailable. The lower the pKa ofthe drug and the higher the pH ofthe solution or injected tissues. the more free-base will be available. Once the local anesthetic is injected the buffering capacity and pH of the tissues nallt T.1) shifts the equilibrium in favor of free-base formation. At physiologic pH of 7.4 approximately 5-20ulo ofthe local anesthetic is in free-base form which is enough to pen- etrate and cause anesthesia. Ifinfection or inflammation is present, the pH olthe tissues may be acidic and there is a significant reduction in the concentration of the free-base form. In this situation, the local anesthetic may not be effective. Key point of all this: The potential action olall local zuresthetics depends on the ability ofthe anesthetic salt to liberate the free-base. Note: Ifyou inject lidocaine (pKa = 7.8) into tissue that has a pH of 7.8, the lidocaine will exist in an equal mixture ofionized and nonionized forms which will be more than enough to Droduce aresthesia.

Stavudine (also known as d4T, or Zerit) is an antiretroviral drug used in the treatment of adults with HIV infection in combination with other antiretroviral agents. Which of the following categorles does this agent belong to? Nucleoside reverse transcriptase inhibitor Protease inhibitor Non-nucleoside reverse transcriptase inhibitor

Nucleoside reverse transcriptase inhibitor These agents chemically are nucleosides and work by inhibiting the viral enzyme known as reverse transcriptase, This results in an inhibition ofthe HIV viral RNA from being made into a DNA segment; thus the genome ofthe HIV virus is not copied from RNA. Other agents in this class include didanosine zidovudine (Ret ovir ; AZT). (Wdex), zalcitabine (Hivid; ddC) and Proteas€ inhibitors suppress viral replication by inhibiting protease' the enzyme responsible for cleaving viral precursor peptides into infective virions. Some agents in this class include indinavir (Crixivan), nelfinavir (nracept), ritonavir (Norvir), and. saquinivir (lnvirase). \onnucleoside reverse transcriptase inhibitors inhibit the catalytic reaction of reverse transcriptase that is independent ofnucleotide binding. Some agents in this class include delavirdine (Rescriptor), adefovfi (Hepsera) and nevirapine (Wramune).

The drug of choice for treating candidiasis is Penicillin Erythromycin Nystatin Chloramphenicol

Nystatin Candidiasis is an infection, usually ofthe oral cavity or vagina, with a candida species, usually C. albicaDs, which causes an inflammatory, pruritic infection characterized by a thick, white dis- charge. It is common, especially in patients who have a deficiency in T-ly,rnphocytes, or who are s).Thisyeast-like fungi is a normal inhabitant of the oral cavity and vaginal tract, however it is normally held in check receiving chemotherapy, and in immunosupressed individuals b; the indigenous bacteria ofthese areas. Note: Angular cheilitis Oilateral rhe ntoutht has been linked to C. albicans. /,4lDSpalie Remember: Nystatin and clotrimazole are the two antifungals that are used as ulcers at the comer of fo\r'' to treat oral candida infections. Nystatin (Mycostatin) is taken as an oral suspension to be sn ished around thc mouth and swallowed. Clotrj.lri.azolc (M))celer, is taken as a troche (lozenge) \\hich is slorvly dissolved in the mouth and swallowed. They work by binding to sterols in the fun- gal cell rnembrane, increasing permeability and permitting the leakage ofintraccllular componcnts. This leads to the death ofthe atTected fungal cell. Note: The suffixes ofthe following generic drug narnes are indicative ofthe conespsonding drug classes: . . . "azole" = azole-type antifungal drugs (e.9., clotrimazole) "coxib" "dipine" = COX-2 inhibitors fe&, celecoxib) = dihydropyridine calcium channel blockers (e.g., nicardipine) . "olol" = beta-adrenergic receptor blockers (e.g., naclolol . "ilol" ) or "alol" = beta-adrenergic receptor blocker that also blocks alphal-adrenergic receptors (e.g., canedilol or labetalol) . "onium" or "urium" : "swish quatemary ammonium compounds, usually competitive peripheral act- ing skeletal muscle relaxers (e.9., pancuronium) . "osin" . . . = alphar-adrenergic reccptor blockers (e.g., terazosin) "pril" - ACE inhibitors (e.g., enalapril) "sartan": angiotensin II receptor blockerc (e.9., olmesartan) "statin" = HMG-CoA reductase inhibitor (e.9., atoflastatin) and swal-

Which route of administration is most known for its significant hepatic "first pass" metabolism Intramuscular Inhalation Sublingual Oral

Oral The oral administration of a drug is the one most acceptable to the patient. It is convenient because drugs can be given in the form oftablets or capsules which contain an exact dose. lt is easy and the patient can take the drug without help from anyone else. One ofthe disadvantages of drugs taken by mouth is that they have to be absorbed (trsu- ulh fiom the small intestine) before they can be transported to their site ofaction. Blood from the intestinal ract passes first to the liver: some drugs are metabolized in the liver ( "lirst-pass elfect") and others n.ray be stored there to be released slowly. This considera- rion makes it clear that oral administration in usually unsuitable in emergencies or on other occasions when a rapid effect is needed. Intramuscular injection is an injection made into a large muscle. The advantages of IM injection are that it results in uniform absorption and that it can be used for solutions too iritant for subcutaneous injection. The speed ofabsorption of drugs given by IM in- jection depends on dre vehicle in which they are dissolved: absorption is rapid from aque- ous solutions and slow from oily solutions.

The safest and easiest route for drug administration is Oral IV Rectal Inhalation

Oral (abbreviated P.O.) Howeveq it is also the most unpredictable and least effective route available. Drugs taken by mouth have to be absorbed (as ually fr om the small intestine) before they can be transported to their site ofaction. Absorption may be slow, unpredictable and inegular due to the presence of variable amounts of food in different stages of digestion and to the varying degrees ofacidity and alkalinity ofthe digestive juices. Moreover, blood from the intestinal tract passes first to the liv€r: some drugs are metabolized in the liver and oth- ers may be stored there to be released only slowly. These considerations make it clear that oral administration is usually unsuitable in emergencies or on other occasions when a rapid effect is needed. Note: After oral administration, drugs will generally be absorbed best from the duodenum. The duodenum has a large surface area due to the presence of villi and n.ricrovilli. Note: A major advantage of IV administration ofa drug is it allows for titration ofthe drug. Other advantages of IV administration include: . . Rapid onset Drugs that cause irritation when administered subcutaneously can be given IV with no irritation . In case of emergency, there is an open line through which emergency drugs can be injected One major disadvantage of IV injection is that since it has such a rapid onset ofaction, overdosage may have effects so immediate that it is impossible to reverse them.

Which two antivirals are classified as neuraminidasae inhibitors Acyclovir (Zovirax) Oseltamivir (Tamiflu) Amantadine (Symmetrel) Rimantadine (Flumadine) Zanamivir (Relenza)

Oseltamivir (Tamiflu) Zanamivir (Relenza) Oseltamivir (Iarr iflu) and zanrmivir (Relenza) inhibit influenza virus neuraminidase enz! mes. potentially altering virus particle aggregation and release. Tamiflu and Relenza are both used to treat acute illness due to influenza (A or B) infectior'. Acyclovir (Zovirax) is an antiviral that inhibits DNA synthesis rather than neuraminidase enzymes. Amantadine is a synthetic anti-viral drug that can inhibit the replication of viruses in cells. lt was initially used to prevent influenza A during flu season, and, if given within 24 to 48 hours of the onset of flu symptoms, to decrease the severity of the flu. Later amantadine was found to cause improvement in the symptoms ofParkinson's disease. Not€: The mechanism of amantadine's antiviral activity involves interference with a viral protein, M2, which is required for the viral particle to become "uncoated" once taken in- side a cell by endocytosis. Rimantadine is a synthetic anti-viral drug that can prevent viruses in cells from multi- plying. Like amantadine, rimantadine initially was used to prevent influenzaA during flu season, and, if given within 24 to 48 hours after the onset of flu symptoms, to decrease the severity ofthe flu. Rimantadine is chemically related to arnantadine, but rimantadine has fewer side effects on the nervous system than amantadine. Note: Rimantadine appears to exen its inhibitory effect early in the viral replicative cycle, possibly inhibiting the uncoating ofthe virus (similqr to amantadine s nechanism oJ action).

What serious dental elfect is associrted with the following drugs: zolendronic acid (Zometa), pamidronate (Aredia), risedronate (Actonel),, ibandronate (Boniva) and alendronate (Fosamax)? Mucositis Osteonecrosis of the jaw Angular cheilitis Oral yeast infection Xerostomia

Osteonecrosis of the jaw Zoledronic acid (Zometa), pamidronate (Aredia), fisedrci^te (Acto el), ib^odtonate alendror.ate (Fosamax) are members ofthe bisphosphonate class ofdrugs used to heat and manage Pag€t's diseas€, osteoporosis and to prevent hypercalcemia ofmalignancy. Bisphoshonate therapy has been associated with osteonecrosis, primarily ofthe jaw; this has been observed mostly in cancer patients, but also in patients with postmenopausal osteoporosis and other diagnoscs. Risk factoN include a diagnosis ofcancer, with concomitant chemotherapy, radiotherupy or corticosteroids; anemia, coagu- lopalh!. infection or pre-existing dental disease. Symptoms included nonhealing cxtraction socket or an e\posed jawbone. There is no data addressing whethe. discontinuation oftherapy reduces the risk ofde- \ eloping osteonecrosis. However, as a precautionary measure, dental exams and preventativ€ dentistry should be perlormed prior to placing patients with risk factors on chronic bisphosphonate therapy. Important: Bisphosphonates have an affinity for hydroxyapatite crystals in bone and act as antitesorptir e agents. Their primary mechanism ofaction involves inhibition ofosteoclastic bone resorption. Ad- r erse effects /6esider osteohecftrsis of the Orher agents that have an effect on bone: . SERlls fse/ectil,e jaw bor€) include GI symptoms and esophagcal erosions. eJtrcgen ,eceptor modulatorsl. R^lo\ifeie and decreascs overall bone fumover. It is used for heating osteoporosis. . Hormones: . (Bohiw), (Evisla) reduces rcsorption ofbone Calcitonin: is a naturally occurring hormone that is produced by parafollicular C-cells in the thy- roid gland. It is known to block bone resorption through its potent inhibitory elTects on osteoclasts . Parathyroid hormone: PTH is the primary regulator of calcium and phosphate metabolism in bone and kidney. Physiologic actions include regulation ofbone metabolism, renal tubular reabsorprion ofcalcium and phosphate, and intestinal calcium absorption. Once-daily adminishation of P'lH (tefiparatide IForleol) stimulates new bone formation via preferential stimulation of os- teoblastic activify over osteoclastic activity. Note: PTH also acts on th€ kidneys by reducing renal clearance ofcalcium which increases plasma calcium. It also stimulates the production ofthe ac- tive form of vitamin D in the kidney. . Vitamin D: helps to ensure that the body absorbs and retains calcium and phosphorus. which are crit- ical for building bone. Note: 1,25-dihydroxycholecalciferol is the biologically active form ofvitamin D (Cholecalciferol)

Amyl nitrile is used ln ths emergency treatment of cyonide poisoning because it: Irreversibly binds cyanide Competes with cyanide for binding of cytochromes Inhibits tubular reabsolption of cyanide Oxidizes hemoglobin

Oxidizes hemoglobin Amyl nitrite oxidizes hemoglobin to methemoglobin which binds cyanide tightly, keeping it in the peripheral circulation and preventing its access to tissues. Amyl nitrite is a vasodilator and a highly volatile substance administered by inhalation only. It is the most rapidly acting ofthe antianginal drugs, producing effects within l0 seconds. Its duration of action is only 3 to 5 minutes. Because of its extreme potency, there are uncomiortable side effects that invariably occur with rts :use (ainting and a pounding headache). lmportant: This drug is rarely prescribed and is not the first drug ofchoice in treating angina. It is abused to produce euphoria and as a sexual stimulant. Other anti-anginal drugs include: 1. Nitrate: Nitroglycerin *** This drug is the single most effective agent available for the management ofacute angina episodes. Note: It dilates mostly veins. 2. Non-nitrate vasodilator: Dipyridamole (Persantine) 3. Beta adrenergic blocking drugs: reduce cardiac rate and force -1. *** . . P ropr anolol (l ndera I) Nadolol (Corgard) . Atenolol (Tenormin) Calcium channel blocking drugs: dialte peripheral and coronary blood vessels . ltrapamil (koptin, Calan) . Diltiazem (Cardizem) . N rfediorne (P ro card ia )

For a patient who is taking anticoagulants whats the most valuable test in evaluating the patients surgical risk PTT (Partial Thromboplastin Time) PT (Prothrombin Time) Platelet count

PT (Prothrombin Time) This test is a one-stage t€st for detecting certain plasma coagulation defects owing to a deficiency offactors V VII, or X. Thromboplastin and calcium are added to a sample of the patient's plasma and simultaneously, to a sample from a normal control. The length of time required for clot formation in both samples is observed. Thrombin is formed from prothrombin in the presence ofadequate calcium, thromboplastin, and the essential tissue coagulation factors. A prolonged PT therefore indicates deficiency in one ofthe factors, as in liver disease, vitamin K deficiency, or anticoagulation therapy with the drug coumadin. International Normalized Ratio (/r'R/. Once prothrombin times are determined, they are expressed as an INR value. INR stands for International Normalized Ratio and essen- tially is the ratio ofthe prothrombin time measured in the patient divided by a standard prothrombin time value, and multiplied by a constant. An INR value of I means normal prothrombin times ofapproximately l2 seconds; normal blood clotting would be present. INR values greater thar 1 indicate that there is an anticoagulant effect. The higher the INR value, the greater the anticoagulant effect. Many patients taking anticoagulants have INR values of2-3 and even uo through 6.

Sulfonamides (aka sulfa drugs) are structurally similar to Penicillins Erytfuomycins Acetylsalicyclic acid (Aspirin) Para-aminobenzoic acid (PABA)

Para-aminobenzoic acid (PABA) Sulfonamides are often referred to as "sulfa drugs" because their molecules contain sul- fur atoms. They have a different antibacterial mechanism from that ofthe antibiotics. The sullbnamides are structurally similar to PABA and this similarity is the basis for their an- tibacterial actions. PABA is needed by bacteria for the synthesis of folic acid. In tum, lblic acid is needed for the synthesis of cellular components within the bacteria to allow for cell growth. Because of structural similarities between sulfonamides and PABA, the sulfonamides compete with PABA and are able to inhibit the actions of PABA. With PABA inhibited, folc acid is not synthesized within the bacteria, and bacterial cellular growth is inhibited. Sulfonamides are predominantly bacteriostatic agents. Examples of sulfonamides: Sulfacetamide, Sulfadiazine, Sulfadoxine, Suliamethizole, Sulfamethoxazole, Sulfanilamide, Sulfasalazine, and Sulfisoxazole. ,,Note*:.. ',.,1::;:r,:,:: 1. Sulfonamides are not used for treatment of dental infections because ofa low degree of effectiveness against oral pathogens. 2 Sulfonamides are used in medicine primarily for the treatment of urinary tract infections. 3. Bactrim is the brand name for the combination of trimethoprim and sulfamethoxazole. It is considered the drug of choice for many urinary tract in- fections. Bacteria susceptible to Bactrim include E. coli, Pneumocystis carinii. Klebsiella pneumoniae, Haemophilus influenzae and Salmonella species 4. The trimethoprim component is an antimicrobial and the sulfamethoxazole is one ofthe sulfonamides. 5. Hypersensitivity reactions ate common. Although blood dyscrasias are relatively rare, they can be fatal.

Which penicillin is degraded in stomach acid and is usually given parenterally (by injection) Penicillin VK Amoxicillin Penicillin G Ampicillin

Penicillin G Acid stable penicilhns (may be used orally) include: . . . . . . Penicillin VK Amoxicillin Ampicillin Nafcillin Oxacillin Cloxacillin . Dicloxacillin Extended spectrum penicillins include: . The aminopenicillins - Broad spectrum penicillins include: . Piperacillin . Ticarcillin *** Ampicillin and Amoxicillin These two penicillins have the widest spectrum ofall the penicillins Penicillinase-resistant penicillins include: . . . Nafcillin Oxacillin Cloxacillin . Dicloxacillin

Which is known as the original penicillin Penicillin V Penicillin G Ampicillin Amoxicillin

Penicillin G All penicillins are derivatives of 6-aminopenicillanic acid and contain a beta-lactam ring structurejoined to a thiazolidine ring. The betalactam ring is essential for its anti- tracterial activity. This basic structure is synthesized by the penicillium mold from two amino acids, L-cysteine and L-valine. Antibiotics containing this betalactam ring are re- fered to collectively as beta-lactam antibiotics and include the penicillins, cephalosporins, and the two newer goups ofagents, the carbapenems and the monobactams. Penicillln G (benzylpenicillin) is the prototype for comparison. By side chain substitutions (speciJically, this means substituting other groups at the R position ofthe penicillin mol' ecule) of thebasic 6-aminopenicillanic acid molecule, the semi-synthetic penicillins are produced which are more acid stable, have a broader spectrum, or are penicillinase re- sistant. Other naturally occrming penicillins include: . Penicillin VK (Pen Vee K, V-cillin K) - preferred for treating oral infections because it is more acid stable (more reliable oral absorption) . . Penicillin G parentzl (Pfzeryen) - always given by IM route Penicillin G benz hine (Bicillin C-R) always given by IM route. It is used for the treatment of syphilis and prevention ofrheumatic fever Remember: L Penicillins are bactericidal; they inhibit cell wall synthesis. 2. Probenecid increases blood levels of natural penicillins and may be given concur- rently for this purpose.

Of the drugs listed which is the most preferable antibiotic for the treatment of non-penicillinase producing gram positive staphylococcal infections Tetracycline Clindamycin Ampicillin Cefaclor (Celcor) Penicillin VK

Penicillin VK It is prudent to use an antibiotic with narrow spectrum ofaction and one that is bactericidal in order to minimize the development ofbacterial resistance. Penicillin VK has these properries. Ampicillin has a broader spectrum of action than penicillin VK and cefaclor is a broad spectrum cephalosporin. Tetracycline and clindamycin are bacteriostatic an- tibiotics and not bacteriocidal. Note: The major disadvantage of the penicillins is their rather high incidence ofallergic reactions. Approximately l0olo ofthe general population is allergic to penicillins. This in- cidence probably holds for any ofthe specific penicillins since there is cross allergy from one to the other Skin rash (a delayed reaction) is the most prevalent allergic manifesta- tion. Life threatening anaphylaxis can occur, but is very rare, particularly with oral dos- ing. In non-allergic individuals, penicillins at normal therapeutic doses have virtually no side effects what so ever Penicillins are bactericidal' that is, they actually cause death of the invading bacteria.

Which drug can be used in a pregnant patient Tetracycline Doxycycline (Vibramycin) Minocycline (Minocin) Penicillin VK

Penicillin VK Penicillin VK is not harmful to the fetus when taken by pregnant mothers. [t is not in- corporated into bony tissue or in the teeth ofchildren like the tetracyclines. Note: Tetracycline and all members of the tetracycline family are contraindicated in children up to 8 years old and in pregnant women. Tetracyclines have the ability to chelate calcium ions and become incorporated in the bony tissues. The teeth ofchildren who have been given the drug may develop a greenish-brown discoloration. This effect is sometimes seen in the newly erupted teeth of infants whose mothers have received tetracycline during pregnancy. R€member: Penicillins, which are bactericidal against susceptible organisms, disrupt s) nthesis ofthe bacterial cell wall and compete for and bind to specific enzyme proteins that catalyze transpeptidation and cross-linking. The enzymes to which tbey bind are called penicillin-binding proteins (PBPs). They consist of transpeptidases, transglyco- sylases. and D-alanine caboxykinases and are implicated in the final phases ofbuilding and reshaping ofthe bacterial cell wall while it is growing and dividing.

Pain that has no organic basis and is fixed upon some portion of the anatomy is referred to as: Intractable pain Referred pain Psychogenic pain Phantom pain

Phantom pain For example, the sensation ofpain felt in a limb, although that limb has been amputated. Other terms to know: .Intractable . pain: is pain that is resistant or refractory to ordinary analgesic agents Referred pain: is pain felt in an area other than the site of odgin, such as pain near the shoulder associated with biliary disease . Psychogenic pain: is pain produced or caused by psychic or mental factors rather than organic factors Remember: Pain threshold refers to the lowest level ofpain a patient will detect.

All of the following antidepressant drugs are seratonin and norepinephrine reuptake inhibitors EXCEPT Venlafaxine (Effexor) Nortriptyline (Pamelor) Desipramine (Norpramin) Desvenlafaxine (Pristiq) Sertraline (Zoloft) Duloxetine(CymbaIta)

Sertraline (Zoloft) Five major categories of antidepressant drugs: . . . . Tricvclic antidepr€ssants: . Amitriptyline . . (E/avll) Doxeprn (Sinequan) Imipramine (Tof'anil) Serotonin and norepinephrine reuptake inhibitors: . . . Yenlafa:.l.ine (Elfexor) Nortriptyline (Pamelor) Desipramine (Norpramin) . . Desvenlafaxine (Pristiq) Duloxetine (Cymbalta) S€lective serotonin reuptake inhibitors: . . . Citaloqam Escitalopram (Celexa) . (Lexapro) . Fluoxetine (Prozac) Second generation miscellaneous: . (Wel I butrin) . . . Bupropror. Trazodone (Desyrel) Nefazodone (Serzone) Mirtazrpine (Remero n) . Monoamine oxidase inhibitors: . . . Phenelzine Q'{ardil) Tranylcypromine (P arn at e Isocarboxazide (Marylan) ) Paroxerine (Paxil) Sertraline (Zoloft)

Which antibiotic/antimicrobial is associated with the highest incidence of drug allergy? Ciprofloxacin (Cipro) Penicillin VK Clindamycin Metronidazole (Flagyl)

Penicillin VK Three groups ofallergic reactions to the penicillins: l. Actte (anaphylactic shock): occurs within 30 minutes. Characterized by urticaria, angioedema, bronchoconstriction, GI disturbances, and shock. Death can result in a short time if treatment is not instituted immediately (parenteral epinephrine). 2. Accelerated: occus 30 to 48 hours after. Manifestations include urticaria, pruritis, administration of wheezing, mild laryngeal edema, and local inflammatory reactions. Not life threaten- ing' 3. Delayed: occurs after 2 to 3 days. Approximately 80-90% ofall allergic reactions occurring with penicillin are ofthis t1pe. Manifested by skin rashes. Hypersensitivity reactions occur in up to l07o ofpatients receiving penicillin. Manifes- tations range from a mild rash to anaphylaxis. The rash may be urticarial, vesicular, bul- lous, or maculopapular. Rarely, thrombopenic purpura develops. lllote: A rash is the most common sign olan allergy to penicillin. R€member: The mechanism ofaction ofthe fluoroquinolones is inhibition of DNA gy- rase, an enzyme that is essential in the transcription, replication, and repair ofbacterial D\.{. Drugs in this class include ciprofloxacin (Cipro), norfloxacin Qtloroxin),levofloxacin (Levaquin), moxifloxacin (Avelox), and gemifloxacin (Factive). Fluoro- quinolones are active against many gram-positive organisms and gram-negative aerobes, including Moraxella catarrhalis, Haemophilus influenzae, E. coli, chlamydia, and My- coplasma pneumoniae. They are not active against Clostridium difficile. These drugs are rr ell tolerated, howevet the most common adverse effects are Gl (e.g., nausea, vomiting, diqrrheq, abdominal pain),CNS (e.g., headache, clizziness, confusion) and dermalologrc le.g., rash, pruritis).

Your patient with a history of breast cancer is undergoing chemotherapy with a drug called cisplatin (platinol). All are expected dverse effects EXCEPT Nausea and vomiting Hair loss (alopecia) Blood test abnormahties (low magnesium, low calcium, low potassium) Peptic ulcers Changes within the tissues of the oral cavity such as mucosititis Low white blood cells Low red blood cells (anemia) Xerostomia

Peptic ulcers Cisplatin is ar\ anti-cancer agent." ("antineoplastic") chemotherapy drug. It is classified as an Remember: Alkylating agents these agents alkylate DNA such that it cannot replicate. Popular agents include Cisplatin and Cyclophosphamide. Notes about cisplatin side ellects (a d dost chemotherapy drugs): . . . Most people do not experience all ofthe side effects The side effects are often predictable in terms oftheir onset and duration The side effects are almost always reversible and will go away after treatment is complete. . The side effects and their severity depend on how much of the drug is given. In other words, high doses may produce more severe side effects. The most common side effects ofchemotherapy are: . . . . . Anemia Fatigue Hair loss lncreased chance ofbruising, bleeding, and infection \ausea and vomiting Orher lesr common side effects include: . . . Appetite and weight changes \ene and muscle problems Dn and/or discolored skin . . . . . Low WBCS Xc.ostomia Mucosititis Kidney and bladder iritation "alkylating Sexual and fertility issues because ofeffects on reproductive organs \ot€: Mucosititis is a common reaction to cancer chemotherapy. [t is an inflammation ofthc mucous membranes. During chemotherapy and radiation therapy, mucosal tissues begin to desquamate and de- leiop into ulcerations. Thc mucosal int€grity is broken and is secondarily infected by oral flora. Palliarive treatment is indicated formucositis. The antineoplastics such as 5-fluorouracil (JFU), methotexate and doxorubicin are commonly associated with the development ofoml mucosihs.

Which term refers to a drug that binds to physiological receptors to result in specific cellular effects producing a predictable pharmacological response Pharmacologic agonist Pharmacologic antagonist

Pharmacologic agonist Drugs that bind to physiologic receptors and mimic the regulatory effects ofendogenous signaling compounds will produce a pharmacologic effect as a result of binding to the receptor. A drug that elicits a full response through this process is referred to as a phar- macologic agonist. A partial agonist is a drug which acts on the physiologic receptor but elicits an effect which is only partly as effective as an agonist drug. An antagonist is a drug which binds to the physiologic receptor but does not trigger an effect. When artagonist is present, the agonist drug cannot reach the receptor site to pro- duce an effect. 1. Competitiv€ antagonism occurs when a response can be achieved by increasing the dose of agonist in the presence ofantagonist. 2. Noncompetitive antagonism occurs when a response cannot be achieved with in- creasing doses of agonist in the presence ofantagonist.

All are CNS stimulants except: Caffeine Doxapram (Dopram) Phenobarbital Methylphenidate Phendimetrazine Strychine

Phenobarbital *** Phenobarbital is a barbiturate (sedative-hypnotic). CNS stimulants are a heterogenous group ofcompounds that produce various degrees of stimulation. Analeptic is a term that refers to a CNS stimulant which has the ability to overcome drug-induced respiratory depression and hypnosis. In the past the CNS stimu- lants were widely used therapeutically, but today they have only limited clinical applica- tion. Note: Their use for respiratory depression caused by an overdose ofCNS depressants is generally not safe or recommended. Analeptics and respiratory stimulants: doxapram and strychnine. These agents have limited use, but are occasionally indicated to stimulate respiration when a patient has pul- monary disease or to hasten recovery from a general anesthetic. Xanthines include caffeine, theophylline and theobromine. These stimulants improve mental alertness. reduce the urge to sleep and elevate the mood. Caffeine is the only approved OTC stimulant. Theophylline and theobromine are weaker CNS stimulants than calleine. Theophylline is the only xanthine important in the treatment ofasthma. It stirnulates the respiratory centers ofthe medulla and is able to cause bronchial dilation in patients $'ith asthma. Note: It has a low therapeutic index and its metabolism is affected b1 ser eral other drugs. Sl mpathomimetic amines include the amphetamines and other related agents nerh.rlphenidate, phendimetrazine, etc). They are potent CNS stimulants. They are used ro treat narcolepsy, obesity and attention deficit disorder.

Ester like local anesthetics are metabolized where Liver Lungs Plasma Kidneys

Plasma Ester-t!'pe local anesthetics all have an ester grouping within their chemical structure. An ester grouping is essentially a bridge or link containing the COOCH2-configuration. Amide-tJ*pe local anesthetics all have an amide grouping within their chemical structure. An amide grouping is essentially a bridge or link containing the - CONHCH2-configuration. Ester-type local anesthetics are mainly available as topical anesthetics and include benzocaine (which does not have an qmi o terminus end therefore cloes not become charged, thus it is poorl), soluble in water), tet:acaitrrc, and dibucaine. They are also available as medical anesthetic preparations such as propoxyc atne (Ravocaine). Ester-type local anesthetics are no longer available as dental anesthetic injectable preparations because of their relatively high incidence of allergy. Only the amides are presently available as dental injectable local anesthetic agents. These include lidocaine, mepivacaine, prilocaine, bupivacaine, and articaine. Ester-type local anesthetics are metabolized by the plasma enzyme plasma cholinesterase. Another name for this enzyme is pseudocholinesterase. This enzyme splits the ester linkage rvithin the chemical structure rendering the anesthetic ineffective. Procaine one of the original est€r-type anesthetics. When procaine is metabolized by plasma cholinesterase, a highly allergic compound called para aminobenzoic acid is formed. Many patients developed an allergy to PABA. Note: PABA can decrease the effectiveness of sulfonamides (antibiotics). )iote: Other chemicals that act like local anesthetics: . Diphenhydramine: H 1 antihistamine . T€trodotoxin: found in a number oftissues ofthe puffer or blowfish . Saxitoxin: is produced by certain stmins ofalgae Remember: Dyclonine hydrochloride is unusual in that it has a ketone linkage between the aromatic moiety and the rcst ofthe anesthetic molecule. It is used as a topical.

All are cholinesterase inhibitors EXCEPT Physostigmine Edrophonium Pyridostigmine Phentolamine Neostigmine

Phentolamine Phentolamine (and phenotybenzamine) are prototypes ofnon-selective alpha-adrenergic receptor blockeIS. Cholinesterase inhibitors act by blocking the enzyme (qce4,lcholi esterase) that degrades acetylcholine in the brain. This results in more acetylcholine in the s).naptic cleft and enhances cholinergic transmission. These drugs act as indirect agonists at both muscarinic and nicotinic sites. Examples of cholinesterase inhibitors include: . . Edrophonium: reversible; very short duration ofaction; used to reveffe cumre-type drugs Neostigmine: reversible; extended duration ofaction; used to reverse curare-type drugs and to treat myasthenia gravis . Physostigmine: reversible; short duration ofaction; used for glaucoma, and for an anti- dote for atropine . . . Pyridostigmine: reversible; extended duration ofaction; used to treat myasthenia gravis Tacrine: reversible; extended duration of action; used to Alzheimer's disease Donepezil: reversible; extended duration ofaction; used to Alzheimer's disease . Malathion: irreversible; long duration ofaction; used as an insecticide . Sarin: ineversible; long duration ofaction; used as a nerve gas Three cholinesterase inhibitors are commonly prescribed for Alzheimer's disease: . . . Donepezil (Aricept), approved to treat all stages of Alzheimer's disease Rivastigmine Gafantamine (Exelon), approved to treat mild to moderate Alzheimer's (Razadyne), approved to treat mild to moderate Alzheimer's \ote: The stimulation of the skeletal muscle by excess acetylcholine eventually results in muscle oaralvsis.

Which penicillin has the widest spectrum of antibacterial activity Penicillin VK Dicloxacillin Amoxicillin Piperacillin

Piperacillin Members ofthe penicillin family having the broadest spectrums ofaction are piperacillin and ticarcillin. In addition to being active against gram-positive cocci staphylococci, pneumococci), and gram-positive rods (bacillus and otfters), these agents are active against Pseudomonas, Proteus, Klebsiella, and Bacteroides These antibiotics are recommended for the treatment of urinary tract inf€ctions and other infections caused by susceptible gram-negative Pseudomonas species and Proteus specles. Penicillin VK has a relative limited spectrum of action against aerobic gram-positive cocci and anaerobes. Dicloxacillin has a similar spectrum as penicillin VK but is active against penicillinaseproducing staphylococcus. .{moxicillin is an aminopenicillin which has an extended spectrum ofaction which in- cludes not only aerobic gram-positive cocci and anaerobes, but some gram-negative bacilli such as Hemophilus, Proteus and Salmonella. Ampicillin also falls into the category of an aminopenicillin. Important: None ofthe penicillins are aclive against viruses, fungi, rickettsiae or other nonbacterial orsanisms.

Your patient is taking the following medications. Which two are used to reduce the signs and symptoms of rheumtoid artheritis Amlodipine (Norvasc) Enalapril (Vasotec) Piroxicam (Feldene) Prednisone Fosamax (Alendronate sodium)

Piroxicam (Feldene) Prednisone Piroxicam (Feldene) is a non-steroidal anti-inflammatory drug Qr'SlID) prostaglandin synthesis. Il is used to manage inflammatory disorders and used for the symptomatic treatment ofacute and chronic rheumatoid arthritis and osteoarthritis. Prednisone is a corticosteroid and has anti-inflammatory actions. It is used for the treatment ofa wide variety of inflammatory diseases including rheumatoid arthritis and os- teoarthritis. R€member: . . Common side effects ofNSAIDs such as piroxicam (Feldene) incl'tde: - Gastric irritation - Heart bum - Nausea Short-term side effects of corticosteroids such as - Insomnia - Indigestion - Arthralgia . Long-term side effects of corticosteroids include: - Edema (abdominal distension) - Psychological disturbances - Peptic ulcer - Osteoporosis - Muscle weakness prednisone include: that inlibits

All of the following nuerons are cholinergic EXCEPT Preganglionic sympathetic neurons Preganglionic parasympathetic neurons Postganglionic sympathetic neurons Postganglionic parasympathetic neurons

Postganglionic sympathetic neurons The autonomic neryous system f,4NS/ controls involtntary (smooth)rnuscle and gland activity. Bloodpressure, pulse, sweating, bladder, and rectal sphincter tone are regulated by the ANS. Function ofthe hean, eyes, uterus, gasEointestinal tract, from the salivaryglands to thc analsphincter, is govemed and maintained by the ANS. urinary bladder, and The ANS is a complcx sct ofncurons that originatc from thc CNS fipe.$callf the h'pothalanur; two antagonistic systems. Thc sympathetic nervous system (fron thoraco-hmbar outJlow) ar,d thc parasympathetic nervous system (ton cratlial-socrdl ou|lo\,t). The aulononlic nervous system has cholinergic neurons that secrete acetylcholine and adrenergic neurons that se- crctc norepinephrine. Prcganglionic ncurons ofboth divisions f.\:l rnla/, eli( and it is composcd of paraslnpathetit) are cholinergic, as arc the postganglionic curons ofthc parastmpathetic branch. Postganglionic neurons ofthc sympathctic branch arc usually adrenergic. Onc rXCl'PltON i s thc sympathetic postganglionic ncurons thal inncflate the swcat (thei, arc choli ergic = secrcte acelylcholine). Note: Adrcnergic blocking agents block thc cffcct of impulses transmittcd by thc adrenergic postganglionic ncurons ofthe sympathetic branch. Fibcrs ofthe somatic and autonomic nervous systcms, with thc structurcs inncrvatcd by the diffcrcnt libcrs and thc chcmical nrcdiators rcsponsible for lransmission at the various loci. Dashed lines indicatc postganglionic autonomic fibcrs.

Poisoning with an organophosphate cholinesterase inhibitor can be treated with: Edrophonium Carbachol Pralidoxime Nicotine

Pralidoxime Pralidoxime (Protopam) is a cholinesterase reactivator which is used as an antidote to reverse muscle paralysis resulting from organophoshate anticholinesterase pesticide poisoning. It is also used to reverse the effects of an overdosage of anticholinesterase agents used in the treatment ofmyasthenia gratts (i.e., pyridostiSmine and ambenomium). Symptoms of organophosphate poisoning include: excessive salivation, bronchocostriction, diarrhea and skeletal muscle iasciculations (twitching). Organophosphates are esters ofphosphoric acid and an organic alcohol that inhibit the enzyme cholinesterase. Examples include: . . . . . . . *** Isoflurophat€: used in the treatment ofglaucoma Malathion: a widely used insecticide Parathion: an insecticide Echothiophate: used in the treatment ofglaucoma Tabun: one ofthe most potent and toxic nerve gases Soman: nerve gas Sarin: nerve gas

Which of the folloing is the mode of action of tetracyclines Bacterial cell wall destruction Prevent protein synthesis in the bacterial cell Interfere with nucleic acid synthesis Cause mutations within bacterial DNA

Prevent protein synthesis in the bacterial cell The tetracyclines inhibit protein synthesis by binding to the 30 S subunit ofthe bacter- ial ribosome. The inhibition of this ribosomal function interferes with the attachment of the growing amino acid chain thus preventing complete formation of peptides from the ribosome. Since no peptides are formed, no proteins are formed. Since proteins are nec- essary for the bacterial cell to metabolically function, the lack thereof will cause a static state in which the bacterium becomes nrlnerable to phagocltosis by the body's imrnune S-v"SIem. Absorption of the tetracyclines from the GI tract is inhibited by divalent and trivalent cations such as CA--, Mg.-, Fe-., and Al**.. These ions form chelation products with the tetracyclines and thus prevent their absorption. This is why tetracyclines should not be given with milk and dairy products (contain Ca--), iron-containing vitamins (contain .Fet!, mineral supplements containing these irons, or antacids (contain Mg").

Which is often coadministered with antibiotics to dely the renal clearance of the antiobiotc Imipenem Probenecid Hydrochlorothiazide Aztreonam

Probenecid This elevates and prolongs the serum concentrations ofthe antibiotic when high tis- sue concentrations are necessary. The majority ofpenicillins are handled by the kidneys as organic acids and excreted by tubular excretion. Probenecid interferes with tubular handling of organic acids within the nephron. Drugs affected by probenecid include most cephalosporins and natural penicillins, and other betalactam-related antibiotics such as aztreonam and imipenem. Note: In some cases, probenecid administration can more than double the serum concen- tration olthe affected drug. The halflife is prolonged as well. lmportant: Nafcillin, oxacillin, cloxacillin and dicloxacillin are lipophilic and are ex- creted by biliary means. No combination with probenecid or dosage adjustment for renal dy'slunction is necessary for these penicillins. l. Imipenem is a beta-lactam antibiotic derived from thienamycin and is the \ot€ first drug to be classified as a carbapenem antibiotic. It is curently the drug of choice for infections due to Enterobacter and Pseudomonas aeruginosa' It is usually combined with cilastatin and is used to treat severe or reslsta.nt ln- fections, especially those that are nosocomial in origin. 2. Aztreonam is a parenteral synthetic betalactam antibiotic (classified monobactum). The spectrum is limited to aerobic gram-negative rods (i.e., Kebsiella, Pseuclonlonqs, and Serratia).Ithas no gram-positive or anaer- obic activity. lt is synergistic with aminoglycosides.

All the following are classified as antiepileptics EXCEPT one. Gabapentin (Neurontin) Carbamazepine (Tegretol) Phenytoin (Dilantin) Phenobarbital (luminal) Propafenone (Rythmol) Diazepam (Valium) Valproic acid (Depakene) Ethosuximide (Zarontin) Primidone (Mysoline)

Propafenone (Rythmol) is an antianhlthmic agcnt uscd to trcat both vcnlricuiar arrhythmias and supmvcntricular tschlcardias. Propsfenone /Rr1/rnol./ .\seizureisanakcrationinbchavior,functio,and,/orconsciousncssthatrcsultstiomanabnormalclcctricaldischargc ol ncurons rn thc brain. Epilepsy, or th€ tcrm scizurc disordcr, is uscd to dcscribc chronic unprovokcd rccurrcnt srizur.s. Serzurcs as classiflcd as eilher panial or generalized, bascd on horv thc abnormal brain activity bcgins. . . Partial seizures: when scizurcs appcar 1() rcsult from abnormal activit] in just onc pan ofthe brain. Generalized seizuresi seizures that sccm to in\rclve all ofthc brain. Fourrypcs ofgcncraiizcd seizurcs cxist; ab- sence tnlso Lalle.! pdit md1). myoclonic, ttonic and tonic-clonic scizurcs (ulso culled grand nal). F!.: xrr.ir antiepifeptic /.rnti@t1fllsant) dr$gs, thc cxact mechanism for rcducing scizDrc activity is not clcarly un- Jrrsii\id. All incrcasc thc threshold ofthc CNS to convulsivc stimuli or inhibit thc sprcad ofscizurc activiry Thc :nri,r!1:!'prlc drugs /,{fDrl can bc groupcd into thc following classes: lHldantoinsfPhen]toin/D//drrnrlisuscdinthctrcatmcntoftonicclonic(grandnal)sciz\\rcs.ltprolongsthc .ti.cri\c rcfBctory pcriod by blocking ncuronal sodium channcls. Phcnytoin-induccd gingival hyperplasis is common and ma] pa.tially or totally obscurc thc crcwns oftccth. Thc ratc ofdcvclopncnl oflhis condition can bc diminishcd by proper oral hygicnc. L GABA analogs: The cxacl mcchanism ofaction ofvalproic acid /Dcpdkdn.i and thc ncwcr GABA analogs is not clcarly undcrs@od. Al1 ofthcm incrcasc thc actions ofGABA, in inhibilory ncurotransmittcr Thcy may inhibit thc voltagc-dcpcndent sodium channcl, thcrcby stabilizjng thc ncuronal membranes. [xamplcs oflhc ncwcr GABA analogs include: gabapcntin (Neuro inc (Lan ida l) , prc9abal;n lLlrirc ) , tin), divalprocx sodium t;a$abirc lcabitril) , fDryakote), and ropiftr..,arc (Topumar) . 3. Succinimides: Ethosu\imlde (Zaro tifi)t \tscd in thc trcatmcnt of abscncc scizures. 4. Barbituratesi Phenob^rbital fclbamzlc (Felbaror, lamotrig- (Lut inal.)wo*s by inhibiting dcpolarization ofncurons by bindinglo the GABA rcccptors, which cnhanccs lhc transmission ofchloridc ions. Primidone (t|r.!o/ir?i is mctabolizcd lo phcnobar- phcnylcthylmalonanide /PtM,4/, which havc anticonvulsant activity. 5. Benzodiazepines; work similarly to barbituratcs, but thcy also incrcasc thc numbcr ofchloridc channcls whilc bital and facilitating thc transmission ofchloridc ions. Examplcs includc diazepam lomzcpam /,lrl|rl,r. Not€: Di^zeprm (Ilalium) (lhlium), clo azcpam is uscd for status cpilcpticus and in cmcrgcncy trcatmcnt ofseizures. May cdusc drcwsrncss. dr7lincss. dnd ata\id. 6. Misceffaneous: oxcarbazcpi^c mazepine is uscd as (hilepttrl),lcvctiftccl^m (Kepptu), and c rbgm zepine prophylaxis for partial scizurcs ilh complcx symptomatology including psychomotor tcmporal lobc scizurcs. It is also uscd to treat tonicclonic scizurcs /grdrd nd, and trigcminal ncuralgia. It rarcly causcs aplastic ancmia, (Klonopnt), (hegreol).C^rba- and and

All of the following are alpha adrenergic receptor blockers EXCEPT Doxazosin (Cardura) Phenoxybenzamine hydrochloride (Dibenzyline) Phentolanine hydrochloride (Regitine) Prazosin (Minipress) Propranolol (Inderal) Terazosrn (Hytrin)

Propranolol (Inderal) Propranolol is a beta-blocker (speclicall- a non-selective beta-blocker) Alpha-blockers are medications that act by competitively inhibiting the action of catecholamines at the alpha receptor site. They act on blood vessels, causing them to relax. Aipha-blockers are commonly used to reduce high blood pressure and to treat an enlarged prostate. There are two types of alpha-blockers, which are classified according to the alpha receplor that they block (alphal . or alpha). Selective alpha antagonists: only block alphal-recepto$ and are more commonly used ro rreat cardiac conditions (h)pertension) and benign prostatic hyperplasia. . \on-selective alpha antagonists: block both types of receptors and are generally not (rqpid heart beat) and palpitations (pounding he.qrt beat). They are used in the presurgical used for cardiac conditions because blocking both recepto$ can ca\se tachycardia management of pheochromocytoma and sometimes in treating Raynaud's phenomenon. Some examples of alpha-blockers: . Doxazosin selective alpha,-blocker. Among the alphal-blockers it is the prefered agent for hypenension due to a longer duration of action. . . Terazosin selective alphar-blocker. Used to treat benign prostatic hypertrophy. Prazosin - selectiv€ alpha'-blocker lt is rarely used to treat hypertension due to unwanted adve$e rcactions. . Phenoxybenzamine and phentolamine both are nonselective alphal and alpha2- blockers. They are used in the presurgical management of pheochromocytoma. *** Aside from hlpotension, which is a major effect ofalpha-blockade, alpha-blockers cause relatively few adverse reactions.

The brief duration of general anesthetic action of an ultra-short-acting barbiturate is due to what factor? Rapid rate of metabolism in the liver Low lipid solubility, resulting in a minimal concentration in the brain High degree of binding to plasma proteins Rapid rate of redistribution from the brain to peripheral tissues Slow rate of excretion by the kidneys

Rapid rate of redistribution from the brain to peripheral tissues Remember; These agents will maintain anesthesia only while in the brain. Because of their high lipid solubility, these agents will rapidly leave the brain for other tissues; thus the patient wakes up within a few minutes of administration. Eramples of ultra-short-acting barbiturates include thiopental (Pentothal) and metho- he\ital (.B/?l,tal/. These agents are administered by intravenous injection. Contraindications to the use of ultra-short-acting barbiturates for general anesthesia: . . . . Porphyria Lir er dysfunction Emphysema (thet- are metabolized in the liver) Previous addiction to sedative hypnotic drugs Remember: These drugs have the ability to produce dependence.

NSAIDs reduce the production of ? associated with pain and inflammation. Leukotrienes Cytokines Prostaglandins Interlerons

Prostaglandins inhibit the cyclooxygenase step ofthe arachidonic acid cascade and thus inhibit thc activity ofprostaglandin synthetase. Prostaglandins are a Non-steroidal anti-inflammatory drugs 0y'S,4/Dl, hormonc-like substances that mediate a range of physiological functions, such as metabolism and nen e transmlsslon. \SAIDs have analgesic, antipyretic, and anti-inflammatory properties (similat to aspitin). They are used for pain control, afihritis, and painful menstruation. Advcrsc reactions include GI upset @os- sible ulcers), and prolongation of bleeding irme (reduction in plqtelel aggregatiotl). Contraindica- tions to ihe use ofNSAlDs are impaired renal function, pregnancy, and GI disease Eramples offSAlDs: . Proprionic acid derivatives: . . Flurbiprofen (,lnsaid) Ibuprofen (Motrin, . \aproxen Advil) (Naprosvn) . .{cetic acid derivatives: . . . . . , . Indomethicin Etodolac llndocll (Lodine) . Sullndac (Clinoril) Alkanoic acid derivatives: . Nabumetone (Relqfen) Fenamic acid derivatives: . . Meclofenamatc (M ec lomen Mefenamic acid Oxicams . . Piroxicam Meloxicam lPorsle4 (Feldene) (Mobic) Cox-2 inhibitors . Celecoxib (Celebrcx) ) 'Oxaprczin . (Daypro) Ketoqofen (Orudis) 'Fenoprofen . (Nalfon) Dicloferac sodirm (Votaren) Ketorolac (Toradol) ' 1r.r/cers) Naproxen sodium (Anaprox, Aleve) group of

Which substance is an enzyme formed in the kidney and released into the bloodstream where it has an important role in the formation of angiotensin Plasmin Lysozyme Renin Heparin

Renin is proteolytic enz)'rne produced by and stored in the juxtaglomerular apparatus that surrounds each arteriole as it enters a glomerulus. Renin acts on the precursor substance angiotensinogen, which is manufachrred by the liver and is present in the blood. Renin converts angiotensinogen to angiotensin I. In turn, angiotensin I is converted to angiotensin II by a converting enzyme associated with the walls ofcapillaries, particularly in the lungs. Important: "The converting enzyme that converts angiotensin I to angiotensin II is known as angiotensin converting enzyme or ACE." Angiotensin II is a potent vasopr€ssor. It not only increases total peripheral resistance but, by stimulating aldosterone release, leads to an increase in plasma volume, venous retum, stroke voh.rme, and ultimately an increase in cardiac output. \ote: Aliskiren (Tehuma) is a renin inhibitor used to treat hypertension. This is the first ofa new class of drugs and little information is available. It is used alone or in combination with other agents for the treatment ofhypertension.

The most common adverse effect associated with the benzodiazepines include all EXCEPT CNS depression (drowsiness and sedation) GI disturbances (nausea, vomiting and diarrhea) Confusion Respiratory depression Disorientation Ataxia

Respiratory depression The benzodiazepines have clinically useful antianxicty, scdative-hypnotic, anticonvulsant and skele- ial muscle relaxanl propcrties. They exert their main effect on central GABA-nergic neurons. With nomral dosrng, thc benzodiazepines have little effect on respiratory systcms in healthy individuals. The benzodiazepines (especially diazepam, lorazepam and midazolan) arc important adjuncts in rhe practice of ancsthesiology. They may be used as preoperative scdatives and induction agents, as \\ cll as supplcrncntal agents for the maintenance ofanesthesia. Important Thc benzodiazepines, barbiturates and narcotic analgesics all produce sedation and have the ability to produce dependence. The pharmacokinetics ofindividual benzodiazepines difler, and thcrelbre there is a wide range in speed of onset and duration of action anrong these compounds. Following oral administration, most ofthe benzodiazepines are rapidly absorbed and highly bound to plasma protein. ln general, most of the metabolitcs are conjugated with glucuronic acid and excreted in the urine and f-eces Note: cl,-Hydroxylation is a rapid route ofmetabolism that is unique to triazolam, midazolaln, and alprazolam. This accounts for the vcry mpid metabolism and short sedative actions ofthese drugs. 1. The term tranquilizer rcfers to a drug that promotes tranquility by calming, soothing, quieting orpacifying without sodating or depressant e{lccts. Antipsychotic agcnts arc considered to be major tranquilizers and antianxigty agents fuenzodiazepites) considercd to be minor tranquilizers. 2. For the treatment of insomnia due to anxiety the benzodiazepines llurazepam, temazepam and triazolarn are usually prescribed. 3. lmportant: Flumazenil (Mazicon), a bcnzodiazepine antagonist, may bc used to rc- verse thc residual effects ofbenzodiazepines in the evcnt ofan overdose. 4. tfthe benzodiazepines are i.effective in helping with insomn ia, zolpidem (Ambien), eszopiclone (L nesta), or zaleplon agents (called GABA-BZ Agonists). (Sonata) are ot\en weful. Thesc are anti-insomnia

Which adverse effect is associated with opioid analgesics and not non-narcotic pain relievers? Allergic response Nausea Vomiting Respiratory depression

Respiratory depression The most significant and well-known adverse reaction to opioids is respiratory depr€s- sion. Death secondary to opioid overdose is nearly always due to respiratory depression. When opioids are appropriately used, the risk of severe respiratory depression is gener- ally small as tolerance rapidly develops to this effect.

The medical uses of a drug having adrenergic agonist actions would include which of the following? a. Reverse an anaphylactic reaction b. Treat hypertension c. Prevent angina pectoris d. Reduce anxiety e. All of the above

Reverse an anaphylactic reaction The prototypical adrenergic agonist agent is epinephrine, Epinephrine stimulates both beta adrenergic receptors and alpha adrenergic receptors within the sympathetic division ofthe autonomic nervous system. Anaphylaxis is characterized by rapid, extreme reduc- tion in blood pressure and bronchospasms. Epinephrine, upon injectable administration, will rapidly reverse the hypotension by causing vasoconstriction via the alphal recep- tor stimulation; it will dilate the bronchial tubes via beta2 receptor stimulation; it will in- crease cardiac output via betar receptor stimulation on the cardiac muscle. Epinephrine is: . Ineffective in treating hypertension because of its alpha receptor stimulatory actions on the vasculature which could cause an even further elevation ofblood pressure . Contraindicated in angina conditions because its cardiostimulatory effects would aggravate this condition . Will not reduce arlxiety but will likely increase anxiety since it has central nervous system stimulatory effects

All of the following drugs are useful for treating what common medical condition? Prednisone Gold injections Methotrexate Nabumetone (Relafen) Piroxicam (Feldene)

Rheumatoid arthritis Rheumatoid arthritis (RA) is a chronic inflammatory disease ofjoints that results in joint pain, swelling, and destruction. RA is characterized by chronic inflammation ofthe synovium, which lines the joint. With disease progression, there is the accumulation of prostaglandins, leukotrienes and other mediators in the inflammatory changes and tissue destruction in the synovial lining. How these drugs work in treating rheumatoid artkitis: . . . . . Prednisone: decreases the inflammatory response Gold injections: may decrease prostaglandin production Methotrexate: unknown, but may affect immune function Nabumetone fRela/en): an NSAID that inhibits prostaglandin synthesis Piroxicam (Feldene): ar NSAID that inlibits prostaglandin synthests \ote: All of the above drugs except gold injections are also useful in the treatment of osteoarthritis (OA). OAis characterized by progressive loss of articular cartilage. This may be the result of excessive loads on the joint or other factors. Agents useful in treat ing OA provide an analgesic and anti-inflarnrnatory action to reduce pain within thejoint.

Antimetbolites are cell cycle specific drugs acting on which stage of the cell cycle G0 phase or resting phase of the cell cycle S phase of the cell cycle G1 phase of the cell cycle G2 phase of the cell cycle

S phase of the cell cycle he S phase ofthe cell cycle = DNA synthesis Antimetabolites are one ofthe oldest and most important classes ofantineoplastic agents. They attack the cells in the S phase of the reproduction cycle by interfering with the biosynthesis ofthe purine and pyrimidine bases. Important: Alopecia (hair loss) occtrs with the administration ofmost chemotherapeu- tic agents one to two weeks after treahrent. Other common side effects include GI upset, increased incidence ofinfection tissue. fespecially Candidiasis), and degeneration of lymphatic Remember: Most chemotherapy drugs have been shown to be teratogenic in humans and should be avoided by pregnant women.

Which schedule of drugs would include a drug that is considered to have a strong potential for abuse or addiction but which hasa legitimate medical use Schedule I Schedule II Schedule III Schedule IV Schedule V

Schedule II Schedule I - a category ofdrugs not considered legitimate for medicinal use. Among the substances so classified by the DEA are mescaline, LSD, heroin, and marijuana. Special licensing procedures must be followed to use these or other Schedule I substances. Schedule II a category ofdrugs considered to have a strong potential for abuse or ad- diction. but which have legitimate medical use. Among the substances so classified by the DEA are Morphine, Cocaine, Pentobarbital, Oxycodone, Methadone, and straight Codeine. Schedule III a category ofdrugs considered to have less potential for abuse or addiction than Schedule I or II drugs. Among the substances so classified by the DEA are var- - acetaminophen and codeine ious analgesic combination compounds containing codeine - \'lenol fi.e., #3) and various analgesic combination compounds containing hy- Lorcet). Schedule IV drocodone (i.e., hydrocodone and acetaminophen - -l'icodin; a category of drugs that have less pot€ntial for abuse or addiction than those of Schedules I to III. Among the substances so classified by the DEA are diazepam (Valiurn),lorazepam (Ativan), triazolam (Halcion), alprazolam (Xanax), drate. Schedule V a category of drugs that have a small potential for abuse or addiction. Among the substances so classified by the DEA are many commonly prescribed medications that contain a small amount ofCodeine. Note: Schedule II and III must have a written prescription signed by the health profes- sional (aws vary stqte to state). The FDA determines which drugs are to be sold by prescription only. The prescription must have the address ofthe patient and dentist as well from as the DEA number olthe dentist. and chloral hy-

The vasoconstrictor epinephrine in local anesthetic injections must be used cautiously in patients taking all of the following antidepressant drugs EXCEPT one in order to avoid transient and significant increases in blood pressure. Tricyclic antidepressants (i.e., Elavil) Selective serotonin reuptake inhibitors (i.e., Prozac and Paxil) Serotonin and norepinephrine reuptake inhibitors (i.e., Effexor and Cymbalta)

Selective serotonin reuptake inhibitors (i.e., Prozac and Paxil) These trvo categories (tt'iclclic qntidepressants snd serotonin and norepinephrine teup- rake inhihitors) of antidepressant drugs significantly increas€ norepinephrine levels in tis- sues. ln rhe presence of a vasoconstrictor administered via a local anesthetic injection, the patient could experience significant elevation of blood pressure due to the vasopressor actions of the combination. \ote: The selective serotonin reuptake inhibitors epinephrine, fi.e., Prozac and Paxil) have no such effect on norepinephrine in tissues and interaction with a vasoconstrictor like epineph- rine is not an issue.

Buspirone (Buspar) is a partial agonist at a specific GABA (gamma-aminobutyric acid) receptor Norepinephrine receptor Dopamine receptor Serotonin receptor

Serotonin receptor Buspirone (BuSpar) is an orally administered antianxiety agent with a short halfJife (2- 4 hours). It is not chemically related to the benzodiazepines, or any other anxiolyic agent. Buspirone also differs from other antianxiety agents in that it does not possess anticonvulsant or muscle relaxant properties, does not impair psychomotor function and does not cause sedation (lacks CNS depressant activiry) or physical dependence. The exact mechanism ofaction olbuspirone is unknown. However, it has a higher affin- ity for serotonin receptors in the CNS and a lesser aflinity for the benzodiazepine- GABA receptors. Important: The antianxiety effect is achieved via a partial agonist effect on CNS sero- tonin 5 -HTla receptors that occurs without affecting the benzodiazepine receptors or causing CNS depression. Remember: GABA-BZ agonists have a chemical struchrre that is dissimilar frorn those ofthe benzodiazepines and on the sedative hypnotics. These agents appear to act through the potentiation ofGABA on benzodiazepine r€ceptors, especially the omega-l subunit. They are used primarily for insomlia. They have exhibited some anxioll4ic action but they have little effect on skeletal muscle or seizure thresholds. They appear to have min- imal disruptive action on the normal sleep cycle, thus preserving deep sleep. Note: They have shown no potential for causing addiction. Examples include Zolpidem (l mbien), es- zopiclone (Lunesta), and zaleplon (Sonata). -fhese drugs have short halfliv€s /o 2 how s).

Insulin regular also known as Humulin R, is considered an Short-acting insulin with a duration of 6-8 hours Long-acting insulin with a duralion of 20-24 hours Intermediate-acting insulin with a duration of l0-16 hours Rapid-acting insulin with a duration of 3-4 hours

Short-acting insulin with a duration of 6-8 hours Insulin preparations mimic the activity ofendogenous insulin, which is required for the proper utilization ofglucose in normal metabolism. They are used in type 1 diabetes and in type 2 diabetes which cannot be controlled completely by the oral antidiabetic drugs or by diet alone. lmportant: Hypoglycemia (low blootl sagay' is the most serious complication ofinsulin thempy. Symptoms include: sweating, weakness, confusion, slurred speech and blurred vi- sion. Administration ofa concentrated glucose source will relieve mild hypoglycemia.

The majority of drugs penetrate biomembranes by ? through membrane phospholipids Actlve transport Facilitated diffusion Filtration Simple diffusion

Simple diffusion The mechanism of drug transfer across biological membranes is by: L Passive transfer . -is essential to various processes of metaboltsm Simple diffusion: Iipid-soluble substances move across the lipoprotein membrane by.' this process. The majority of drugs penetrate biomembranes by this process through membrane phospholipids. The amount of drug dissolving in the membrane at any time is directly proportional to the concentration gradient and its degree of lipid solubility . (Note: Only nonionized drugs are soluble in lipid). Filtration: water-soluble molecules small enough to pass through membrane channels may be carried through the pores by the bulk flow of water. Drugs ofmolecu- lar u eights of 60,000 or less can "filter" through capillary membranes. 3. Specialized transport . Active transport: lipid-insoluble substances (for example gluco.re) are shuttled across plasma membranes by forming complexes with specific membrane constituents called carriers. These carrier molecules within a cell fumish energy for ransportation ofthe drug to regions ofhigher concentration. Facilitated diffusion is the term given to carrier-based transfer when the driving force is simply the concentration difference ofthe drug across the membrane. Note: Osmosis is the movement of a pure solvent, as water, througl't a semipermeable membrane from a solution that has a lower solute concentration to one that has a higher concentration. The membrane is impermeable to the solute but is permeable to the sol- vent. Remember: The physiochemical properties ofdrugs that influence their passage across biologic membranes are: lipid solubility, degree of ionization, and molecular size and shaoe.

Local anesthetics depress which of the following nerve fibers first Large myelinated fibers Small unmyelinated fibers Small myelinated fibers Large unmyelinated fibers

Small unmyelinated fibers These conduct pain and temperature Important: They depress the large myelinated fibers last. The small nerves have the greater surface-volume ratio (this accounts for the rapid onset ofaction). Clinically the general order of loss of function is as follows: l. Pain 2. Temperature 3. Touch 4. Proprioception 5. Skeletal muscle tone Note: Pain threshold refers to the lowest level ofnain a natient will detect. Remember: Local anesthetics reversibly block nerve impulse conduction and produce the reversible loss of sensation at their administration site. They do not produce a loss of consciousness. They appear to become incorporated within the nerve membrane or to bind to specific membrane sodium ion channels, restricting sodium pemeability rn re- sponse to partial depolarization.

Growth hormone is called Dopamine Aldosterone Vasopressin Somatotropin

Somatotropin Grorlth hormone (GI{) is produced by, and secreted from the anterior pituitary gland. Basic Metabolic eff€cts of growth hormone: . Increased rate ofprotein synthesis in all cells ofthe body . . Decreased rate ofcarbohydrate utilization thoughout the body Increased mobilization offats and use of fat for energy Human growth hormone is prepared commercially and used as replacement therapy in patients with growth hormone deficiency. The commercial preparation is prepared as the purified polypeptide hormone of recombinant DNA origin with the same amino acid sequence as that produced by the pituitary gland. Human growth hormone is indicated in children for the treatment of growth failure due to lack ofadequate endogenous growth hormone secretion. It has been used in adults who have a growth hormone deficiency as a result ofpituitary disease. Note: Human growth hormone is adminislered as subcutaneous injection or intramuscu- Iar injection, usually tbree times per week.

Which of the following is a pharmacologic antagonist of aldosterone in the collecting tubule? Mannitol (Osmitrol) Glycerin (Glyrol) Spirinolactone (Aldactone) Urea (Ureaphil)

Spirinolactone (Aldactone) Potassium-sparing diuretics result in increased sodium and decreased polassium concentrations at the end ofthe distal convoluted tubules. There are two cat€gories ofpotas- sium-sparing diuretics: l. Antagonist of aldosterone in the collecting tubules: . Spironolactone (Aldactone) Note: Spironolactone has been found to be effective in the treatment of primary aldosteronism. It may be useiul in the treatment of heart failure because hyperal- dosteronism is comfironly seen in this condition. 2. Block the sodium channels in the collecting tubules: . *** Triamterene (Dyrenium) . Amiloride (Midamor) The most impodant toxic effect of potassium-sparing diuretics is hyperkalemia. Osmotic diuretics are highly filtered by the glomerulus and exert a solute-induced di- uresis in the proximal tubule. They are used to reduce excess edema associated with neurosurgery or traura to the CNS. Examples include: mannitol, glycerin, and urea. Remember: Carbonic anhydrase inhibitors fi.e., lcetazolamide) arerelatively weak di- uretics because ofthe ability of more distal sites in the loop ofHenle to increase theirre- absorption of sodium. Note: Acetazolamide is used to prevent and reduce the symptoms of altitude sickness.

Which herbal supplement below is known to be somewhat effective in treating mild forms of mental depression? Ginkgo biloba St Johns Wort Garlic Echinacea Ginseng

St Johns Wort Some studies have shown the St Johns Wort can inhibit the re-uptake of serotonin at neurcnal synapses resulting in the elevation of serctonin within the CNS. This effect is similar to those antide- pressants u'ithin the flnoxetine (Plozac) fatrily. Of all the herbal supplements available to the consumer, St Johns Wort is associated with use as a mild antidepressant. Ginkgo biloba is a herbal supplement used as a peripheral anery vasodilator Garlic is a herbal supplement used to lower cholesterol and to inhibit platelet aggregation resulting in a decrease in blood clotting. Echinacea is a herbal supplement and immune stimulator used to minimize the severity ofthe common cold and seasonal flu. Ginseng is a herbal supplement used to stimulate the immune system. Chamomile has a long history of use in Europe for digestive ailments. The active constiuents of chamomile have anti-inflammatory properties, and ease spasm and discomfort in the digcstive tract. \ote: Chamomile contains coumarin, a natumlly-occurring compound with anticoagulant or blood- rhinning effects. It should not be combined with warfarin or other anticogulants- S.li palmetto has long been used in Europe to treat an enlarged prcstate or benign prostatic hyperpla- sia (BPH).

Which stage of general anesthesia begins with unconsciousness? Stage I Stage II Stage III Stage IV

Stage II Four stages ofgeneral anesthesia: . . Stage I: Amnesia/analg€sia Stage II: Delerium: begins with unconsciousness and ends with loss of€yelid rellex, purposcless ments and h)per-reaction, dilated pupils, reflex vomiting, tachycardia, and hlTe.tension . . Stage llli Surgical rnesthesia: four planes with progressive loss ofreflexes and n1uscle control Stage IV: Medullary paralysis: cessation ofrcspiration, ending with death without proper trearment Notei These four stages ofanesthesia apply to the inhalaflts and not to the intravenous general anesthetics. Agents used lbr generrl anesthesia: . Inhalafion agentsi volatile halogen-containing Iiquids - Halothane: poses a risk with epinephrine; associated with hepatitis; poor skeletal muscle rclaxant. llowever, it is one ofthe mor€ widely used anesthetics for general surgical anesthesia. - Desrlurane, sevoflurane, isoflurane and enflurane: pose Jess risk with epinephrine; not associated wirh hepatitis; good skeletal muscle relaxants. \ote: Nitrous oxide fN,O) is not considered a gencral ancsthetic since hlpoxic levels are required to producc anesthesia. It is considcred a sedative. It is used alone to produce sedation or in combination with the above agenrs to supplement the anesthetic response. lt is a gas at room temperaturc and .Intrar'enous . agents: f ltrashort-acting barbiturates: thiop€ntal (Pentothal) and methohexit^l (Brei[al) . . Benzodiazepines: di^zeptm (yaliun), tuidazol^m (Versed.), ^nd pressure. lor^rcp^m (Ativan) \eurolep(ic-opioid combinations which are called neuroroleptanalgesics combine fentanyl trd d.operidol . Other agents: (a phenothiazine) . Propolol (Diprivan), . a short-acti g hypnotic agent; vasodilator Ketamine (Ketalar). is considcrcd a dissociative anesthetic. It blocks N-methyl-D-asparate A,VDA) (glutanale) rccpetors. Nofer In some circumstances, ketamine has been known to produce illusions or hallucinations that are enhanced by environmental stinruli {rpon eme.gence liom anes- thesia. Diaz€pam is giv€n with ketamine to avoid this. . Etomidate (Anitlate): is a carboxylated imidazole derivative \\.hich acts similar to the ultrashon- acting barbituratcs.

The protoype depolarizing neuromuscular blocking agent is Tubocurarine (Curare) Mivacurium (Mivacron) Succinylcholine (Anectine) Vecuronium (Norcuron) Doxacurium (Nuromax) Pancuronium (Pavulon) Atracuium (Tracrium) Cisatracurium (Nimbex) Rocuronium (Zemuron)

Succinylcholine (Anectine) Neuromuscular blocking drugs are important for producing complete skelctal muscle relaxation and facilitate endotracheal intubation, as an adjunct to surgical anesthesia. These agcnts interact with nico- linic receptors at ihe skeletal neuromuscularjunction. Thcrc are t$o classes ofneuromuscular blockers: l. \ondepolarizing: These agents competitively compete with acetylcholine at the nicotinic recep- rrrr. Thesc agents bind to nicotinic cholinergic receptors and prevent acetylcholine from stimulating moror nenes. resulting in muscle paralysis. The prototlTre nondepolarizing agent is tubocurare |rrrer Other agents include mivacurium, vecuronium, doxacurium, pancuronium. atracunum, crsatracurium and rocuronium. Note: Neostigmine or pyridostigmine (which are cholineslerase i4- hibitots) can reverse the blockade ofthese agents. :. Depol^rhing (non-(onpelilire): S:'lccinylcholine (Afiecttue) is thc only member ofthis group used rn the Unired Statcs. It acts like a nicotinic agonist and depolarizes (dese/tsilizes) theneuromuscular end pla1e. It binds to the ACh receptor and stimulates depolarization causing initial excitation fol- lo\1ed bt- block of n eurotransmis sion and muscle paralysis. Important: Succinylcholine should be used with caution in patients with low Ievels of pseudocholinesterase, which breaks down succinylcholinc. Respiratory failure may result. *** Succinylcholine may cause muscarinic responses such as bradycardia and incrcased glandular se- cretions, Very important: The major danger ofall ofthese ncuromuscular blocking drugs is too much paral- ysis (the patient cdnnol hrealhe). 1 Dantrolene is an skeletal muscle relaxing agent that acts within the skeletal muscle fiber rather than on the neuromuscularjunction fdoes not block the icolinic receptors).lt inhibits the depolarization-induced release of Car- from the sarcoplasmic reticulum The principal therapeutic applications ofdanffolene are for the re lief of spasticities associated with upper motor ncuron disorderc (i.e., slroke, cerebral palsv, and muhiple sclercsis) and for the pro- phylaxis and treatment of malignant hyperthermia. 2. Botulinum toxin A fBolo-rl acts on the motor nervc terminal to prcvent the release of ACh. It is used in ophthalmology to relax the extraocular muscles, and for muscle dystonias as well as to remove wrinkles.

All are skeletal muscle spasmolytic drugs EXCEPT Methocarbamol (Robaxin) Cyclobenzaprine (Flexeril) Baclofer (Lioresal) Succinylcholine (Anectine) Carisoprodol (Soma)

Succinylcholine (Anectine) Succinylcholine is the prototype depolarizing neuromuscular blocking drug. Spasmolytic drugs (skeletal muscle relax4,l/t are agents that relieve muscle spasms without paralysis. They act in the CNS or in the skeletal muscle cell mther than at the neuromuscu- lar end plate. These drugs are used in certain chronic diseases ofthe CNS (i.e., multiple sclerosis, cerebtal palsy, cerebrovascular accidents) that are associated with painful muscle spasms. By reducing the spasms there is a reduction in pain and improved mobility for the pa- tient. Drugs used for chronic muscle spasm: . Baclofen (Lioresal): is a derivative ofGABA; its site ofaction in reducing muscle spasms is the spinal cord. It stimulates CABABreceptors that are linled to the G protein. Gt, re- sulting in an increase in K conductance and a decrease in Ca,' conductance. Used is the treatment of mriltiple sclerosis and other spinal cord diseases. All ofthe other skeletal mus- cle relaxants do not bind to the GABABrecepto$: Diazepam (Valium) and tizanidine (Za . aflex) also act rn the spinal cord and are effective muscle relaxants. Carisoprodol fsoranJ: is used in the treatment ofmuscle spasms and pain associated with acute temporomandibularjoint pain. lts precise mechanism ofaction is not clear but many effects have been ascribed to its central deDressive action. Drugs Lrsed for acute muscle spasm: . C!clobenzaprine (Flexeril): relieves muscle spasm through a central action, possibly at the brain stem level. lt is used to relieve acute, painful musculoskeletal conditions. It is not efllctir e for muscle spasm secondary to cerebral or spinal cord disease. . \lethocarbamol (RobcLrin): is a centrally acting muscle relaxant that is used to relieve acute. painful musculoskeletal conditions and in the management oftetanus. \ote: *** Quinine is widely used for the effective relief of noctumal leg cramps.

What needs to be written on a prescription

Superscription = Patient's name, address, age, date Inscription = Name of drug and the strength of the drug Subscription = Directions to the pharrnacist (dosage form and amount to be given - Disp.) Transcription = or signa, Directions to the patient (SigJ Signature = Signature of person prescribing medication must appear

A drug that reduces the actions of the sympathetic nervous system is called Sympathomimetic Sympatholytic Sympathetic amine Adrenergic agent (Beta-blocker)

Sympatholytic (aka adrenergic blocking agebt or adrenergic receptor blocker) Four kinds ofadrenergic receptor blockers: L Beta-adreDergic blockers: fNorc: . Nonselectir€: blocks both betal- and beta2-receptors . Propmnolol . Nadolol . Betal-s€lective: . . Atenolol Metoprolol 2. Alpha-adrenergic blockers . 'Carteolol ' . . . P€nbutolol Esmolol Bisoprolol Thel are allused to trcsl hlpertension) Sotalol . Timolol . Betaxolol 'Acebutolol \onselective: blocks both alphat- and alpha2- receptors ' . Phentolaminc Phenoxybenzamine 'AlPhat-selective: . . Prazosin Terazosin . . Doxazosin Tamsulosin i. Centrally acting Nlpha2-agonists: Act through stimulation ofceDtral inhibitory alpha'-adrenergic rc- ceprors.They inhibit sympathetic cardioaccelemtor and vasoconskictor centers. Stimulation ofalpha-adrenergic rcceptors in the brainstem results in rcduccd sympathetic outflow from thc CNS 'Clonidine . Methyldopa 4. \euronal depleting agents: cncompasses awide variety ofdrugs having different mechanisms ofaction. depletion ofmediator in thc nouron Ierminal or an inability ofth€ medialor to be released from the terminal. whatever their specific mechanism, the result is usually the same . Reserpine - depletes granules -a containing NE in nerve endings, releases NE . . Guanethidine - blocks adrcnergic nerve endings by a series ofactions Metyrosine - inhibits tytosine hydroxylase; used to treat pheochromocytoma r-ote; Canedilol and labetalol ar€ nonselective bet|-blockers thai also block alphal-receptors. Tley ate used for healt failure.

Inhaled ammonia is the drug of choice for acting against Anaphylaxis Heart attack Syncope Urticaria

Syncope Inlaled ammonia irritates trigeminal nerve sensory endings, with a resulting reflex stim- ulation of medullary respiratory and vasomotor centers. An aromatic amrnonia vaporole is crushed betrveen the fingers and held near the patient's nose. Note: The administration oforlgen rrrll aid in combating tissue anoxia. The sl mptoms of syncope include beads ofsweat on the upper lip, a weak thready pulse, cold clammy skin, pallor and a dizzy feeling. The loss of normal vasomotor tonus pro- duces pooling of blood peripherally so that the normal blood volume becomes insuffi- cient. Placing the patient in a supine position and elevating th€ feet gives the patient a transf'usion ofwhole blood by utilizing the forces of gravity. Note: The head should not be more than about 10 degrees lower than the rest ofthe body. Tl pes of syncope: . Vasor agal . \eurogenic . . C)nhostatic -f ) I Trear uirh high-flowing 1007o oxygen Hyperventilation syndrome Oxygen is not indicated Note: 100% oxygen is contraindicated for a person who suffers from chronic obstruc- tive pulmonary disease (COPD).

The muscarinic effects of cholinergic agonists include all EXCEPT Miosis Flushing Bronchoconstriction Increased GI motility Increased urination Tachycardia Salivation Sweating

Tachycardia Bradlcardia is a muscarinic effect ofcholinergic agonists. Direct-acting cholidergic drugs include: methacholine, carbachol, bethanecol and pjlocarpine. Indirecl-acting cholin€rgic drugs (cholikesterase inhibltor.r/ includc: physostigmine, edrophonium, neostigmine. pyridostigmine, malathion, echothiophate, sarin and soman. Effects of anticholinesterases: Muscarinic: . . Miosis Salivation 'Sweating ' . Bradycardia . Bronchoconstiction .lncreased Urination GI motility Nicotinic: . Muscle twitching . weakness . Tachycardia . Increase in BP Antimuscarinic drugs block the effect ofacetylcholine and all drugs that stimulate muscarinic rcceptors. Atropine and scopolamine arc prototlpes. Other antimuscarinic drugs include: glycopltolate, propantheline, benztropinc, cyclopcntolate, tropicamide, trihexyphenidyl, homatropine, oxybutynin, and rpralroplum. Antimuscarinic drugs produce the following effects: . . . . . . . Salivary glandsr red\tced secretion (atropine) GI tract: reduced peristalsis, reduced secrelion Su eat glands: reduced sectetion (atropilte) (ptupa theli e, g6\:opvrrolate) E1-es: mydriasis /lotuatropine, clclopentolale, tropicamide) Bronchi: bronchodilation, reduced secretion Bladder: urinary retcnt\or (oq'but)nin) C\S: antimotion sickness 6copolamine) antitremor activity Noreq; -to (ipratropium) treat parkinsonism fbenztrcp[ e, trihe$'phenidyl) l. Coohaindications for using antimuscarinic drugs include: narrow-angle glaucoma, prc- static h)?crplasia, and tachycardia. 2. Pilocarpine and cevime\ne (both cholinergic dsorTr.J/t are used to stimulate salivary flow 3. SAL-TROPINElatropine sulfate, USP tablelt is indicatcd to reduce salivation and is endorsed with the ADA Seal ofAcceptance

Antibiotic regimens for patients with prosthetic implants include all of the following drugs as standard therapy of fitst choise in patients not allergic to penicillin EXCEPT Cephalexin Cephradine Amoxicillin Tetracycline

Tetracycline From the "Advisory Statement, Antibiotic Prophylaxis for Dental Patients with Total Joint Replace)nent" published by the American Dental Association and the American Acad- emy' ofOnhopedic Surgeons, in the J Amer Dent Assoc 1997; 128(7):1004-8 and J Amer Dent Assoc 2003;134:895-898 -either cephalexin, or cephradine or amoxicillin can be used as the drugs ofchoice lor standard prophylaxis medication in the patient with a total joint replacement. . Cephalexin (Keflex) is a lst generation cephalosporin; prophylaxis dose is 2 grams orally I hour prior to the dental procedure. . Cephradine (Velose, is a lst generation cephalosporin; prophylaxis dose is 2 grams orally t hour prior to the dental procedure. . Amoxicillin is a mernber of the penicillin family; prophylaxis dose is 2 grams orally t hour prior to the dental procedure. \ote: For patients not allergic to penicillin and unable to take oral medications the suggested medications are cefazolin ber of the penicillin family). f4 lst generation cephalosporin) or ampicillin (mem- The regimen is as follows: . Cefazofin (Ancefl 1 gor ampicillin 2 g intramuscularly or intravenously t hour prior to the dental Drocedure

Which antimicrobial below is associated with photosensitivity Tetracycline Penicillin VK Metronidazole (Flagyl) Clindamycin

Tetracycline The photosensitivity reaction caused by the tetracycline family of antibiotics results in the appearance ofred rashes or blotches over the skin in the presence of sunlight. Common adv€rse effects caused by the other agents: . Penicillin VK: hypersensitivity resulting in skin rash and rare anaphylaxis . . Metronidazole (Flagyl): dizziness, headache, and nausea Note: Metronidazole is not a true antibiotic since it is not found in natural organisms; it is a synthetic substance produced in the chemical laboratory. Clindamycin: diarrhea, abdominal pain; known to cause pseudomembranous colitis

Which antibiotic exerts its antibacterial activity through inhibition of protein synthesis in the bacterial cell Cephalosporins Ampicillin Tetracycline Penicillin G Penicillin VK

Tetracycline The tetracyclines are a goup of broad-spectrum, bacteriostatic antibiotics that inhibit protein synthesis in the susceptible organism by binding to the 30S ribosome subunit, thereby impeding the binding of aminoacyl IRNA to the receptor site on the messenger RNA ribosome complex. The inlibition ofthis ribosomal function interferes with the attachment ofthe growing amino acid chain thus preventing complete formation ofpeptides from the ribosome. Since no peptides are formed, no proteins are formed. Since proteins are necessary for the bacterial cell to metabolically function, the lack thereof will cause a static state in which the bacterium becomes vulnerable to phagocytosis by the body's im- mune system. T€tracyclines are useful in treating the following infections: . . . . . Medical infections caused by susceptible gram-positive and gram-negative bacteria Infections caused by Mycoplasma, Chlamydia Exacerbations of chronic bronchitis L)ryne disease Treatment ofacne . . (caused by Bon'elia burgdorferi) , Protozoa or Rickettsia Treatment of gononhea and syphilis in patients allergic to penicillin Dental: treatment ofperiodontitis associated with the presence of Actinobacillius actinomycetemcomitans (AA) (i.e., Iocalized aggressive periodontitis) Important: Tetracyclines are not the drug of choice for Streptococcus or Staphylococ- cus.

Which of the following refers to the efficacy of a drug? The relative concentrations of two or more drugs that produce the same drug effect The ability of a drug to produce a desired therapeutic effect regardless of dosage The dose of a drug that will kill a patient

The ability of a drug to produce a desired therapeutic effect regardless of dosage Efficacy refers to the number of receptors that must be activated to yield a maximal response. A drug with high efficacy needs to stimulate only a small percentage ofreceptors, whereas a drug with lesser efficacy (but still considered to be a a larger proportion of receptors. full agonist) has to activate Factors Goyerning Drug Action: Two factors that determine the effect of a drug on physiologic processes are aflinity and intrinsic activity. Affinity is a measure ofthe tightness that a drug binds to the receptor. Intrinsic activity is a measue ofthe ability ofa drug once bound to the receptor to generate an effect activating stimulus and producing a change in cellular ac- tivity. Potency is the relative concentrations of two or more drugs that produce the same drug effect. The effect usually chosen is 50% ofthe maximal effect and the dose causing this effect is called the EC56. Potency is determined mainly by the affinity of the receptor for the drug. Note: The smaller the 8C56, the greater the potency of the drug. Potency is a comparative term (o e drug is more polent than another drug). Exampl€: Drug #l in a dose of l0 mg produces the same magnitude ofresponse as Drug #2 in a dose of 50 mg. The following is true: Drug #l is flrve times as potent as Drug #2. Also: tf Drug #l has a great€r eflicacy than Drug #2, then Drug #l is capable ofproducing a greater maximum effect than Drug #2. When comparing drugs with respect to intensity of response, the drug that produces the greatest maximum effect is the one with the highest ellicacy.

Antipsychotic include all o the following EXCEPT The phenothiazines The thioxanthenes The butyrophenones The benzodiazepines

The benzodiazepines Antipsychotics, also known as major tranquilizcrs orneuroleptics, are used commonly in the treatmcnt of a variety ofpsychotic disorders. By fa. the most widely used group ofantipsychotic agents uscd in medicine are the phenothiazines, followed by the butyrophenones and the thioxanthenes. The first antipsychotic on thc markct, chlorpromazine, is the key drug among the phenothiazine an- tipsychotic agents. Currently, antipsychotics are divided into two gcncrations. The first generation in- cludes thc older, "typical" drugs that treat the positive but not the negative symptoms associated with a psychotic state. Second-generation drugs have far fewer extrapyramidal symptoms and tardive dyskinesia and they are used to treat both positive and negative symptoms ofschizophrenia. Note: With the exception ofrispcridone, th€y are prolactin sparing. fDJ, The exact mechanism of antipsychotic drug action is unknown. These drugs are thought to work by blocking postsynaptic dopamine receptors in the h,?othalamus, basal ganglia, limbic system, bminstem, and medulla, and to some extent serotonin receptors- Typicaf antipsychotic drrgs (frst fEPt generationJ are more potent antagonists ofD2 dopamine receptors than ofDr receptors. First generation drugs include the phenothiazines (chlorpromazine, fluphenazine, perphena:ine, procltlorperazine, trifluoperazine, mesoridazine, and thioidazine), the thioxanthines ( thiothixene) and the btrytophenor\es (haloperidol). \e\\er (second generallor/ or atypical antipsychotic agents affcct different receptor sites compared * ith first generation antipsychotics. They bind dopamine, including D1, D2, Da, and D5 receptors, with selecrilely lor limbic dopaminc rcceptors. They have increased affinity for serotonin /J-11I, receptors compared $ith D" receptors. They exhibit reduced ability or an inability to induce EPS. Second gener- alion agcnts include clozapine, risperidone, olanzapine, quetiapine, ziprasidone, and aripiprazole. Important: |. Aripiprazole is the first ofa new class of atypical antipsychotic agents called dopamine sistem stabilizers or dopamine partial agonists.It combines the actions ofD2 and serotonin 5-HT2a ^ripipr^zole recePtor antagonism. :. Clozapine is specific for limbic rec€ptors and not for striated (muscle) r.ceptors,which explains the low incidence ofEPS andTD.

All are ccepted sites for IM injections EXCEPT The buttocks The biceps muscle The deltoid muscle The anterior thigh

The biceps muscle For young children, the anterior thigh is usually the place to give IM injections. Absorption from an intrarnuscular injection is often faster and there is a higher bioavail- abilit]' than with oral administration. Proper depth of needle in muscle: In big muscle (adult) go in one inch; in children go in one-quarter of an inch. Never go beyond two-thirds ofthe needle length.

Levodopa is used in the treatment of parkinsons disese to replensih the brains supply of serotonin, the neurotransmitter thats deficient in this disorder Administering carbidopa in addition to levodopa reduces the required dose of levodopa by about 75% The first statement is true; the second statement is false The first statement is false; the second statement is true Both stalements are true Both statements are false

The first statement is false; the second statement is true *** Levodopa is used in the treatment of Parkinson's disease to replenish th€ brain's supply ofdopamine, the neurotransmitter thal is deficient in this disorder Parkinson's disease ergic actilitr in the basal i/PD) results from a relative excess ofcholinergic activity and a deficiency ofdopamin- ganglia. It is a chronic, debilitating disease with no kno$'i cure. Drug treatment for PD has cenrered on increasing the availability ofdopaminc in the CNS, inhibiting the effscts ofacetylcholine, and a$cmpring to prevent further cell m€mbmne damage through neuroprotective trials. Note: The D2-recep_ :..r subllTre is lhe p mary modulatorofboth clinical improv€ment and adverse rcactions such asdystonia and hrllucinations. Lerodopa has been thc single most important drug in the treatment ofPD. Administering carbidopa in com- reduces lhe required dose of levodopa by about 75%. when levodopa is grr er alone. much ofrhe dose is metabolized befor€ the drug reaches the brain. Therefor€, large doses are re- bination \r'ilh levodopa fsir?emet, qurf3d. and these are apt to caus€ unwanted side-effects. Carbidopa inhibits the pcripheral decarboxylation of lerr.dopa. This action simultaneously reduces the likelihood ofperipheml side effecls and allows more lev- riopa ro reach the brain. Since carbidopa does not cross the blood-brain barrier, the levodopa in the brain is conlened rhere to dopamine. Thus, co-administration ofcarbidopa plus levodoPa in the form ofSinemet al_ lo\\s a significant reduction oflevodopa dosage without rcducing the desired effccts. Other rntiparkinson agents: . llfonoamine oridas€ B (MA)-B) inhibitorsr seleglline (El.lepy, and rasallllne (Azilect).T1rese drugs ir- reversibly inhibir the enzyme MAO-B, which is responsible for the oxidative deamination ofdopamine in the brain. This causes dopamine to accumulate in surviving nerve cells and reduces the symptoms ofPD . . Gfutamate antagonist (anliirury amartadine ($'mmetrel) appearc to potentiate dopamineryic responses. Dopamine agonists: bromocriptine, pcrgolide, apomoryhtne (Apokrn), pramipexole (Mirupex), and ropinitule . (Requi . These d gs are direct dopamine receptor agonists. Anticholinergic agents (antimuscarinic drugs): benztropine (Cogentin) and ftlhexyphentdyl (Tasmar). These drugs supprcss central cholinergic activity and may inhibit reuptakc and storage ofdopamine in the CNS. rhus prolonging the actron of dopamine. . Catechol-O-methyl t r^osfer^se (COMT) ilhibitorsi tolcapone i1zQsma, drugs are inhibitors ofCOMT, another enzlme that metabolizes dopamine. . and entacapone Certain antidepressrnts and antihistamines such as diphenhydramine that have antimuscarinic actions may be given in the early stages ofdisease. fcotntan) T1\ese

Nitrous oxide is used as a single agent to produce general anesthesia Nitrous oxide is used to produce sedation and mild anesthesia The first statement is true; the second statement is false The first statement is false; the second statement is true Both statements are true Both statements are false

The first statement is false; the second statement is true Nitrous oxide is unable to produce general anesthesia except ifit is given at concentrations greater rhan 80q'o. At thesc concentrations, thc lack ofoxygen would cause hypoxia in the paticnt. lnhalant aneslhetics such as halothane and isoflurane can produce general anesthcsia al concentrations ap- pro\imating 3-5%. As such they are very useful in anesthesia. \rtrous is used to produca sedation and mild analgesia. It is usually used in concentrations of30- 509'o along rvith pure oxygen. lt is a colorless, nonirritating gas at room tempemture and prcssuc, and is not flammable nor cxplosivc. The onset of sedation is within 5 minutes and the recovery is lu:t as rapid. lt is excreted unchanged by th€ lungs. The most common complaint from patients taking nitrous oxide is mild nausea. \ote: -\l*ays give patient 10070 oxygen at thc end ofthe pfocedure to prcvent diffusion hypoxia. -\dlerse effects ofnitrous oxide: . Decreased mental performance, audiovisual ability, and manual dexterity ..A.t high doses and/or high exposure: reduced t'ertility, spontaneous abortions, neurological and lrdney disease as well as bone marrow suppression Nrlrous oxide is contraindicated in patients: . . . . . . . . . Head injury With bowel obstruction Pneumothorax Middle ear and sinus disease With upper respiratory infections COPD (emph),sema ol bt'onchitis) In the first trinester ofprcgnancy With whom communication is difficult fi.e., autistic ptttietlts) Having a contagious disease since it is difficult to sterilize entire tubes Important; Prolonged exposure (e.g., morc than 24 hou,'s./ causes bone marrow depression.

Drugs travel through the blood stream by binding to albumin plasma protein However only a small number of drugs travel through the bloodstream by this mechanism The first statement is true; the second statement is false The first statement is false; the second statement is true Both statements are true Both statements are false

The first statement is true; the second statement is false Most drugs travel though the blood stream by binding to albumin protein, which is abundant in plasma. In this way, drugs can be carried to all the tissues and organs. A dmg which is bound to plasma albumin always has some fraction which is not bound. The unbound portion is free to leave the blood compartment to be taken up by tissues where the drug will elicit its pharmacological ellect. The remaining bound fraction of drug then aontinuously releases more free drug to be taken up by tissues. Eventually all drug in the blood compartment will be taken up by this process. lmportant: Interactions betw€en two or more drugs can occur ifthey compete for binding on the plasma albumin. If drug A is bound to albumin prior to the patient taking drug B, and drug B has a geater binding affinity to albumin than drug A, then when drug B is taken, it will displace drug A from albumin to result in large amounts ofunbound drugAwhich could lead to adverse reactions due to the sudden large amounts gaining access to the tissues. One ofthe most fundamental aspects of drug action is the relationship between dose adminis- tered and the effect obtained (drugs are dose dependent), Dose and response are related and can be represented by a dose-response curve. There are two basic types ofdose-response curves: . The graded dose-response owm). Estto- gstrogens progestin agent. (Dn, curve plots the degree ofa given response against the con- centration ofthe drug. These curves are useful for determining characteristics ofagonists and antagonists. . The quantal dose-elTect curve: In this case, a given quantal eIlect is chosen (e.9., .t certain degree of cough suppression), and the concentntion ofthe drug is plotted against the percent- age of a specific population in which the drug produces the effect. The median effective dose (8D50 or the dose at which 50%o of the individuals exhibit the specifed quantal effecr) and the median lethal dose (2D50 or the dose at which death is produced in 50'% oJ the expelimental animals in preclinical sludies) canbe estimated from quantal dose-eIlect curves. With this tlpe of curve, the relative effectiveness ofvarious drugs for producing a desired or undesired effect, as well as the relative safety between various drugs, can be determined.

Amiodarone (cordarone) is the most potent and broad spectrum antiarrythmic compound currently available Procainamide is an antihypertensive drug used to treat high blood pressure The first statement is true; the second statement is false The first statement is false; the second statement is true Both statements are true Both statements are false

The first statement is true; the second statement is false Procainamidc is a Class lA antianhythmic agcnt that is uscd in thc trcatment ofseveral cardiac ar- rhythmias including atrial fibrillation, atrial flutter, parcxysmal atrial tachycardia. and ventricular tachycardia. It is not used as an antihypertensive. It is a derivative ofthe ester local anesthetic pro- caine. Procainamide has properties similar to those ofother Class lA agcnts, quinidine and disopyramide. These drugs decrease myocardial conduction velocity, excitability, and contractility by inhibiting the influx of sodium through "fast" channels oI the myocardial cell membrane, thereby increasing the rccovery period al1er repolarization. Note; is prirnarily used to treat supmventricular tachyanhythmias. Remember; Generally speaking, the use ofbeta-blockers Quinidine (Class II agents) as antianhythmics is reserved fbr patients who require only control ofventricular ratc during atrial tachyarrhythnias or who have mildly symptomatic vent cular arrhythmias. Side effects includc bradycardia and hy- potension. Important: Amiodarone (Cordatone) is generally considered a Class Ill agcnt even though it also blocks sodium channels, a Class I action. It is uniqu€ in that it is the most potent and "broad-spec- trurn" antianhythmic compound cunently available. It blocks sodium, calcium, and potassium channels as rvell as beta rcceptors. Il has impressive ellicacy in suppressing both supraventricular and \ enlricular arhythmias. Note: Adverse effects include pulmonary fibrosis, thyroid abnormaliti!-s. skin discoloration, and peripheral neuropathy. \ot.s L Cornmon adverse effccts ofsome antiafihythmics: . Procainamide: nental changes, torsade de pointes (TDP - i.s an uncontnutn vari- a t ofvetlti'icul4r |acht-cqrdia) . Quinidine; hypotension, cinchonism torsade de pointes . Lidocaioei convulsions . Propranolol; bronchoonstriction, heart block @ eadache, ringing in lhe ears, deafness), awl . Calcium channel blockers: flushing, headache, hypotension, gingival hyperplasia and reduced cont.actilitv ofthe heart

The term bioavailability of a drug refers to The movement of a drug into the body tissues over time The dissolution of a drug in the GI tract The measurement of the rate and amount of therapeutically active drug that reaches the systemic circulation The relationship between the physical and chemical properties of a drug and the systemic absorption of the drug The amount of drug destroyed by the liver prior to systemic absorption from the GI tract

The measurement of the rate and amount of therapeutically active drug that reaches the systemic circulation he bioavailability of a drug is allected by: . The dissolution ofa dnrg in thc Gl tract . The destnrction ofa drug by the liver Phrrmrcokinetics focuscs on the lisn), and cxcr.tlon (elim . proccsscs concemed with absorption, distribution, biotransfomation r'nalior) of drugs. Absorption: thcre are four prim.ry frctors that must be considered in evaluating drug absorption: l. Drug chxncteristics: . . Formulrtion ot the drug Co[centration of the dru$ the higher the concentration, the more quickly the drug is absorbcd . Lipophilic drug formulrtions are morc rcadily rbsorbsble: nonionized drugs are more lipid soluble and may readily difuse across cell membmn€s . Acidic drugs become nonionized in the acidiry ofthe stomach .nd then dilluse across mcmbranes: basic drugs tend to ionize and arc not well absorbed in the stomach 2. Routes of edministration: the most common routcs for giving medications include oral, topical, subcuta- ncous, infamlscular, intravenously, and rectal. 3. Blood flowi circulation at the sitc ofadministration is importrant in the drug absorption circlJl^tion (as see'n fu congestiw heart fail rc) vtill result in dccrcascd drug absorptlon. 4. Cell membrrne chtractedstics: dtugs cross membranes via passivc diffusion or active tmnsport . Distribution: is thc trrnsport ofa dnrg in body fluids from thc bloodstesm ous tissues in thc body . Biotranstormation a/n (at Foccss. (metabo' Dc:crcased lhe sile ofabsorptiotr) to vai- etabolism)t is the chemical inactivation of a drug lhrough convcrsion to a more waler-sol- uble compoud that can be excreted from th€ body. It occurs primarily in the liver. Involves two major stcps: Phas€ I makcs the drug morc hydrophilic through oxidation, reduclion, or hydrolysis. Phase lI is called curonidrtion, it involves conjugrtion. . Eliminrtio[: is the proccss by which drugs and their metabolites are removcd from the body. The live. 8nd the kidney are wo major organs responsible for elimination. Most climination occurs through excrction by thc kid- neys. Thc proc€sses involved in r€nal elimiDation consist ofglom€rular liltrution, tubuhr secretion' rdd par- tirl rerbsorDtion.

Local anesthetics should theoretically be less effective in acutely inflamed tissues than in normal tissue because in inflamed tissue The pH rises, thus inactivating the anesthetic The pH rises, thus decreasing available free base The pH decreases, thus decreasing available free base The pH remains the same, the extracellular fluid dilutes the anesthetic

The pH decreases, thus decreasing available free base At body pH (7.4), a local anesthetic when infiltrated, will chemically exist as a portion which is ionized (has a proton attached) and as a portion which is non-ionized (ha.s proton attqched). Note: A proton is nothing more than a hydrogen (H.) alom. That portion which is ionized has difficulty penetrating the nerve and will not be effective. That portion which is non-ionized will penetrate the n€rve to cause anesthesia. That portion which is non-ioniz€d is also known as the free base. The more proportion ofthe anes- thetic which is in the free base form. the more effective it will be. When tissue conditions are normal (p11 7.4), approxrmately 10-207o portion ofan infil- trated local anesthetic is in the form of the free base (non-ionized This is enough to penetrate the nerve to cause anesthesia. When tissues are acidic, as in the case of tis- form). sue infection, less free base portion exists and more ionized portion is present. There is not enough free base form to penetrate the nerve to cause anesthesia. Therefore, the local anesthetic when infiltrated to the tissue site is not effective at the normal anesthetic doses.

Pharmaceutical agents that bring about tissue responses resembling those produced by stimulation of the sympathetic nervous system are called? Cholinomimetic Antiadrenergic Parasympathomimetic Sympathomimetic

The terms sympathomimetics, syrnpathomimetic amines, adrenergic agonists and adrenergic agents are almost always synonymous. Epinephrine is the sympathomimetic agent used in dentistry. It is the vasoconstrictor for anesthetic solulions and the vasoconstrictor component in gingival retraction cords. As a component of local anesthetic preparations, it is used to prolong the duration of local anesthesia. In medicine, sympathomimetics are used as pressor agents to maintain blood pressure in vascular shock. They are used as bronchodilators for asthma attacks and for allergic states including anaphylactic shock. Syrnpathomimetics used in medicine include dopamine, epinephrine, norepinephrine, isoproterenol, and phenylephrine. Important: Epinephrine is indicated in medicine to treat bronchospasm and hypersen- sitivity reactions. It is the agent of choice for reversing anaphylactic reactions. lt is used to restore cardiac activity in cardiac arest.

What do the following penicillin antibiotics have in common Amoxicillin and clavulanate potassium (Augmentin) Ampicillin and sulbactam (Unasyn) Cloxacillin Dicloxacillin

Theyre all penicillinase resistant ntibiotics A functional part ofthe chemical molecule of all the penicillins is the so-called betalactam ring, which is a four-membered imbedded ring structure consisting ofthree carbons and one nitrogen atom which is responsible for the antibacterial activity of penicillins. Any alteration to the beta-lactam ring will also alter the antibacterial activity. Penicillinase is an enzyme secreted by bacteria which splits open the betalactam ring. This renders the penicillin molecule ineffective against those penicillinase secretors. Cloxacillin and dicloxacillin resist the actions ofpenicillinase because they have a pro- tected beta-lactam ring which prevents the actions ofthe enzyme. Augmentin and Unasyn contain the agents clavulanate potassium and sulbactam re- spectively which block the actions ofpenicillinase from reaching the betalactam ring. The majority ofpenicillins are directly excreted into the urine through renal tubular cell secretion. Probenecid (Benemid), an inhibitor of renal tubular cell secretion raises the blood ler els ofthe penicillins by diminishing their tubular secretion. Probenecid is sometimes given simultaneously with penicillin to raise the blood levels for increased activ- irr. \ote: Probenecid is a drug used to treat gout.

All these drugs are classified as what drugs Lepirudin (Reflidan) Danapnord (Orgaran) Bivalirudin (Angiomax) Argatroban

Thrombin inhibitor type anticoagulants These agents are administered intravenously for prevention ofpost-operative deep vein thrombosis following elective hip replacement surgery; for prophylaxis or treatment of thrombosis in adults with heparin-induced thrombocytopenia. Their mechanism ofaction is through the direct inhibition of thrombin within the coagulation pathway, thus inhibiting fibrin formation.

Which route of drug administration is generally used for local drug effects Topical administration Oral ingestion Subcutaneous injection Intravenous injection

Topical administration Common routes of administration ofdrugs: 1. Parenteral administration . . . Intravenous (lV) Intramuscular Subcutaneous - (not by wa!" ofintestine or GI tract) diectly into the bloodstream. Acts very rapidly. |1M/ - injected into a muscle area, where it is promptly absorbed injection beneath the skin. Absorption may be less rapid. 2. Enteraf administration . . @y v,q) ofthe intestine or GI tract) Buccal or sublingual a tablet is placed under the tongue or in the cheek Or^l- (most common route), the drrrg is swallowed. It is the most convenient for safe drug administration. It is safe, painless and economical. . Rectal tum 3. Inhalation - the drug in solution (enema) or suppository form is inserted into the rec- - the drug is given as an aerosol into the respiratory tract. 4. Topical administration the drug is placed on the skin for a local effect. 5. Transdermal - the drug is placed within a "patch" and placed on the skin to be ab- sorbed into the systemic circulation.

Monoamine Oxidase (MAO) inhibltors are often used as third-line agents in cases of refractory and atypical depression. Which Two drugs below are MAO inhibitors? Doxepin (Sinequan) Tranylcypromine (Parnate) lmipramine (TofraniI) Phenelzine (Nerdil)

Tranylcypromine (Parnate) Phenelzine (Nerdil) MAO inhibitors are used are often used as third-line agents in cases ofrefractory and atypical depression. MAO inhibitors antagonize the action of monoamine oxidase (MAO) re- sponsible for the degradation of the naturally occurring monoamines norepinephrine, dopamine, and serotonin. It is theorized that the increased level of monoamines in the brain is responsible for the antidepressant effect ofthe MAO inhibitors. Note: Local anesthetics containing epinephrine are genemlly contraindicated in patients who are taking MAO inhibiton. The major limitation for the widespread use of the MAO inhibitom for the treatment of de- pression has been the potential for serious adverse side effects, including hypertensive cri- sis. rvhich can be fatal. A hypertensive crisis can occur within several hours after ingestion of a substance that contains tyramine. Tyramine releases norepinephrine and other sympath- onrimetic amines, thereby raising blood pressure. Early symptoms include occipital headache, palpitations, stiff neck, nausea, vomiting, and sweating. Among the drugs that interact with \1AO inhibitors are meperidine (Demerol), epinephrine and ephedrine. l. Miscelianeous antidepressants include: \ot€ . . . Mirtazapine Bupropion (Remeron): alphaz -noradrenergic antagonnist (Wellbrr'tu ,SZ/: norepinephrine / dopamine rer.rptake inhibitor - Trazodone (Desl,rel): mechanism not clear; possible serotonin reuptake in- hibitor

In dentistry a cholinergic agent is used Produce a dry field for taking impressions Calm an anxious patient Treat dry mouth by inducing salivation Reduce nausea

Treat dry mouth by inducing salivation Cholinergic drugs used in dentistry are: Pilocarpine (Sa/agezl, and Cevimeline . Pifocarpine (Salagen) - is indicated for treatment ofxerostomia caused by salivary gland hypofunction resulting from radiotherapy for cancer of the head and neck. It's pharmacologic category is cholinergic agonist. Prevalent side effects are excess sweat- ing and nausea./heartbum/diarrhea due to the cholinergic nature ofthe drug. . Cevimefine (Evoxac) - is indicated for the treatment of symptoms of xerostomia in patients with Sj<igren's syndrome. It's pharmacologic category is cholinergic agonist. Prevalent side effects are increased sweating and nausea/heartburn/diarrhea due to the cholinergic nature ofthe drug.

The maximum reconmended dose of a local anesthetic that can be administered to a child <10 years of age is determined by Age Weight Height Gender

Weight Erample: For Lidocaine (2%) wrth epinephrine, a dosage of 4.4 mg/kg should not be exceeded (wL\imum :{ote: I kg = 2.3 lbs is 300 ng). 2Yo: 20 mg I mLx 1.8 mL /l carpule = 36 mg/l calpule

Which two groups of antidepressant drugs has the highest incidence of dry mouth (xerostomia) Tricyclic antidepressants (i.e., Elavil) Selective serotonin reuptake inhibitors (i.e., Prozac and Paxil) Serotonin and norepinephrine reuptake inhibitors (i.e., Effexor and Cymbalta) Monoamine oxidase inhibitors (i.e., Nardil)

Tricyclic antidepressants (i.e., Elavil) Serotonin and norepinephrine reuptake inhibitors (i.e., Effexor and Cymbalta) These two categories ofantidepressant drugs induce significant dry mouth in:up to 75yo ofpatienls taking these medications. These effects are due to the secondary anticholin- ergic nature ofthese agents. Note: Drug-induced dry mouth must be treated palliatively with artificial salivary substitutes. The seldctive serotonin reuptake inhibitors (i.e., Prozac, Paxil) and monoamine oxidase inhibitors have no secondary anticholinergic effects and therefore do not cause any sig- nificant dry mouth.

All of the following drugs are useful for treating which disease Isoniazid Streptomycin Rifampin Ethambutol Pyrazinamide

Tuberculosis Tuberculosis is a bacterial infection caused by Mycobacterium tuberculosis. The antitubercular drugs either inhibit the growth ofthe bacteria or kill the bacteria. Because the \llcobacterium organism tends to develop resistance to any single antitubercular drug, combination drug therapy is standard in the treatment oftuberculosis. .A.ntitubercular drugs: . lsoniazid: often given in a four drug regimen along with rifampin, pyrazinamide and ethambutol . . . . Streptomycin: often given in combination with isoniazid Rifampin: usually given in combination with other agents Ethambutol: usually given in combination with olher agents Pl razinamide: popular in combination with rifampin ]Iechanism of action: . Isoniazid: is bacteriostatic for resting organisms and bactericidal for dividing organisms. It interferes with lipid and nucleic acid biosynthesis in growing organisms. . Streptomycin: is a bactericidal antibiotic. It acts by interfering with normal protein s)rlthesis. . Rifampin: inhibits DNA-dependent RNA-polymerase activity, thereby supressing RNA synthesis. It can be bacteriostatic or bactericidal and is most active against bac- teria undergoing cell division. . Ethambutol: impairs cellular metabolism, causing cessation of cell multiplication and cell death. It is bactericidal and is active only against Mycobacterium. . Pyrazinamide: the mechanism ofaction is unknown. It may be bacteriostatic or bac- tericidal against M. tuberculosis, depending on the concentration.

All are class II antiarrythmic drugs EXCEPT Propranolol (Inderal) Acetbutolol (Sectral) Metoprolol (Lopressor) Veraparnil (Calan)

Veraparnil (Calan) Thc \ aughan-williams classification system traditionally has been used to classity antianhyth- mic drugs. This scheme places the available agents into onc of four classes, usually denoted by Roman numcrals I lV This systcm is loosely based on the channel or receptor involvcd. Class l: Sodium channel blockers . Class I drugs are further classified on the basis oftheir effects on action potential duration: py'orpaceJ. These agents prolong thc action potential. . . lA agcnts include; Qu:Lnidinc |{il (Quinidex), procatnamide (Procan SR/ and disopyramidc fB agents include: LidocainelX.ylocai e). mexiletine (Mexilil) and tocainide (Tonocard. These agents short€n the action potential. . fC agents include: Flecainide (Tambocor) and propafenone (Rythnol). These agents have no effect on action potential dulalion. Cfass II: Beta-adrenergic blockers Propranolol (Inderal) is the prototype antianh)'thmic beta-blocker Other drugs in this class include: esmolol (Breibloc) acetbutolol (Secn'al) ar'd metoprolol (Lopressor). These agents increase refractory period, decrease conduction veloc- ity and reduce automaticity. Cfass IfI: Potassium channel blockers - Amiod^rone (Uasolec) (Cordarore) is the prototype drug in this class. Sotalol (Betapace), tbutilide (Corvert) and dofetilide (Tiko$n) arc also in this class. These drugs increase refractory period and reduce automaticity. Class Il': Calcium channel blockers Yerapamil (Calan)is the prototype. Diltiazem is also included in this group. These drugs increase refractory period, decrease conduction velocity and reduce automaticity. Note: Miscellaneous antiarrhythmic drugs include: adenosine (Adenocard) which increases refiactory period and reduces automaticity and digoxin (Lanoxin) which increases the force ofcontraction ofthe heart muscle and decrease conduction velocity.

All can produce drug dependence and addiction EXCEPT Codeine Morphine Hydrocodone Meperidine (Demerol) Oxycodone Voltaren

Voltaren Voltaren is a NSAID which do not produce drug dependence and addiction. The other analgesics belong to the opiate class of drugs. They are also klown as nar- cotic analgesics since the actions ofthis family is to cause drowsiness and sleep as a side effect. The opiates produce drug dependence leading to addiction. Psychic dependence, physical dependence, and tolerance can develop upon repealed administration. Psychic de- pendence is unlikely if an opiate is taken for a short period for pain relief. Physical dependence is a condition in which continued administration of the drug is required to prevent unpleasant withdrawal symptoms. Tolerance occurs when increasingly large doses ofopiate are required in order to produce the same degree ofanalgesia. Opiate drugs used in dentistry to provide pain reliefafter dental surgery include: . . . . *** Codeine Hy drocodone O)icodin) Oxycodone (Percocet rylox) Meperidine (Demerol)

Which of the following competitively blocks vitamin K-binding sites and inhibits the synthesis of vitamin K dependent coagulation factors VII, lX,X, and ll (prothrombin) and anticoagulant proteins C and S? Heparin Warfarin (Coumadin) Aspirin Anagrelide

Warfarin (Coumadin) At therapeutic levels, warfarin decreases liver synthesis of vitamin K-dependent clotting factors by 30o% to 50olo. These clotting factors have different half-lives. Factor WI has the shortest halfJife {6- t hr) vs. factor lI andx (up to 72 hr). Oral anticoagulants do not re- verse ischemic damage or lyse an established thrombus but rather prevent extension ofthe existing thrombus and the formation ofnew thrombi by blocking synthesis of clouing factors. Oral anticogulants are used after a myocardial infarction to prevent coronary occlusion, in the treatment ofpulmonary embolism, and in the treatment ofvenous thrombosis. l{ote: Enhanced anticoagulant effects are seen when these drugs are combined with as- pirin. For this reason, use acetaminophen (Tylenol) for pain control. Patients on anticoagulant therapy may have excess bleeding after dental treatment. Al- ways check medical history. If patient is on anticoagulants, his/her physician should have documentation of INR values to assess anticoagulant effects. INR stands for Inter- national Normalized Ratio and essentially is the ratio of the prothrombin time meas- ured in the patient divided by a standard prothombin time value, and multiplied by a constant. An INR value of I means normal prothrombin times of approximately 12 sec- onds; normal blood clotting would be present. INR values greater thar I indicate that there is an anticoagulant effect. The higher the INR value, the greater the anticoagulant effect. Many patients taking anticoagulants have INR values of2, 3 and even up through

All are toxic effects of mercury EXCEPT Iritability Xerostomia Loosened teeth Gum disorders Slurred speech Tremors

Xerostomia Excessive saliva is a prominent toxic effect ofmercury not xerostomia These symptoms are all of the chronic form, which results from the inhalation of the vapors ofdust ofmercurial compounds or from repeated ingestion ofvery small amounts. The presence of mercury in the body is determined by a urine test. Treatment may include gastric lavage with milk and egg white or sodium bicarbonate, chelation with British anti- lewisite (BlI), and fluid therapy. Not€: British Anti-Lewisite (BAL) or Dimerc prol and p€nicillamin€ are two drugs cur- rently marketed for promoting the excretion of mercury, lead, and several other agents. A few additional agents are available for the treatment of poisoning by metals other than mercury k.9., edetote colciunt disodium for lead and deferoxamine for iron). Mercury that is absorbed into the circulatory system may be deposited in any tissue. Higher-than-average accumulations occur in the brain, livel and kidney. Mercury does not collect irreversibly in human tissues. There is an average half-life of 55 days for transport through the body to the point of excretion. Thus, mercury that came into the bodl years ago is no longer present in the body. l. Children and adults wbo are to be treated for lead poisoning should only be foftr' given the Edetate Calcium Disodium (Calcium Disodium Versenate) form of .., ,._ :, "EDTA." 2. Penicillamine is also a highly effective chelator ofcopper and is ofprimary im- portance in the management of Wilson's drsease Thepatolenticulqr degeneratiotr). 3. Deferoxamine is a drug that chelates to absorbed iron very well and is elimi- nated in urine. 4. For carbon monoxide poisoning the fteatment is 100%o oxygen therapy volves breathing oxygen thxtugh a tight-ftting uask). 5. Cyanide poisoning can be treated with rapid oxygen administration and the an- tidotes sodium nitrite and sodium thiosulfate.

The following drugs are noted for causing what prominent oral side effect? Amitryptyline(Elavil) Diphenlydramine (Benadryl) Atropme Diazepam (Valium)

Xerostomia Xerostomia can be caused by certain drug classes that inhibit the production and secre- tion of saliva. . Amitriptyline (Elavil) is representative of the tricyclic antidepressants, a class of drugs lhat causes significant xerostomia. They probably work through an anticholin- ergic action. . Diphenhydramine (Bena&yl) is representative olthe sedating-type antihistamines, a class ofdrugs that causes significant xerostomia. They probably work through an an- ticholinergic action. . Atropine is a powerful anticholinergic which blocks the production of saliva in the salivary glands. Other anticholinergics will have a similar action. . Diazepam (Vctlium) is representative of the benzodiazepine tranquilizers. These drugs have moderate anticholinergic actions to reduce the outflow of saliva. \ote: The xerostornia actions produced by these classes ofdrugs are reversible with normal salivary flow regained after discontinuance ofthe drug.

All of the following drugs are benzodiazepines EXCEPT one. Chlordiazepoxide (Librium) Diazepam (Valium) Flurazepam (Dalmane) Midazolam (Versed) Alprazolam (Xanax) Zolpldem (Ambien) Tiazolam (Halcion) Oxazepam (Serax) Temazepam (Restoril) Clorazepate (Tranxene)

Zolpldem (Ambien) The benzodiazepines are used as oral preparations to alleviate anxiety and to inducc sleep, and intravenously to cause conscious-scdation for outpatient surgery. Benzodiazepines act by potentiating the action ofGABA, an amino acid and an inhibitory neurotansmitter, which results in increased neuronal inhibition and CNS depression. Tolerance and physical dependence can occur with prolonged high dosage. They seem lo be much saler than barbituratcs. Other use- ful properties include being an anti-conwlsant and a skeletal musclc relaxant. Pharmacological effects of benzodiazepines: antianxiety, sedation, anticonvulsant, amnesia, and skeletal muscle rclaxation. Adverse effects: . CNS e{Iects a cqr) . . - fatigue, slurred speech, drowsiness/sleepiness and conf'usion (do Gl effects dry mouth, nausea Other effects hypotension, muscle relaxation Benzodiazepines used orally as tmnquilizen: chlordiazepoxide irn). alprazolam somnia: flurazepam (Libtittm), di.azepam (Xonax) and lorazepam (Ativan).Those used as hypnotics to overcome in- (Dalmane) andtiazolam (Halcion). Drazepam (Valium) ar'd Clonazepam tKlonopin can be used as anticonvulsants. Midazolam (Ilersed) comes as a liquid for preoperative sedation in children and as injectable for IV conscious sedatron. ) \ote: Benzodiazepines should never be taken with any form ofalcohol. Serious potentiation ofthe sedative eflects ofeach will occur leading to unexpected inebriation and respiratory de- presslon. Important: Zolpidem (Ambien), age s (cdlled eszopiclone (Lunesta), and zaleplon not drive (Sonata) are anti-insomnia GABA-BZ Agonisls).These agents appear to act through the potentiation of GABA on benzodiazepine receptors, especially the omega-1 subunit. Note: They have shown no poten- tial for causins addiction.


Conjuntos de estudio relacionados

anatomy and physiology chapter 4

View Set

Chapter 7 Imperialism and the Spanish American War

View Set

Giddens Concepts Combined (RNShade)

View Set

Chapter 5 Exam (Earthquakes Throughout the U.S. and Canada)

View Set

Market-Based Management Chapter 5: Market Segmentation and Segmentation Strategies

View Set